You are on page 1of 124

Basic Concepts of Geometry 5

1 Basic Concepts of Geometry


(2) We say line is a set of points which can be
Points to Remember: extended in both directions infinitely and
indicated by arrow heads.
 Distance between two points on a number l
line: P R Q
Distance between two distinct points can be (3) We can name the line in two ways.
calculated by subtracting smaller coordinate (i) By using names of any two distinct points on a
from the bigger coordinate. line. (Example: Line PQ or line PR or line RQ)
X W M L P A B C D E (ii) By using single small alphabet.
–4 –3 –2 –1 0 1 2 3 4 5 (Example: Line l)
Example: • Plane:
(1) Co-ordinate of point A is 1 (1) Plane is an undefined term in geometry.
Co-ordinate of point D is 4
(2) We say ‘plane’ is a flat surface which extends
4>1 infinitely in all directions.
d(A, D) (3) We can name a plane in two ways:
= Greater co-ordinate – Smaller co-ordinate
(i) By using names of any three
= 4–1
E
non-collinear points in a plane. P
Q

\ d(A, D) = 3 Units (Example: Plane PQR) R

(2) Co-ordinate of point M is – 2 (ii) By using a single capital letter.


(Example : Plane E)
Co-ordinate of point X is – 4
–2>–4 • Collinear and Non-collinear points:
d(X, M) If there exist a line passing through three or
= Greater co-ordinate – Smaller co-ordinate more distinct points, then the points are called
= – 2 – (– 4) = – 2 + 4 collinear points, otherwise they are called
‘non-collinear points.’
\ d(X, M) = 2 Units
Example:
• Point: (1)
(1) Point is an undefined term in geometry.
A B C D

(2) We say point is a dot made by a sharp pencil In above figure, points A, B, C, D are collinear
on a paper. points.
(2)
(3) It is represented with the help of a single capital Q R
alphabet.
S
The point A is shown as:
P
In above figure, points P, Q, R, S are non-
•A
collinear points.
Note:
 Distance and Betweenness:
(i) Point is not a figure in geometry, it just shows
the position. • If P, Q and R are three collinear points and
if d(P, Q) + d(Q, R) = d(P, R) then point Q is
(ii) Point does not have length, breadth and height.
between P and R.
• Line:
(1) Line is an undefined term in geometry. P Q R

(5)
6 Master Key Mathematics - II (Geometry) (Std. IX)

(iv) d(J, H)
MAstEr KEy QUEstioN sEt - 1
solution:
PrACtiCE sEt - 1.1 (Textbook Page No. 5) The co-ordinate of point J is –2.
The co-ordinate of point H is –1.
(1) Find the distances with the help of the number
line given below: –1 > –2
Q P K J H O A B C D E d(J, H)
–5 –4 –3 –2 –1 0 1 2 3 4 5 = Greater co-ordinate – smaller co-ordinate
= –1 – (–2)
(i) d(B, E)
= –1 + 2
solution:
= 1
The co-ordinate of point B is 2.
\ d(J, H) = 1 unit
The co-ordinate of point E is 5.
5>2 (v) d(K, o)
d(B, E) solution:
= Greater co-ordinate – smaller co-ordinate The co-ordinate of point K is –3.
= 5–2 The co-ordinate of point O is 0.
= 3 0 > –3
\ d(B, E) = 3 units d(K, O)
= Greater co-ordinate – smaller co-ordinate
(ii) d(J, A)
= 0 – (–3)
solution:
= 0+3
The co-ordinate of point J is –2.
= 3
The co-ordinate of point A is 1.
\ d(K, o) = 3 units
1 > –2
d(J, A) (vi) d(o, E)
= Greater co-ordinate – smaller co-ordinate solution:
= 1 – (–2) The co-ordinate of point O is 0.
= 1+2 The co-ordinate of point E is 5.
5>0
= 3
\ d(J, A) = 3 units d(O, E)
= Greater co-ordinate – smaller co-ordinate
(iii) d(P, C) = 5–0
solution: = 5
The co-ordinate of point P is –4.
\ d(o, E) = 5 units
The co-ordinate of point C is 3.
(vii) d(P, J)
3 > –4
solution:
d(P, C)
The co-ordinate of point P is – 4.
= Greater co-ordinate – smaller co-ordinate
The co-ordinate of point J is – 2.
= 3 – (–4)
–2>–4
= 3+4
d(P, J)
= 7
= Greater co-ordinate – smaller co-ordinate
\ d(P, C) = 7 units
= – 2 – (– 4)
Basic Concepts of Geometry 7

= –2 + 4 (iii) x = –3, y = 7
= 2 solution:
\ d(P, J) = 2 units The co-ordinate of point A is –3.
The co-ordinate of point B is 7.
(viii) d(Q, B)
7 > –3
solution:
d(A, B)
The co-ordinate of point Q is –5.
= Greater co-ordinate – smaller co-ordinate
The co-ordinate of point B is 2.
= 7 – (–3)
2 > –5
= 7+3
d(Q, B)
= 10
= Greater co-ordinate – smaller co-ordinate
\ d(A, B) = 10 units
= 2 – (–5)
= 2+5 (iv) x = –4, y = –5
= 7 solution:
\ d(Q, B) = 7 units The co-ordinate of point A is –4.
The co-ordinate of point B is –5.
(2) if the co-ordinates of A is x and that of B is y,
–4>–5
find d(A, B).
d(A, B)
(i) x = 1, y = 7
= Greater co-ordinate – smaller co-ordinate
solution:
= – 4 – (–5)
The co-ordinate of point A is 1.
= –4+5
The co-ordinate of point B is 7.
= 1
7>1
\ d(A, B) = 1 unit
d(A, B)
= Greater co-ordinate – smaller co-ordinate (v) x = –3, y = –6
= 7–1 solution:
= 6 The co-ordinate of point A is –3.
\ d(A, B) = 6 units The co-ordinate of point B is –6.
–3 > –6
(ii) x = 6, y = –2
d(A, B)
solution:
= Greater co-ordinate – smaller co-ordinate
The co-ordinate of point A is 6.
= –3 – (–6)
The co-ordinate of point B is –2.
= –3 + 6
6 > –2
= 3
d(A, B)
\ d(A, B) = 3 units
= Greater co-ordinate – smaller co-ordinate
= 6 – (– 2) (vi) x = 4, y = – 8
= 6+2 solution:
= 8 The co-ordinate of point A is 4.
\ d(A, B) = 8 units The co-ordinate of point B is – 8.
4>–8
8 Master Key Mathematics - II (Geometry) (Std. IX)

d(A, B) \ Points L, M and N are noncollinear points.


= Greater co-ordinate – smaller co-ordinate \ Relation of betweenness does not exist.
= 4 – (–8) (v) d(X, y) = 15, d(y, Z) = 7, d(X, Z) = 8.
= 4+8
solution:
= 12
\ d(X, Y) = 15 ..... (i)
\ d(A, B) = 12 units
d(Y, Z) + d(X, Z) = 7 + 8

(3) From the information given below, find which \ d(Y, Z) + d(X, Z) = 15 ..... (ii)
of the point is between the other two. if the \ d(X, Y) = d(Y, Z) + d(X, Z)
points are not collinear, state so. [From (i) and (ii)]

(i) d(P, r) = 7, d(P, Q) = 10, d(Q, r) = 3. \ Points X, Y and Z are collinear points.

solution: \ Relation of betweenness exists.

d(P, Q) = 10 ..... (i) X - Z - Y.


d(P, R) + d(Q, R) = 7 + 3 (vi) d(D, E) = 5, d(E, F) = 8, d(D, F) = 6.
\ d(P, R) + d(Q, R) = 10 ..... (ii) solution:
\ d(P, Q) = d(P, R) + d(Q, R) [From (i) and (ii)] \ d(E, F) = 8 ..... (i)
\ Points P, Q and R are collinear points. d(D, E) + d(D, F) = 5 + 6
\ Relation of betweenness exists.
\ d(D, E) + d(D, F) = 11 ..... (ii)
P - R - Q.
\ d(E, F) ≠ d(D, E) + d(D, F) [From (i) and (ii)]
(ii) d(r, s) = 8, d(s, t) = 6, d(r, t) = 4. \ Points D, E and F are noncollinear points.
solution: \ Relation of betweenness does not exist.
\ d(R, S) = 8 ..... (i)
(4) on a number line, points A, B and C are such
d(S, T) + d(R, T) = 6 + 4 that d(A, C) = 10, d(C, B) = 8 Find d(A, B)
\ d(S, T) + d(R, T) = 10 ..... (ii) considering all possibilities.
\ d(R, S) ≠ d(S, T) + d(R, T) [From (i) and (ii)] solution:
\ Points R, S and T are non-collinear points. Case (1) A - B - C
\ Relation of betweenness does not exist. \ d(A, C) = d(A, B) + d(B, C)
(iii) d(A, B) = 16, d(C, A) = 9, d(B, C) = 7. \ 10 = d(A, B) + 8.
solution: \ 10 – 8 = d(A, B)
\ d(A, B) = 16 ..... (i) \ d(A, B) = 2 units.
d(C, A) + d(B, C) = 9 + 7 Case (2) A - C - B
\ d(C, A) + d(B, C) = 16 ..... (ii) \ d(A, B) = d(A, C) + d(C, B)
\ d(A, B) = d(C, A) + d(B, C) [From (i) and (ii)] \ d(A, B) = 10 + 8.
\ Points A, B and C are collinear points. \ d(A, B) = 18 units.
\ Relation of betweenness exists. Case (3) B - A - C
A - C - B. \ d(A, B) + d(A, C) = d(C, B)
(iv) d(L, M) = 11, d(M, N) = 12, d(N, L) = 8. \ d(A, B) + 10 = 8
solution: \ d(A, B) = 8 – 10
\ d(M, N) = 12 ..... (i) \ d(A, B) = – 2
d(L, M) + d(N, L) = 11 + 8 But distance between two points cannot be
negative.
\ d(L, M) + d(N, L) = 19 ..... (ii)
\ B - A - C is not possible.
\ d(M, N) ≠ d(L, M) + d(N, L)
[From (i) and (ii)] \ d(A, B) = 2 units or d(A, B) = 18 units
Basic Concepts of Geometry 9

(5) Points X, y, Z are collinear such that \ l(RT) = 2.5 + 3.7


d(X, y) = 17, d(Y, Z) = 8, find d(X, Z).
\ l(rt) = 6.2 units
solution:
Case (1) Consider X - Y - Z (iii) if X - y - Z and l(XZ) = 3 7 , l(Xy) = 7 , then
l(yZ) = ?
\ d(X, Z) = d(X, Y) + d(Y, Z)
solution:
\ d(X, Z) = 17 + 8.
\ d(X, Z) = 25 units. X Y Z

Case (2) X - Z - Y X-Y-Z (Given)


\ d(X, Y) = d(X, Z) + d(Z, Y) l(XZ) = l(XY) + l(YZ)
\ 17 = d(X, Z) + 8. \ 3 7 = 7 + l(YZ)
\ 17– 8 = d(X, Z) \ 3 7 – 7 = l(YZ)
\ d(X, Z) = 9 units. \ \ l(yZ) = 2 7 units
Case (3) Consider Y - X - Z
(7) Which figure is formed by three non-collinear
\ d(X, Y) + d(X, Z) = d(Y, Z)
points?
\ 17 + d(X, Z) = 8.
solution:
\ d(X, Z) = –17 + 8
\ d(X, Z) = –9 units A B
But, distance between two points cannot be
negative.
Y - X - Z is not possible. C

\ d(X, Z) = 25 units or d(X, Z) = 9 units \ By joining three non-collinear points, a


triangle is formed.
(6) Sketch proper figure and write the answers of
the following questions.
ProBLEMs For PrACtiCE
(i) if A - B - C and l(AC) = 11, l(BC) = 6.5 then
l(AB) = ? (1) observe the number line and answer the
solution: following questions:
E D C B A O P Q R S T
A B C
–5 –4 –3 –2 –1 0 1 2 3 4 5
A-B-C (Given)
l(AC) = l(AB) + l(BC) Find: (i) d(Q, T) (ii) d(E, S)

\ 11 = l(AB) + 6.5 (iii) d(O, Q) (iv) d(O, D)

\ 11 – 6.5 = l(AB) (2) Draw the figures according to the given


information and answer the question:
\ l(AB) = 4.5 units
When A - B - C, l(AC) = 12, l(BC) = 7.5, then
(ii) if r - s - t and l(st) = 3.7, l(rs) = 2.5 then l(A B) = ?
l(rt) = ?
(3) in each of the following decide whether the
solution: relation of betweenness exists or not among
the points A, B and D. Name the point which
R S T
lies between the other two.
R-S-T (Given) (i) d(A, B) = 5, d(B, D) = 8, d(A, D) = 11
l(RT) = l(RS) + l(ST) (ii) d(A, B) = 5, d(B, D) = 15, d(A, D) = 17
10 Master Key Mathematics - II (Geometry) (Std. IX)

(4) if r - s - t, l(st) = 3.75, l(rs) = 2.15, then


goes in a particular direction to infinity is called
l(rt) = ?
Ray.
(5) if X - y - Z, l(XZ) = 5 2 , l(Xy) = 2 2 , then Ray AB.
l(yZ) = ? A B

 Opposite Rays:
ANsWErs
Two rays having a common origin and lying
(1) (i) 3 units (ii) 9 units on the same line are said to be opposite rays.
(iii) 2 units (iv) 4 units
A O B
(2) l(AB) = 4.5 units.
(3) (i) Relation of betweenness does not exist. Here, Ray OA and Ray OB have a common
origin O and they lie on same line.
(ii) Relation of betweenness exists. Point B is
between points A and D. \ OA and OB are called opposite rays.

(4) l(RT) = 5.9 units  Length of a line segment:


(5) l(YZ)= 3 2 units. The distance between the end points of a line
segment is called the length of the segment.
Length of segment AB is denoted by l(AB) or
Points to Remember: AB.
 Congruent segments:
 Line segment: Two segments are said to be congruent if they
The set consisting of points A and B and all the are of the same length i.e. l(AB) = l(CD) then
points between A and B is called the segment seg AB @ seg CD.
AB and written as ‘seg AB’.
 Properties of Congruent segments:
A B (1) Reflexivity: Every segment is congruent to
itself. seg PQ @ seg PQ.
Note:
(2) Symmetry: If the first segment is congruent to
(1) The points A and B are called end points of the
second, then the second is congruent to first.
seg AB.
If seg PQ @ seg AB then seg AB @ seg PQ.
(2) A line segment is a subset of a line.
(3) Transitivity: If first segment is congruent
 Midpoint of segment:
to second segment and second segment is
The point M is said to be the midpoint of congruent to third segment then first segment
seg AB, if (1) A - M - B (2) AM = MB is congruent to third segment.

A M B If seg PQ @ seg AB and seg AB @ seg XY then


seg PQ @ seg XY.
 Comparison of segment:
If AB < CD, then we say that seg AB is smaller
than seg CD and denoted by seg AB < seg PrACtiCE sEt - 1.2 (Textbook Page No. 7)
CD.
(1) the following table shows points on a number
A B line and their co-ordinates. Decide whether
the pair of segments given below the table are
C D congruent or not.

 Ray: Point A B C D E
A portion of line which starts at a point and Co-ordinate –3 5 2 –7 9
Basic Concepts of Geometry 11

(i) seg DE and seg AB \ l(AD) = 4 units ....(ii)


solution: \ l(BC) ¹ l(AD) [From (i) and (ii)]
The co-ordinate of point D is – 7 and \ seg BC is not congruent to seg AD.
the co-ordinate of point E is 9
(iii) seg BE and seg AD
9>–7
d(D, E) solution:

= Greater co-ordinate – Smaller co-ordinate The co-ordinate of point B is 5.

= 9 – (– 7) The co-ordinate of point E is 9.

= 9+7 9>5

\ d(D, E) = 16 d(B, E)

\ l(DE) = 16 units ....(i) = Greater co-ordinate – Smaller co-ordinate

The co-ordinate of point A is – 3. = 9–5

The co-ordinate of point B is 5. = 4

5>–3 \ d(B, E) = 4 units

d(A, B) \ l(BE) = 4 units ....(i)

= Greater co-ordinate – Smaller co-ordinate The co-ordinate of point A is – 3.

= 5 – (– 3) The co-ordinate of point D is – 7.

= 5+3 –3>–7

\ d(A, B) = 8 units d(A, D)

\ l(AB) = 8 units ....(ii) = Greater co-ordinate – Smaller co-ordinate

\ l(DE) ¹ l(AB) [From (i) and (ii)] = – 3 – (– 7)


= –3+7
\ seg DE is not congruent to seg AB.
= 4
(ii) seg BC and seg AD \ d(A, D) = 4 units
solution: \ l(AD) = 4 units ....(ii)
The co-ordinate of point B is 5. \ l(BE) = l(AD) [From (i) and (ii)]
The co-ordinate of point C is 2. \ seg BE @ seg AD
5>2
(2) Point M is the midpoint of seg AB. if AB = 8
d(B, C) then find the length of AM.
= Greater co-ordinate - Smaller co-ordinate
solution:
= 5–2
M is midpoint of seg AB .... (Given)
= 3 1
\ l(AM) = l(AB)
\ d(B, C) = 3 units 2
\ l(BC) = 3 units ....(i) 1
\ l(AM) = ×8
The co-ordinate of point A is – 3. 2
The co-ordinate of point D is – 7. \ l(AM) = 4 units
–3>–7
d(A, D) (3) Point P is the midpoint of seg CD. if CP = 2.5,
= Greater co-ordinate – Smaller co-ordinate find l(CD).

= – 3 – (– 7) solution:
= –3+7 Point P is midpoint of seg CD. .... (Given)
1
= 4 \ l(CP) = l(CD)
2
\ d(A, D) = 4 units
12 Master Key Mathematics - II (Geometry) (Std. IX)
1
\ 2.5 = l(CD) (ii) Write a pair of points equidistant from point
2 Q.
\ 2.5 × 2 = l(CD)
Ans. Point R and point P.
\ l(CD) = 5 units (iii) Find d(U, V), d(P, C), d(V, B), d(U, L).
solution:
(4) if AB = 5 cm, BP = 2 cm and AP = 3.4 cm,
d(U, V)
compare the segments.
The co-ordinate of point U is – 5.
solution:
The co-ordinate of point V is 5.
AB = 5 cm, BP = 2 cm, AP = 3.4 cm
5 > –5
5 > 3.4 > 2
d(U, V)
\ l(AB) > l(AP) > l(BP)
= Greater co-ordinate – Smaller co-ordinate
(5) Write the answers to the following questions = 5 – (–5)
with reference to figure.
= 5+5
T S R P Q \ d(U, V) = 10 Units
(i) Write the name of the opposite ray of ray RP.
d(P, C)
Ans. Ray RS is opposite of ray RP.
The co-ordinate of point P is – 2.
(ii) Write the intersection set of ray PQ and
ray RP. The co-ordinate of point C is 4.

Ans. Intersection set of ray PQ and ray RP is ray PQ. 4>–2

(iii) Write the union set of ray PQ and ray QR. d(P, C)

Ans. The union of ray PQ and ray QR is line QR. = Greater co-ordinate – Smaller co-ordinate

(iv) State the rays of which seg QR is a subset. = 4 – (– 2)

Ans. Seg QR is a subset of ray QR, ray RQ, ray QS = 4+2


and ray QT. \ d(P, C) = 6 Units
(v) Write the pair of opposite rays with common
end point r. d(V, B)

Ans. Ray RP and ray RS. The co-ordinate of point V is 5.

(vi) Write any two rays with common end point The co-ordinate of point B is 2.
s. 5>2
Ans. Ray ST and ray SR. d(V, B)
(vii) Write the intersection set of ray SP and ray = Greater co-ordinate – Smaller co-ordinate
st. = 5–2
Ans. The intersection of ray SP and ray ST is point \ d(V, B) = 3 Units
S.
d(U, L)
(6) Answer the questions with the help of The co-ordinate of point U is – 5.
figure.
The co-ordinate of point L is – 3.
R U Q L P A B C V D
–3>–5
–6 –4 –2 0 2 4 6
(i) state the points which are equidistant from d(U, L)
point B. = Greater co-ordinate – Smaller co-ordinate
Ans. (a) Point A and Point C. = – 3 – (– 5) = – 3 + 5
(b) Point P and Point D. \ d(U, L) = 2 Units
Basic Concepts of Geometry 13

ProBLEMs For PrACtiCE The part of the statement following ‘If’ is called
(1) the co-ordinates of the points on a line are as the antecedent and the part following ‘then’ is
follows: called the consequent.

Points P Q R S T For example, consider the statement: The


diagonals of a rhombus are perpendicular
Co-ordinates –3 4 2 –5 9
bisectors of each other.
Check whether given pair of segments are The statement can be written in the conditional
congruent or not. form as, ‘If the given quadrilateral is a rhombus
(i) seg PQ and seg QS. then its diagonals are perpendicular bisectors
(ii) seg RS and seg PQ. of each other.’
(iii) seg PS and seg RT. If the antecedent and consequent in a given
(2) If P is midpoint of seg AB and AB = 7 cm, find conditional statement are interchanged, the
AP. resulting statement is called the converse of
the given statement.
(3) if Q is midpoint of seg CD and d(C, Q) = 6, find
length of seg CD. If a conditional statement is true, its converse
is not necessarily true. Study the following
(4) if AB = 7 cm, BP = 4 cm, AP = 5.4 cm, compare
the segments. examples.
Conditional statement: If a quadrilateral is a
(5) In the below figure,
rhombus then its diagonals are perpendicular
l
D C A B E bisectors of each other.

A, B, C, D and E are the points of line l. Converse: If the diagonals of a quadrilateral


are perpendicular bisectors of each other then
(i) Name the opposite rays with point A as it is a rhombus.
origin.
In the above example, the statement and its
(ii) Name the ray opposite to ray BE.
converse are true.
(iii) Find the intersection of ray CE and ray BC.
Now consider the following example.
Conditional statement: If a number is a prime
ANsWErs number then it is even or odd.
(1) (i) not congruent (ii) congruent Converse: If a number is even or odd then it is
(iii) not congruent a prime number.
In this example, the statement is true, but its
(2) AP = 3.5 cm
converse is false.
(3) l(CD) = 12 cm.
 Axioms/Postulates:
(4) AB > AP > BP
These are simple statements which we accept as
(5) (i) ray AD and ray AE
true and need not be proved. Such statements
(ii) ray BD (iii) seg CB are called Axioms or Postulates.
Some Euclid’s Postulates are given below:
l There are infinite lines passing through a
Points to Remember: point:
 Conditional statements and converse: In the following figure, lines l, m, n and o passes
The statements which can be written in the ‘If- through point P. Similarly, we can draw infinite
then’ form are called conditional statements. lines passing through P.
14 Master Key Mathematics - II (Geometry) (Std. IX)

n
In the above figure,
m
a and b form a pair of interior angles.
a is an acute angle, i.e. Ða < 90º
l
P b is an acute angle, i.e. Ðb < 90º
\ Ða + Ðb < 90º + 90º
o
\ Ða + Ðb < 180º

l There is one and only one line passing \ line l intersects line m at point T.
through two distinct points.
 theorem:
Line l passes through the points A and B.
l
l A property is supposed to be true if it can be
B proved logically. It is then called Theorem.
A
Proof:
l A circle of given radius can be drawn taking
The logical argument made to prove a theorem
any point as its centre.
is called its proof.
A circle of radius 3 cm with centre P is drawn.
 theorem:
l If two lines intersect each other then the vertically
3 cm
P opposite angles formed are congruent.

A D
l All right angles are congruent to each other.
In the following figures, ÐB = ÐQ = ÐM = 90º P

Q
C B
A

Given: Line AB and line CD intersect each


R P
other at point P.
B C to prove: (1) ÐAPC @ ÐBPD
(2) ÐAPD @ ÐBPC
L
Proof:
ÐAPC + ÐAPD = 180º ...(i)
(Linear pair of angles)
N M
ÐBPD + ÐAPD = 180º ...(ii)
l if two interior angles formed on one side of
(Linear pair of angles)
a transversal of two lines add upto less than
two right angles then the lines produced in \ ÐAPC + ÐAPD = ÐBPD + ÐAPD
that direction intersect each other. [From (i) and (ii)]
n
\ ÐAPC = ÐBPD
l
i.e. ÐAPC @ ÐBPD
a Similarly, we can prove,
T b ÐAPD @ÐBPC.

m
Basic Concepts of Geometry 15

PrACtiCE sEt - 1.3 (Textbook Page No. 11) ProBLEMs For PrACtiCE
Q.1. Write the following statement in ‘if-then’ (1) Write the following statement in ‘if-then’
form. form:
(i) the opposite angles of a parallelogram are (i) All sides of rhombus are congruent.
congruent. (ii) In an equilateral triangle all sides are
solution: congruent.
(2) Write the converse of following theorems:
If a quadrilateral is a parallelogram then its
opposite angles are congruent. (i) Opposite angles of a cylic quadrilateral are
supplementary.
(ii) the Diagonals of a rectangle are congruent. (ii) In an Isosceles triangle the angles opposite to
solution: equal sides are congruent.
If a quadrilateral is a rectangle then its diagonals
are congruent. ANsWErs
(iii) In an isosceles triangle, the segment joining (1) (i) If the quadrilateral is a rhombus, then all
the vertex and the mid point of the base is sides are congruent.
perpendicular to the base. (ii) If the triangle is equilateral, then all three
sides are congruent.
solution:
(2) (i) If opposite angles of a quadrilateral are
If a triangle is an isosceles then segment joining
supplementary, then the quadrilateral is
vertex and midpoint of the base is perpendicular
cyclic.
to the base.
(ii) In a triangle, the sides opposite to congruent
Q.2. Write converse of the following statements: angles are congruent.
(i) The alternate angles formed by two parallel
lines and their transversal are congruent. ProBLEM sEt - 1 (Textbook Page No. 11)
solution: (1) select the correct alternative from the answers
Converse of above statement: of the questions given below:
If alternate angles formed by two lines and its (i) How many mid points a segment have?
a transversal are congruent then the lines are (A) Only one (B) Two (C) Three (D) Many
parallel. Ans. (A)
(ii) If a pair of the interior angles made by a (ii) How many points are there in the intersection
transversal of two lines are supplementary then of two distinct lines?
the lines are parallel. (A) Infinite (B) Two (C) One (D) Not a single
solution: Ans. (C)
Converse of above statement: (iii) How many lines are determined by three distinct
If two parallel lines are intersected by a points?
transversal then interior angles so formed are (A) Two (B) Three (C) One or three (D) Six
supplementary. Ans. (C)
(iii) The diagonals of a rectangle are congruent. (iv) Find d(A, B), if co-ordinates of A and B are – 2
and 5 respectively.
solution:
(A) –2 (B) 5 (C) 7 (D) 3
Converse of above statement does not exit.
Ans. (C)
(v) If P - Q - R and d(P, Q) = 2, d(P, R) = 10, then find
d(Q, R).
(A) 12 (B) 8 (C) 96 (D) 20
16 Master Key Mathematics - II (Geometry) (Std. IX)

Ans. (B) d(P, Q) – d(P, R) = d(Q, R) is a false statement.


(2) on a number line, co-ordinates of P, Q, r are 3, (3) Co-ordinates of some pairs of points are given
– 5 and 6 respectively. State with reason below. Hence find the distance between each
whether the following statements are true or pair.
false.
(i) 3, 6
(i) d(P, Q) + d(Q, R) = d(P, R)
solution:
(ii) d(P, R) + d(R, Q) = d(P, Q)
Let the co-ordinate of point A be 3
(iii) d(R, P) + d(P, Q) = d(R, Q) and the co-ordinate of point B be 6.
(iv) d(P, Q) – d(P, R) = d(Q, R) 6>3
solution: d(A, B) =
The co-ordinate of P is 3. = Greater co-ordinate – Smaller co-ordinate
The co-ordinate of Q is – 5. = 6–3
The co-ordinate of R is 6. = 3
d(P, Q) = 3 – (–5) = 3 + 5 = 8 units \ d(A, B) = 3

d(Q, R) = 6 – (–5) = 6 + 5 = 11 units \ the distance between the given


d(P, R) = 6 – 3 = 3 units pair of points is 3 units.

(i) d(P, Q) + d(Q, R) = d(P, R) (ii) – 9, – 1


d(P, Q) + d(Q, R) = 8 + 11 = 19 units ....(i) solution:
d(P, R) = 3 units ....(ii) Let the co-ordinate of point A be – 9
\ d(P, Q) + d(Q, R) ¹ d(P, R) and the co-ordinate of point B be – 1.
[From (i) and (ii)] –1>–9
\ d (P, Q) + d (Q, R) = d(P, R) is a false d(A, B)
statement. = Greater co-ordinate – Smaller co-ordinate
(ii) d(P, R) + d(R, Q) = d(P, Q) = – 1 – (– 9)
d(P, R) + d(R, Q) = 3 + 11 = 14 units ....(i) = –1+9
d(P, Q) = 8 units ....(ii) = 8
\ d(A, B) = 8 units
\ d(P, R) + d(R, Q) ¹ d(P, Q)
[From (i) and (ii)] \ the distance between the given
d(P, R) + d(R, Q) = d(P, Q) is false statement. pair of points is 8 units.
(iii) d(R, P) + d(P, Q) = d(R, Q) (iii) – 4, 5
d(R, P) + d(P, Q) = 3 + 8 = 11 units ....(i) solution:
d(R, Q) = 11 units ....(ii) Let the co-ordinate of point A be – 4
\ d(R, P) + d(P, Q) = d(R, Q) and the co-ordinate of point B be 5.
[From (i) and (ii)] 5>–4
d(R, P) + d(P, Q) = d(R, Q) is a true statement. d(A, B)
(iv) d(P, Q) – d(P, R) = d(Q, R) = Greater co-ordinate – Smaller co-ordinate
= 5 – (– 4)
d(P, Q) – d(P, R) = 8 – 3 = 5 units ....(i)
= 5+4
d(Q, R) = 11 units ....(ii)
= 9
\ d(P, Q) – d(P, R) ¹ d(Q, R)
\ d(A, B) = 9 units
[From (i) and (ii)]
\ the distance between the given
pair of points is 9 units.
Basic Concepts of Geometry 17

(iv) 0, – 2 (vii) 80, – 85


solution: solution:
Let the co-ordinate of point A be 0 Let the co-ordinate of point A be 80
and the co-ordinate of point B be – 2. and the co-ordinate of point B be – 85

0>–2 d(A, B)
= Greater co-ordinate – Smaller co-ordinate
d(A, B)
= 80 – (– 85)
= Greater co-ordinate – Smaller co-ordinate
= 80 + 85
= 0 – (– 2) = 165
\ 0+2 \ d(A, B) = 165 units
\ d(A, B) = 2 \ the distance between the given
pair of points is 165 units.
\ the distance between the given
pair of points is 2 units. (4) Co-ordinate of point P, on a number line is –7.
Find the co-ordinates of points on the number
(v) x + 3, x – 3
line which are at a distance of 8 units from
solution: point P.
Let the co-ordinate of point A be x + 3 solution:
and the co-ordinate of point B be x – 3 Let co-ordinate of Q be x be a point on positive
(x + 3) > (x – 3) side of point P.
d(A, B) Co-ordinate of P is – 7.
= Greater co-ordinate – Smaller co-ordinate x>–7
= (x + 3) – (x – 3) d(P, Q) = x – (– 7)
= x+3–x+3 \ 8=x+7
\ x=8– 7
= 6
\ x=1
\ d(A, B) = 6 units
\ Co-ordinate of point Q is 1.
\ the distance between the given
pair of points is 6 units. Let co-ordinate of ponit R be y be a point on
negative side of point P.
(vi) – 25, – 47 Co-ordinate of point P is – 7
solution: –7>y
Let the co-ordinate of A point be – 25 \ d(P, R) = – 7 – y
and the co-ordinate of B point be – 47 \ 8=–7–y
d(A, B) \ 8+7=–y
= Greater co-ordinate – Smaller co-ordinate \ – y = 15
\ y = – 15
= – 25 – (– 47)
= – 25 + 47 \ Co-ordinate of point r is – 15

= 22 (5) Answer the following questions.


\ d(A, B) = 22 units (i) If A - B - C and d(A, C) = 17, d(B, C) = 6.5 then
d(A, B) = ?
\ the distance between the given
solution:
pair of points is 22 units.
A-B-C (Given)
18 Master Key Mathematics - II (Geometry) (Std. IX)

\ d(A, C) = d(A, B) + d(B, C) (iii)A triangle is a figure formed by three


\ 17 = d(A, B) + 6.5 segments.
\ 17 – 6.5 = d(A, B) Ans. If a polygon is three-sided closed figure, then it
is a triangle.
\ d(A, B) = 10.5 units
(iv) A number having only two divisors is called a
(ii) If P - Q - R and d(P, Q) = 3.4, d(Q, R) = 5.7, then prime number.
find d(P, R) = ?
Ans. If a number is a prime number then it has only
solution: two divisors.
P-Q-R (Given)
(8) Write the converse of each of the following
\ d(P, R) = d(P, Q) + d(Q, R)
statements.
\ = 3.4 + 5.7
(i) If the sum of measures of angles in a figure is
\ = 9.1 units 180º, then the figure is a triangle.
\ d(P, r) = 9.1 units Ans. Converse: If a figure is a triangle then sum of
(6) Co-ordinate of point A on a number line is all angles of this figure is 180º.
1. What are the co-ordinates of points on the (ii) If the sum of measures of two angles is 90º then
number line which are at a distance of 7 units they are complement of each other.
from A? Ans. Converse: If two angles are complementary then
solution: their sum is 90º.
Co-ordinate of point A is 1. (iii) If the corresponding angles formed by a
Let co-ordinate of point B be x be a point on transversal of two lines are congruent then the
positive side of point A. two lines are parallel.
x>1 Ans. Converse: If two parallel lines are intersected
\ d(A, B) = x – 1 by a transversal then the pair of corresponding
\ 7=x–1 angles is congruent.

\ 7+1=x (iv) If the sum of the digits of a number is divisible


\ x=8 by 3 then the number is divisible by 3.
Ans. Converse: If a number is divisible by 3 then sum
\ Co-ordinate of point B is 8.
of digits of this number is divisible by 3.
Let co-ordinate of point C be y be a point on
negative side of point A. (9) Write the antecedent (given part) and the
1>y consequent (part to be proved) in the following
statements.
\ d(A, C) = 1 – y
(i) If all sides of a triangle are congruent then its all
\ 7=1–y
angles are congruent.
\ y=1–7
Ans. Given: In DABC, side AB @ side BC @ side AC.
\ y=–6
to prove: ∠A @ ∠B @ ∠C.
\ Co-ordinate of point C is – 6.
(ii) The diagonals of a parallelogram bisect each
(7) Write the following statements in conditional other.
form: Ans. Given: (1) oPQRS is a parallelogram.
(i) Every rhombus is a square.
(2) Diagonals PR and QS intersect at point M.
Ans. If a quadrilateral is a square then it is a
rhombus. to prove: (1) PM = RM (2) QM = SM.
(ii) Angles in a linear pair are supplementary. (10) Draw a labelled figure showing information
Ans. If adjacent angles are supplementary, then they in each of the following statements and write
form a linear pair. the antecedent and the consequent.
Basic Concepts of Geometry 19

(i) Two equilateral triangles are similar. (3) With the help of the given figure, which of the
Ans. A P following statement is true?

P Q R
Q R (A) d(P, R) + d(R, Q) = d(P, Q)
B C
(B) d(P, R) + d(P, Q) = d(R, Q)
Given: (1) In DABC, side AB @ side BC @ side AC.
(C) d(Q, R) – d(P, Q) = d(P, R)
(2) In DPQR, side PQ @ side QR @ side PR.
to prove: DABC ~ DPQR (D) d(P, R) – d(R, Q) = d(P, Q)

(ii) If angles in a linear pair are congruent then each (4) If d(P, R) = 7, d(Q, R) = 3 and d(P, Q) = 4, what
of them is a right angle. can we say about points P, Q and R?
Ans. (A) P – R – Q (B) Q – P – R
A
(C) P – Q – R (D) R – P – Q
(5) A statement which is universally true and need
D B C
not be proved is called ................. .
Given: (1) ∠ABC and ∠ABD form a linear pair. (A) Axiom (B) Postulate
(2) ∠ABC @ ∠ABD. (C) both a and b (D) Theorem
to prove: ∠ABC = ∠ABD = 90º (6) From a single point, we can draw .................
(iii) If the altitudes drawn on two sides of a lines.
triangle are congruent then those two sides are (A) One (B) Infinite
congruent. (C) Two (D) Five
Ans. A (7) Two different points determine ................. line.
(A) Six (B) Infinite
N M
(C) Two (D) One and only one
B C (8) The co-ordinate of point E is ................. .
Given: In DABC F C G B H A K D I E J
l
(1) seg BM ^ side AC, A - M - C. –8 –7 –6 –5 –4 –3 –2 –1 0 1 2
(2) seg CN ^ side AB, A - N - B.
(A) – 4 (B) 1
(3) seg BM @ seg CN.
(C) 2 (D) – 5
to prove: Side AB@ side AC.
(9) The co-ordinate of point J is ................. .
MCQ’s F C G B H A K D I E J
l
–8 –7 –6 –5 –4 –3 –2 –1 0 1 2
(1) The number associated with the point is called
................. of that point. (A) – 7 (B) 2
(A) Origin (B) Midpoint (C) 0 (D) – 5
(C) Co-ordinate (D) Graph (10) Which points are at a distance of 3 units from
(2) With the help of the given number line, what is point A?
d(P, T)? F C G B H A K D I E J
l
D C P Q R S T M N A B –8 –7 –6 –5 –4 –3 –2 –1 0 1 2
–6 –5 –4 –3 –2 –1 0 1 2 3 4 5 6 (A) Points C, E (B) Points G, I
(A) 3 units (B) 4 units (C) Points D, B (D) Points H, E
(C) – 3 units (D) – 4 units
20 Master Key Mathematics - II (Geometry) (Std. IX)

(11) On the given number line, d(K, J) = ................. . (18) In the adjoining figure, angles are in linear pair.
F C G B H A K D I E J What is the value of x?
l
–8 –7 –6 –5 –4 –3 –2 –1 0 1 2
(4x + 60)º (x + 20)º
(A) 3 units (B) 2 units
(C) 4 units (D) 5 units
(12) If P – Q – R, l(PR) = 7 units, l(PQ) = 4 units, (A) 40 (B) 80
l(QR) = ................. .
(C) 20 (D) 30
(A) 11 units (B) 3 units
(19) Which of the following are pair of linear pair
(C) 6 units (D) 5 units angles.
(13) If AB = 10 units, AC = 7 units, BC = 3 units then A
which of the following is correct? S
(A) A-B-C D

(B) C-B-A
P Q R B C
(C) A-C-B
(D) Point A, B, C are non collinear. (A) (B)
(14) If the co-ordinates of points P and Q are 3 and – 5 P
L M T
respectively then d(P, Q) = ................. .
(A) 9 units (B) 2 units
(C) 8 units (D) 7 units N Q
S
(15) If d(P, Q) = 10 units, d(P, R) = 18 units and P-Q-R R
then d(Q, R) = ................. . (C) (D)
(A) 28 units (B) 10 units
(A) A and B (B) B and D
(C) 18 units (D) 8 units
(C) C and D (D) A and C
(16) If the co-ordinates of points P and Q are 2
(20) AB = CD, CD = EF then ................. .
and – 2 respectively then d(P, Q) = .............. .
(A) AB = EF, by property of symmetry
(A) 2 units (B) 3 2 units
(B) AB = EF, by property of reflexivity
(C) 2 2 units (D) 4 2 units
(C) AB = EF, by property of transitivity
(17) What is mÐPQS?
(D) AB ¹ EF
S
P ANsWErs
?
100º
Q (1) (C) (2) (B) (3) (D) (4) (C)
(5) (C) (6) (B) (7) (D) (8) (B)
R (9) (B) (10) (B) (11) (C) (12) (B)
(A) 70º (B) 90º (13) (C) (14) (C) (15) (D) (16) (C)
(C) 80º (D) 60º (17) (C) (18) (C) (19) (D) (20) (C)
Basic Concepts of Geometry 21

ASSIGNMENT - 1
Time : 1 Hr. Marks : 20

Q.1. Observe the number line and answer the following: (4)
F P N M L O A B C D E

–5 –4 –3 –2 –1 0 1 2 3 4 5

Find (1) d(M, C) (2) d(L, P) (3) d(O, N) (4) d(A, E)

Q.2. Solve the following: (10)


(1) The points A, B, C are on a line such that d(A, C) = 10, d(C, B) = 8 then find d(A, B).
(2) Draw the figure, according to the given information and answer the question when A-B-C,
l(AC) = 11, l(BC) = 6.5 then l(AB) = ?
(3) P is midpoint of seg AB. If AB is 15, then find length of AP.
(4) Write the given statement in ‘if-then’ form for prime numbers are divisible by 1 and itself.
(5) Diagonals of a parallelogram bisect each other. Write the proof for the given statement.

Q.3. Solve the following: (6)


(1) Prove: If two lines intersect each other, then vertically opposite angles are congruent.
(2) Co-ordinate of a point P on a number line is – 7. Find the co-ordinates of points which are at a
distance of 8 units from point P.

vvv
22 Master Key Mathematics - II (Geometry) (Std. IX)

2 Parallel Lines

Theorem - 1 : Interior angles theorem


Points to Remember:
 Theorem : If two parallel lines are intersected by
 Parallel Lines a transversal, the interior angles on either side
Parallel Lines : The lines which are l
of the transversal are supplementary.
Given : line l line m and n
coplanar and do not intersect each m
other are called parallel lines. line n is their d a m
In the above figure line l is parallel to line m transversal. c b
l
Hence as shown
Symbolically, it is written as 'line l line m'
in the figure ∠a,
Transversal : A tranversal is a line that intersects ∠b are interior angles formed on one
two or more lines in distinct points. side and ∠c, ∠d are interior angles
n
In the adjoining figure, formed on other side of the transversal.
line n is the transversal To prove : ∠a + ∠b = 180°
a d
b c for lines l and m. The
l ∠d + ∠c = 180°
e h two lines and the
Proof : Three possibilities arise regarding the
f g m transversal determine
eight angles at the point sum of measures of ∠a and ∠b.
of intersection. (i) ∠a + ∠b < 180°
Pairs of Corresponding angles : (ii) ∠a + ∠b > 180°
(i) ∠b and ∠f (ii) ∠c and ∠g (iii) ∠a + ∠b = 180°
(iii) ∠a and ∠e (iv) ∠d and ∠h Let us assume that the possibility
Pairs of Alternate Interior angles : (i) ∠a + ∠b < 180° is true.
(i) ∠b and ∠h (ii) ∠c and ∠e Then according to Euclid's postulate, if
Pairs of Alternate Exterior angles : the line l and line m are produced will
intersect each other or the side of the
(i) ∠a and ∠g (ii) ∠d and ∠f
transversal where ∠a and ∠b are formed.
Pairs of Interior angles :
But line l and line m are parallel lines
(i) ∠b and ∠e (ii) ∠c and ∠h ...(Given)
 Important Properties ∴ ∠a + ∠b < 180° impossible ...(i)
(1) When two lines intersect, the pairs of opposite Now let us suppose that ∠a + ∠b > 180°
angles formed are congruent. is true.
(2) The angles in a linear pair are supplementry ∴ ∠a + ∠b > 180°
(3) When one pair of corresponding angles is But ∠a + ∠d = 180°
congruent, then all the remaining pairs of and ∠c + ∠b = 180°
corresponding angles are congruent. ...(Angles in linear pair)
(4) Whe one pair of alternate angles is congruent, ∴ ∠a + ∠d + ∠b + ∠c = 180° + 180° = 360°
then all the remaining pairs of alternate angles ∴ ∠c + ∠d = 360° – (∠a + ∠b)
are congruent.
If ∠a + ∠b > 180° then
(5) When one pair of interior angles on one side of [360° – (∠a + ∠b)] < 180°
the transversal is supplementary, then the other
∴ ∠c + ∠d < 180°
pair of interior angles is also supplementary.
(22)
Parallel Lines 23

∴ In that case line l and line m produced But, ∠c + ∠b = 180°


will intersect each other on the same side ...(ii) (Interior angles theorem)
of the transversal where ∠c and ∠d are ∴ ∠d + ∠c = ∠c + ∠b ...[From (i) and (ii)]
formed. ∴ ∠d = ∠b
∴ ∠c + ∠d < 180° is impossible
∴ That is ∠a + ∠b > 180° is impossible
MASTER KEY QUESTION SET - 2
...(ii)
∴ the remaining possibility, PRACTICE SET - 2.1 (Textbook Page No. 17)
∴ ∠a + ∠b = 180° is true
(1) In the given figure, D
...[From (i) and (ii)]
line RP line MS and
∴ ∠a + ∠b = 180° R 85° P
line DK is their H
Similarily, ∠c + ∠d = 180° transversal,
Note that, in this proof, because of ∠DHP = 85°. M G S
the contradictions we have denied Find the measure of K
the possibilities ∠a + ∠b > 180° and following angles :
∠a + ∠b < 180° (i) ∠RHD (ii) ∠PHG
(iii) ∠HGS (iv) ∠MGK
Theorem - 2 : Corresponding angles theorem
Solution :
Statement : The corresponding angles formed by a (i) ∠DHP + ∠RHD = 180°
transversal of two parallel lines are of equal ...(Angles in linear pair)
measures. n
a ∴ 85 + ∠RHD = 180
Given : line l line m l
c ∴ ∠RHD = 180 – 85
and line n is a b m ∴ ∠RHD = 95°
transversal.
To prove : ∠a = ∠b (ii) ∠PHG = ∠RHD
...(Vertically opposite angles)
Proof : ∠a + ∠c = 180°
∴ ∠PHG = 95°
...(i) (Angles in linear pair)
But, ∠b + ∠c = 180° (iii) line RP line MS ...(Given)
...(ii) (Interior angles theorem) On transversal DK,
∴ ∠a + ∠c = ∠b + ∠c ...[From (i) and (ii)] ∠DHP = ∠HGS
∴ ∠a = ∠b ...(Corresponding angles theorem)
∴ ∠HGS = 85°
Theorem - 3 : Alternate angles theorem (iv) ∠HGS = ∠MGK
...(Vertically opposite angles)
Statement : The alternate angles formed by a ∴ ∠MGK = 85°
transversal of two parallel lines are of equal
measures. (2) In the given figure, line p line q and line l
n and line m are transversals. Measures of some
Given : line l line m
l angles are shown. Hence find the measures of
and line n is a d c
b
∠a , ∠b, ∠c, ∠d. p
transversal. m
q
a
To prove : ∠d = ∠b 110° b
e
Proof : ∠d + ∠c = 180° l
c 115°
...(i) (Angles in linear pair) d m
24 Master Key Mathematics - II (Geometry) (Std. IX)

Solution : (4) In the given figure, P


∠a + 110 = 180 ...(Angles in linear pair) sides of ∠PQR and
X
∴ ∠a = 180 – 110 ∠XYZ are parallel to
each other. Prove
∴ ∠a = 70°
that ∠PQR ≅ ∠XYZ. Y Z
Name an angle as 'e' as shown in the figure,
Construction : Extend ray Q
line p line q ...(Given) M R
XY to intersect ray QR at
On transversal l, point M, such that Q-M-R
∠e = ∠a ...(Corresponding angles theorem) Proof : line PQ line XY ...(Given)
∴ ∠e = 70 i.e. line PQ line XM ...(X - Y - M)
∴ ∠b = ∠e ...(Vertically opposite angles) On transversal QR,
∴ ∠b = 70° ∠PQR ≅ ∠XMR ...(i)
(Corresponding angles theorem)
line p line q ...(Given)
line YZ line QR ...(Given)
On transversal m,
On transversal XM,
∠c = 115°
∠XYZ ≅ ∠XMR ...(ii)
...(Corresponding angles theorem)
(Corresponding angles theorem)
∠d + 115 = 180 ...(Angles in linear pair)
∠PQR ≅ ∠XYZ ...[From (i) and (ii)]
∴ ∠d = 180 – 115
(5) In the given figure, P
∴ ∠d = 65° line AB line CD A
n p R B
(3) In the given and line PQ is 105°
figure, line l 45° transversal. Measure T
l
line m and d of one of the angles C D
line n line a
m is given. Hence find Q
c b
p. Find ∠a, the measures of the following angles :
∠b, ∠c from (i) ∠ART (ii) ∠CTQ (iii) ∠DTQ (iv) ∠PRB.
the given measure of an angle. Solution :
Solution : (i) ∠BRT + ∠ART = 180°
Name an angle as 'd' as shown in the figure, ...(Angles in linear pair)
∠d = 45° ...(Vertically opposite angles) ∴ 105 + ∠ART = 180
line l line m ...(Given) ∴ ∠ART = 180 – 105
On transversal p, ∴ ∠ART = 75°
∠d + ∠a = 180°...(Interior angles theorem) (ii) line AB line CD ...(Given)
∴ 45 + ∠a = 180 On transversal PQ,
∴ ∠a = 180 – 45 ∠ART = ∠CTQ
∴ ∠a = 135° ...(Corresponding angles theorem)
∠b = ∠a ...(Vertically opposite angles) ∴ ∠CTQ = 75°
∴ ∠b = 135° (iii) line AB line CD ...(Given)
line n line p On transversal PQ,
On transversal m, ∠BRT = ∠DTQ
...(Corresponding angles theorem)
∠c = ∠b ...(Corresponding angles theorem)
∴ ∠DTQ = 105°
∴ ∠c = 135°
(iv) ∠PRB = ∠ART...(Vertically opposite angles)
∴ ∠PRB = 75°
Parallel Lines 25

PROBLEMS FOR PRACTICE


Points to Remember:
(1) If AB CD, then
find ∠PCD and
A 56° B Theorem - 4 : Angle sum property of a triangle
∠CPD from the 100° P Statement : The sum of the measures of all angles
adjoining figure. of a triagle is 180°.
D A E
C D

(2) In the adjoining


A 65° B
figure, measures 35°
B C
of two angles are
y Given : �ABC is any triangle
given. If x z
line ED seg AB E C D To prove : ∠BAC + ∠ABC + ∠ACB = 180°
and E - C - D, then Construction : Through point A, draw a
find the values of x, y and z. line DE side BC, such that D - A - E.
Proof : line DE side BC
(3) In given figure, AB DE, ∠ABC = 75° and
on transversal AB,
∠CDE = 145° then find ∠BCD. D
E ∠DAB = ∠ABC ...(i)
B 145°
A (Alternate angles theorem)
75° on transversal AC,
C ∠EAC = ∠ACB ...(ii)
(4) If AB CD, m∠APQ = 50° and m∠PRD = 127° (Alternate angles theorem)
find x an y. P
∠DAB + ∠EAC = ∠ABC + ∠ACB
A 50° y B
...[Adding equations (i) and (ii)]

127° Adding ∠BAC on both sides,


x
Q C R D ∴ ∠DAB + ∠EAC + ∠BAC = ∠ABC +
E
∠ACB + ∠BAC
(5) If AB CD, EF ⊥ CD and ∠GED = 126°. ∴ ∠DAB + ∠EAB = ∠ABC + ∠ACB +
Find ∠AGE, ∠GEF, ∠FGE ∠BAC ...(Adding addition property)
G F ∴ ∠ABC + ∠ACB + ∠BAC = ∠DAB +
A B ∠EAB
∴ ∠ABC + ∠ACB + ∠BAC = 180°
...(Angles in linear pair)
C E D
Theorem - 5 : Interior angles test
ANSWERS Statement : If the X

(1) ∠CPD = 80°, ∠PCD = 44° interior angles formed A P B


(2) x = 65°, y = 80°, z = 35° by a transversal of two
distinct lines are Q
(3) ∠BCD = 40° C D
supplementary, then
(4) x = 50°, y = 77° t h e t w o l i n e s a re Y
(5) ∠AGE = 126°, ∠GEF = 36°, ∠FGE = 54° parallel.
Given : A transversal XY intersects lines AB and
CD such that ∠BPQ + ∠PQD = 180°
To prove : line AB side CD
26 Master Key Mathematics - II (Geometry) (Std. IX)

Proof : (Indirect method) Theorem - 7 : Corresponding Angles Test


Let us assume line AB is not parallel to
Statement : If a pair of n
line CD.
corresponding angles a
∴ They intersect each other at point T. formed by a transversal of c l
A X b
two lines is congruent m
P D
T then the two lines are
parallel.
B
C Q Given : line n is a transversal for lines l and m
∠a = ∠b
Y
To Prove : lines l line m
In �PQT,
Proof : ∠a + ∠c = 180° ...(i)
∠TPQ + ∠PQT + ∠PTQ = 180°
(Angles in linear pair)
...(Angle sum property of a triangle)
But, ∠a = ∠b ...(ii) (Given)
∴ ∠BPQ + ∠PQD + ∠PTQ = 180° ...(i)
[P - T - B, Q - T - D] ∴ ∠b + ∠c = 180° [From (i) and (ii)]
But ∠BPQ + ∠PQD = 180° ...(ii) (Given) ∴ line l line m
...(By Interior angles test)
∴ ∠BPQ + ∠PQD + ∠PTQ = ∠BPQ +
∠PQD ...[From (i) and (ii)] Corollary 1 :
E
∴ ∠PTQ = 0° Statement : If a line is
A B
∴ lines AB and CD are not two distinct perpendicular to two
L
lines lines in a plane, then the
But, this contradicts the given that two two lines are parallel to C M D
each other. F
lines AB and CD are distinct.
∴ Our assumption that line AB is not Given : line EF ⊥ line AB
parallel to line CD is false. line EF ⊥ line CD
∴ line AB side CD To Prove : line AB line CD
Proof : ∠ELB = 90° ...(i) (Given)
Theorem - 6 : Alternate Angles Test
∠LMD = 90° ...(ii) (Given)
n
Statement : If a pair of ∴ ∠ELB = ∠LMD [From (i) and (ii)]
alternate angles formed a c l
∴ line AB line CD
by a transversal of two b
m ...(By Corresponding angles test)
lines is congruent then
the two lines are parallel. Corollary 2 : p
Given : line n is a transversal for lines l and m Statement : If two lines a
l
∠a = ∠b in a plane are parallel to b
m
To Prove : line l line m a third line in the plane c
n
Proof : ∠a + ∠c = 180° ...(i) then those two lines are
(Angles in linear pair) parallel to each other.

But, ∠a = ∠b ...(ii) (Given) Given : line p is the transversal for


lines l, m and n.
∴ ∠b + ∠c = 180° [From (i) and (ii)]
line l line m
∴ line l line m
...(By Interior angles test) line m line n
To Prove : line l line n
Parallel Lines 27

Proof : line l line m ...(Given) (3) If ∠a ≅ ∠b and ∠x ≅ ∠y, then prove that line l
line n. l m n
on transversal p,
∠a = ∠b ...(i)
(Corresponding angles theorem) k
b a
∴ line m line n ...(Given)
on transversal p, x y
∠b = ∠c ...(ii) Proof :
(Corresponding angles theorem) ∠a ≅ ∠b ...(Given)
∴ ∠a = ∠c [From (i) and (ii)] ∴ line l line m ... (i)
∴ line l line n (Corresponding angles test)
...(By Corresponding angles test) ∠x ≅ ∠y ...(Given)
∴ line n line m... (ii) (Alternate angles test)
∴ line l line n ... [From (i) and (ii)]
PRACTICE SET - 2.2 (Textbook Page No. 21)
l (4) In the following figure, if ray BA ray DE,
(1) In the given figure, ∠C = 50° and ∠D = 100°. Find the measure of
∠y = 108° and m
∠ABC.
x
∠x = 71°. Are the
y [Hint : Draw a line passing through point C and
lines m and n n parallel to line AB]
parallel? Justify. D E
A B
100°

Proof : ∠y = 108° ...(i) 50°


∠x = 71° ...(ii)
C F
Adding (i) and (ii)
Construction : Through C, draw a line parallel to line AB.
∠x + ∠y = 71 + 108
Solution : line AB line CF. ...(i) (Construction)
∴ ∠x + ∠y = 179°
line AB line DE. ...(ii) (Given)
∠x and ∠y forms a pair of interior angles
∴ line DE line CF. ...[From (i) and (ii)]
Since, they are not supplementary
line m is not parallel to line n On transversal DC,
∠EDC + ∠DCF = 180°
(2) In the given figure, n
...(Interior angles theorem)
if ∠a ≅ ∠b then prove a l
∴ 100 + ∠DCF = 180
that line l line m. c
∴ ∠DCF = 180 – 100
b m ∴ ∠DCF = 80°
Proof : Consider 'c' as
shown in the ∠BCF = ∠BCD + ∠DCF
figure. ...(Angle addition property)

∠a = ∠b ...(i) (Given) ∴ ∠BCF = 50 + 80

∠a = ∠c ...(ii) (Vertically opposite angles) ∴ ∠BCF = 130°

∴ ∠b = ∠c ...[From (i) and (ii)] line AB line CF

∴ line l line m On transversal BC,


...(By Corresponding angles test) ∠ABC = ∠BCF
...(Alternate angles theorem)
∴ ∠ABC = 130°
28 Master Key Mathematics - II (Geometry) (Std. IX)

Alternate Method D E ∴ ∠EAB = ∠ABD


A B 100° ...(Alternate angles theorem)
M N
50°
∴ ∠EAF + ∠BAF = ∠ABC + ∠DBC
...(Angles addition property)
C x+x=y+y ...[From (i) and (ii)]
∴ 2x = 2y
Construction : Extend ray AB to intersect seg DC at
point M, such that D-M-C. ∴ x=y
Solution : line AN line DE. ...(Construction) ∴ ∠FAB = ∠ABC
On transversal DC, ∴ line AF line BC
...(By Alternate angles test)
∠EDC = ∠NMC
...(Corresponding angles theorem) (6) A transversal EF of line AB and line CD
∴ ∠NMC = 100° intersects the lines at point P and Q respec-
∴ ∠AMC + ∠NMC = 180° tively. Ray PR and ray QS are parallel and
...(Angles in linear pair) bisectors of ∠BPQ and ∠PQC respectively.
Prove that line AB line CD.
∴ ∠AMC + 100 = 180
∴ ∠AMC = 80° E
A P B
In �BMC,
S
∠BMC + ∠BCM + ∠CBM = 180° R
...(Angle sum property of a triangle) C Q D
∴ 80 + 50 + ∠CBM = 180
∴ 130 + ∠CBM = 180 Solution : F

∴ ∠CBM = 180 – 130 Let, ∠BPR = ∠QPR = x ...(i)



( ray PR bisects ∠BPQ)
∴ ∠CBM = 50°
∠CQS = ∠PQS = y ...(ii)
∴ ∠ABC + ∠CBM = 180° ∴
( ray QS bisects ∠PQC)
...(Angles in linear pair)
∴ ∠BPQ = ∠BPR + ∠QPR
∴ ∠ABC + 50 = 180
...(Angles addition property)
∴ ∠ABC = 180 – 50
∴ ∠BPQ = x + x ...[From (i)]
∴ ∠ABC = 130°
∴ ∠BPQ = 2x ...(iii)
(5) In the given figure, ray AE ray BD. ray AF is Similarly, we will get
the bisector of ∠EAB and ray BC is the bisector
∴ ∠PQC = 2y ...(iv)
of ∠ABD.
ray PR ray QS ...(Given)
Prove that line AF line BC.
F On transversal PQ,
E
x ∴ ∠QPR = ∠PQS
x B
...(Alternate angles theorem)
A y
y ∴ x=y ...[From (i) and (ii)]
D
Proof : C
∴ 2x = 2y ...(Multiplying throughout by 2)
∠EAF = ∠BAF = x ...(i) ∴
∴ ∠BPQ = ∠PQC ...[From (iii) and (iv)]
( ray AF bisects ∠EAB)
line AB line CD
∠DBC = ∠ABC = y ...(ii)
∴ ...(By Alternate angles test)
( ray BC bisects ∠ABD)
ray AE ray BD ...(Given)
On transversal AB,
Parallel Lines 29

PROBLEMS FOR PRACTICE PROBLEM SET - 2 (Textbook Page No. 22)


(1) Line AB line CD and line EF is the (1) Select the correct alternative and fill in the
transversal. blanks in the following statements.
ray PX and ray QY are the bisectors of ∠EPB
(i) If a transversal intersects two parallel lines then
and ∠PQD respectively E the sum of interior angles on the same side of
To prove : ray PX ray QY. X
the transversal is .......... . n
A o B
P o (a) 0° (b) 90° l
Y (c) 180° (d) 360° a
× b
C Q × D Explanation : m

line l line m
F
On transversal n,
(2) From the information given in the adjoining
∠a + ∠b = 180° ...(Interior angles theorem)
figure, prove ray AB ray EF
B A If a transversal intersects two parallel lines
then the sum of interor angles on the same
57°
side of the transversal is 180°.
(ii) The number of angles formed by a transversal
E F of two lines is .......... .
22° 145°
n
(a) 2 (b) 4
35° 1 2 l
(c) 8 (d) 16 3 4
C D
(3) From the information given in the adjoining Explanation : 5 6
figure, prove line AB line CD 7 8 m

P line n is the transversal for lines l and m.


A x 75° B The number of angle formed by a transversal
of two lines is 8.
C 105° y D
(iii) A transversal intersects two parallel lines. If
the measure of one of the angles is 40° then the
Q
measure of its corresponding angle is .......... .
(a) 40° (b) 140° n
(4) ∠EAB = 110°, ∠FBH = 70°.
(c) 50° (d) 180° a = 40°
Prove that ray AE line BC l
Explanation : c
A E b
m
110°
line l line m
F On transversal n,
70° B C ∠a = ∠b ...(Corresponding angles theorem)
H ∠a = 40°
∴ ∠b = 40°
A transversal intersects two parallel lines. If
the measure of one of the angles is 40° then
the measure of its corresponding angle is 40°.
30 Master Key Mathematics - II (Geometry) (Std. IX)

(iv) In �ABC, ∠A = 76°, ∠B = 48°, then ∠C = ........... (ii) A pair of supplementary angles
A (a) ∠QPR and ∠BPA
(a) 66° (b) 56°
76°
(c) 124° (d) 28° (iii) A pair of congruent angles
(a) ∠QPR and ∠BPA
Explanation :
(b) ∠QPB and ∠RPA
In �ABC, B 48° ?° C
∠A + ∠B + ∠C = 180° (3) Prove that, if a line is perpendicular to one of
...(Sum of measures of all angles the two parallel lines, then it is perpendicular
of a triangle is 180°) to the other line also.
∴ 76 + 48 + ∠C = 180 Solution :
∴ 124 + ∠C = 180 Given : Line AB line CD and line EF intersects them
∴ ∠C = 180 – 124 at points M and N respectively.
∴ line EF ⊥ line AB
∠C = 56° E
To prove : line EF ⊥ line CD
In �ABC, ∠A = 76°, ∠B = 48°,
A B
then ∠C = 56°
M
(v) Two parallel lines are intersected by a
C D
transversal. If measure of one of the alternate
interior angles is 75° then the measure of the N
other angle is .......... . n Proof : line AB line CD F ...(Given)
(a) 105° (b) 15° On transversal EF
l
(c) 75° (d) 45° a = 75° ∠EMB = ∠MND ...(i) (Corresponding
b
Explanation : m angles theorem)
line l line m But, ∠EMB = 90° ...(ii) (Given)
∴ ∠MND = 90° ...[From (i) and (ii)]
On transversal n,
∠a = ∠b ...(Alternate angles theorem) ∴ line EF ⊥ line CD
∠a = 75°
(4) In the adjoining figure, measures of some
∴ ∠b = 75° angles are shown. Using the measures find the
Two parallel lines are intersected by measures of ∠x and ∠y and hence show that
a transversal. If measure of one of the line l line m. E
alternate interior angles is 75° then the A 130° M B
l
measure of the other angle is 75°. x
C D
(2) Ray PQ and ray PR are perpendicular to each 50°
m
Proof : y N
other. Points B and A are in the interior and
∠BMN = ∠AME F
exterior of ∠QPR respectively. Ray PB and
...(Vertically opposite angles)
ray PA are perpendicular to each other. Draw
∴ ∠x = 130°
a figure showing all these rays and write :
(i) A pair of complementary angles ∠CNF = ∠MND
...(Vertically opposite angles)
(ii) A pair of supplementary angles
∴ ∠y = 50°
(iii) A pair of congruent angles.
∠BMN + ∠MND = 130 + 50
Solution : Q
∴ ∠BMN + ∠MND = 180°
(i) A pair of B ∴ line l line m ...(By Interior angles test)
complementary angles
(a) ∠QPB and ∠BPR P R
(b) ∠BPR and ∠RPA
A
Parallel Lines 31

(5) In the given figure, line AB line CD line ∴ ∠c = 80°


EF and line QP is their transversal. line q line r ...(Given)
If ∠y = : ∠z = 3 : 7 Q On transversal p,
A B
then find the x ∠f = ∠b
measure of ∠x. C y D ...(Corresponding angles theorem)
E z F ∴ ∠f = 100°

P ∠g = ∠c
Solution : ...(Corresponding angles theorem)
∠y = ∠z = 3 : 7 ...(Given) ∴ ∠g = 80°
Let the common multiple be a (7) In the adjoining AB CF and line BC line ED
P
∠y = 3a ∠z = 7a then prove that ∠ABC = ∠FDF. A
line AB line EF ...(Given)
B C
On transversal PQ, Proof :
∴ ∠x = ∠z E D
...(Alternate angles theorem)
line AB line CF F ...(Given)
∠x = 7a
On transversal BC,
line AB line CD ...(Given)
∴ ∠ABC = ∠BCD ...(i)
On transversal PQ,
(Alternate angles theorem)
∠x + ∠y = 180°
line BC line ED ...(Given)
...(Interior angles theorem)
∴ On transversal CF,
7a + 3a = 180
∴ ∠FDE = ∠BCD ...(ii)
∴ 10a = 180
(Corresponding angles theorem)
180
∴ a = ∴ ∠ABC = ∠FDE ...[From (i) and (ii)]
10
∴ (8) In the given figure, P
a = 18
line PS is a A
∠x = 7a Q B
transversal of
∴ ∠x = 7 × 18 Y
parallel line AB and X
∴ ∠x = 126° line CD. If ray QX, C R D
(6) In the adjoining ray QY, ray RX, S
p
figure, if b ray RY are angle
a
line q line r c q bisectors, then prove that QXRY is a rectangle.
d
and line p is their f Proof :
e r
[Rays QX, QY,
transversal and if g h Let, ∠AQX = ∠RQX = a ...(i) RX, RY are the
∠a = 80°, find the ∠BQY = ∠RQY = b ...(ii) bisectors of
measrues of ∠f and
∠CRX = ∠QRX = c ...(iii) ∠AQR, ∠BQR,
∠g.
∠DRY = ∠QRY = d ...(iv) ∠QRC, ∠QRD
Solution : respectively]
∠a + ∠b = 180° ∠AQR + ∠BQR = 180°
...(Angles in linear pair) ...(Angles in linear pair)
∴ 80 + ∠b = 180 ∴ 2a + 2b = 180
∴ ∠b = 100° ∴ a+b = 90
∠c = ∠a ∴ ∠RQX + ∠RQY = 90° ...[From (i) and (ii)]
...(Vertically opposite angles)
32 Master Key Mathematics - II (Geometry) (Std. IX)

∴ ∠XQY = 90° ...(v) (C) d and q (D) c and d


(Angles addition property) (4) Which are the pairs
of interior angles? a b
Similarly, we can prove d c
c+d = 90 (A) a, b and a, d
p q
(B) b, d and d, p
∠XRY = 90° ...(vi) s r
(C) c, q and q, r
line AB line CD ...(Given)
(D) c, q and d, p
On transversal PS, P
(5) If line l is parallel
∠AQR + ∠CRQ = 180° A
to line m and line Q l
...(Interior angles theorem)
PS is a transversal.
∴ 2a + 2c = 180
If ∠DRS = 65° what R
m
∴ a + c = 90 ...(vii) D
is ∠AQR?
In �XQR, (A) 65° (B) 125°
S

∠QXR + ∠XQR + ∠XRQ = 180° (C) 115° (D) 105°


...(Angle sum property of a triangle)
(6) If line l line m and C
∴ ∠QXR + a + c = 180 ...[From (i) and (iii)] ∠GFB = 60°, l
∴ ∠QXR + 90 = 180 ...[From (vii)] ∠DBF = ......... A B D
∴ ∠QXR = 90° ...(viii) (A) 60° (B) 120° C F G
m

Similarly, we can prove (C) 180° (D) 70° H


∠QYR = 90° ...(ix) (7) Interior angles theorem states that
In QXRY, (A) If lines are parallel then interior angles are
∴ ∠XQY = ∠QXR = ∠XRY = ∠RYQ = 90° congruent
...[From (v), (vi), (viii) and (ix)] (B) If lines are parallel then interior angles are
∴ QXRY is a rectangle ...(By definition) supplementary
(C) If interior angles are congruent then lines are
MCQ’s parallel

(1) What kind of angles are c and q? (D) If interior angles are supplementary then
lines are parallel
(A) Corresponding
angles
a b (8) line l line m, ∴ ∠RPA = ∠PQC (.........)
d c x
(B) Linear pair angles (A) Corresponding R
p q
angles test A B
(C) Alternate angles s r l
(B) Corresponding P
(D) Interior angles
angles theorem m
(2) What kind of angles C Q D
(C) Interior angles
are b and q? S
a b theorem
(A) Corresponding d c (D) Alternate angles test
angles
p q (9) line AB line CD,
(B) Linear pair angles s r
∴ m∠APQ + m∠CQP = 180° (.........) x
(C) Alternate angles R
(A) Linear pair axiom
(D) Interior angles A B
a b (B) Corresponding P
(3) Which is a pair of d c angles theorem
alternate angles? C Q D
p q (C) Interior angles
(A) c and q s r theorem S
(B) a and q (D) Alternate angles theorem
Parallel Lines 33

x B
(10) In the adjoining R (17) Ray AB ray CE,
A 56°
figure, if A B ∠APC = 100°,
line AB line CD, 60° P ∠BAD = 56°,
100° P
∠PQD = ......... ∠PCD = .........
C Q D
(A) 60° (A) 136° (B) 44°
S
(B) 120° (C) 100° (D) 124° C D E
(C) 140° (D) 30° x
R l m n
(11) In the adjoining A B (18) In the,
figure, if 60° P adjoining
line AB line CD, figure, ∠x p
a b
∠CQS = ......... C Q D and ∠y are
S which type of x y
(A) 30° (B) 120°
x angles?
(C) 140° (D) 60° R
(A) Corresponding angles
(12) In the adjoining A B
figure, if 60° P (B) Alternate angles
line AB line CD, (C) Interior angles m
C Q D l n
∠QPB = ......... (D) Supplementary anges
S
(A) 60° (B) 120° (19) If ∠a = ∠b, then
(C) 140° (D) 30° R
x which of the b
a
(13) In the adjoining following is
A B y
figure, if true? x
60° P
line AB line CD, (A) Line l line m
∠SQD = ......... C Q D (B) Line l line n
(A) 60° (B) 110° S
(C) Line m line n (D) Line l ⊥ line m
(C) 120° (D) 30° (20) If a transversal intersects two parallel lines such
(14) Identify the pair of A that the ratio between the interior angles on one
corresponding angles, of its side is 2 : 7, then what is the measure of
E F
if ray BA ray EP and the greater angle?
BC is transversal. (A) 20° (B) 140° (C) 150° (D) 120°
(A) ∠ABC and ∠BPE B P C
(B) ∠ABP and ∠PEF ANSWERS
(C) ∠ABC and ∠EPC (1) (D) (2) (A) (3) (C) (4) (D)
(D) ∠ABC and ∠EPB (5) (C) (6) (B) (7) (B) (8) (B)
(15) If line l line m, Q P (9) (C) (10) (A) (11) (D) (12) (B)
l
∠PEF = 100°, E 100° (13) (C) (14) (C) (15) (B) (16) (C)
∠EFR = ......... m (17) (B) (18) (B) (19) (A) (20) (B)
S F R
(A) 100°
(B) 80° B
(C) 180° (D) 120° A 56°

(16) If ∠APC = 100° then


100° P
∠CPD = .........?
(A) 280° (B) 180°
(C) 80° (D) 56° C D E
Parallel Lines 34

ASSIGNMENT - 2
Time : 1 Hour Marks : 20

Q.1. Solve the following : p (10)


b a
(1) line q line r and line p is the transversal, q
c
∠a = 80°, find ∠f and ∠g: d
f e r
g h

l
(2) In the given figure, ∠y = 108° and ∠x = 71°
m
check whether lines m and n are x
parallel or not? Justify you answer: y
n

n
(3) In the given figure, if a = b then
a l
prove : line l line m

b m

P
A

(4) Line AB line CF and B C


line BC line ED.
Prove : ∠ABC = ∠FDE E D

F
(5) Prove : If a line is perpendicular to one of the two parallel lines, then it is perpendicular to
the other line also.
Q.2. Solve the following: (6)
(1) Prove : The sum of the measures of all angles of a triangle in 180° D E
(2) If ray BA ray DE, ∠C = 50° and A B
100°
∠D = 100°, find ∠ABC
50°

Q.3. Solve the following: P (4)


(1) In the given figure, line AB line CD and A Q B
line PS is a transversal. Ray QX, ray QY Y
X
ray RX and ray RY are the angle bisectors.
C R D
Prove QXRY is a rectangle.
S

vvv
Triangles 35

3 Triangles

MaSTER KEy QuESTIon SET - 3


Points to Remember :
PRacTIcE SET - 3.1 (Textbook Page No. 27)
Theorem - 1
(1) In the following figure, for ÐacD is an exterior
l Remote interior angles theorem. A angle of DaBc.
Statement: ÐB = 40º, ÐA = 70º.
A
The measure of an exterior Find the measure of
angle of a triangle is ÐacD.
B C D
equal to the sum of the
measures of its remote B Solution:
ÐACD is an exterior angle of DABC.
C
(Given)
D
interior angles.
Given : ÐACD is an exterior angle of DABC. ÐACD = ÐA + ÐB (Remote Interior angles
To prove : ÐACD = ÐABC + ÐBAC theorem)
Proof : ÐACD + ÐACB = 180º ... (i) \ ÐACD = 70 + 40 (Given)
(Linear pair axiom) \ ÐACD = 110°
InD ABC, ÐABC + ÐBAC + ÐACB = 180º ...(ii)
(2) In DPQR, ÐP = 70º, ÐQ = 65ºthen find ÐR.
(Sum of the measures of all angles of a
Solution:
triangle is 180º)
In DPQR, ÐP + ÐQ + ÐR = 180º
\ ÐACD + ÐACB = ÐABC + ÐBAC + ÐACB
[From (i) and (ii)] (Sum of the measures of all angles of a
triangle is 180º)
\ ÐACD = ÐABC + ÐBAC (Cancelling
common angle) \ 70 + 65 + ÐR = 180
\ 135 + ÐR = 180
l Very Important: For any three non zero real
numbers a, b and c. If a > b + c then a > b and a > c. \ ÐR = 180 – 135
\ ÐR = 45º
Theorem - 2
(3) The measures of angles of a triangle are xº,
l Exterior angle property of a triangle.
(x – 20)º and (x – 40º). Find the measure of
Statement: A each angle.
The exterior angle of Solution:
a triangle is greater As xº, (x – 20)º and (x – 40)º are measures of
than each of its remote B angles of a triangle,
interior angles.
C D
\ xº + (x – 20)º + (x – 40)º = 180º
Given : ÐACD is an exterior angle of DABC. (Sum of the measures of all angles of
To prove : (1) ÐACD > ÐABC, a triangle is 180º)
(2) ÐACD > ÐBAC \ x + x – 20 + x – 40 = 180
Proof : ÐACD is exterior angle of DABC(Given) \ 3x – 60 = 180
\ ÐACD = ÐABC + ÐBAC \ 3x = 180 + 60
(Remote interior angles theorem)
\ 3x = 240
Measures of ÐACD, ÐABC and ÐBAC
are non zero real numbers. \ x = 240 = 80
3
\ ÐACD > ÐABC and ÐACD > ÐBAC Measure of first angle = x = 80º

(35)
36 Master Key Mathematics - II (Geometry) (Std. IX)

Measure of second angle = (x – 20)º = (80 – 20)º = 60º (6) D In the adjoining
Measure of third angle = (x – 40)º = (80 – 40)º = 40º figure, line aB ||
\ The measures of angles of the triangles
B
R
line DE. Find the
are 80º, 60º, 40º. 40º measures of ÐDRE
70º E and ÐaRE using
(4) The measure of one of the angles of a triangle given measures of
A

is twice the measure of its smallest angle and some angles.


the measure of the other is thrice the measure Solution:
of the smallest angle. Find the measures of the Ray AB || ray DE (Given)
three angles. On transversal AD, ÐBAD @ ÐADE
Solution:
(Alternate angles theorem.)
Let the measure of the smallest angle of the But ÐBAD = 70º
triangle be xº
\ ÐADE = 70º
\ Measure of other two angles will be 2xº and 3xº
\ ÐRDE = 70º .....(i) (A - R - D)
x + 2x + 3x = 180 In DRDE, ÐDRE + ÐRDE + ÐDER = 180º
(Sum of the measures of all angles of triangle is (Sum of the measures of all angles of a triangle is
180º) 180º)
\ 6x = 180
\ ÐDRE + 70 + 40 = 180 [From (i) and given]
\ x = 180 = 30
6 \ ÐDRE = 180 – 110
Measure of the first angle = x = 30º \ ÐDRE = 70º
Measure of the second angle = 2x = 2 × 30 = 60º
\ ÐARE + ÐDRE = 180º (Linear pair axiom)
Measure of the third angle = 3x = 3 × 30 = 90º
\ ÐARE + 70 = 180
\ The measures of angles of the triangle
\ ÐARE = 180 – 70
are 30º, 60º, 90º.
\ ÐARE = 110º
(5) In the adjoining
In DaBc, bisector of Ða
T
(7) C
figure, measures of
E
100º y and ÐB intersect at point
some angles are
o. If mÐC = 70º then find
given. using the
O
ÐaoB.
140º y
Nx z xx
measures find the
y
M R B A
values of x, y, z. Solution:
Solution: Let ÐCBO = ÐOBA = x
ÐNEM + ÐTEN = 180º
\ y + 100 = 180
(Linear pair axiom)
ÐCAO = ÐOAB = y } ... (i)
... (ii)
(Ray BO and AO bisects ÐB and ÐA
\ y = 180 – 100 respectively.)
\ y = 80º
In DABC,
ÐEMN + ÐEMR = 180º (Linear pair axiom)
ÐABC + ÐBAC + ÐACB = 180º
\ z + 140 = 180
(Sum of the measures of all angles of a triangle
\ z = 180 – 140
is 180º)
\ z = 40º
\ ÐCBO + ÐOBA + ÐCAO + ÐOAB
In DENM, ÐENM + ÐNEM + ÐEMN = 180º + ÐACB = 180º ...(Angle addition property)
(Sum of the measures of all angles of a triangle
\ x + x + y + y + 70 = 180 ...From (i) and (ii)
is 180º)
\ x + y + z = 180 \ 2x + 2y = 180 – 70
\ x + 80 + 40 = 180 \ 2(x + y) = 110
\ x = 180 – 120 \ x + y = 55 ... (iii)
\ x = 60º In DAOB,
Triangles 37

ÐAOB + ÐOBA + ÐOAB = 180º Solution:


(Sum of the measures of all angles Ðb = 70º (Vertically Opposite angles)
of a triangle is 180º)
Ðc + 100 = 180 (Linear pair axiom)
\ ÐAOB + x + y = 180 ... (From (iii))
\ Ðc = 180 – 100
\ ÐAOB + 55 = 180
\ ÐAOB = 180 – 55 \ c = 180 – 100

\ ÐAOB = 125º \ Ðc = 80º ...(ii)


In the given triangle
(8) In the adjoining figure,
line aB || line cD Ða + Ðb + Ðc = 180º
and line PQ is [Sum of the measures of all angles of a triangle
A P B
xx
transversal. Ray PT is 180º]
y y T and ray QT bisects \ Ða + 70 + 80 = 180
ÐBPQ and ÐPQD
C Q D \ Ða = 180 – 150
respectively. Prove
that ÐPTQ = 90º. \ Ða = 30º

Proof: (10) D In the adjoining


Let ÐBPT = ÐTPQ = x ... (i) figure, line DE || line
}
G
ÐPQT = ÐTQD = y ... (ii) GF ray EG and ray
(Ray PT and QT bisects ÐBPQ and ÐPQD x yy
FG are bisectors of
respectively.) E
x ÐDEF and ÐDFM
F
respectively.
M
line AB || line CD (Given) 1
Prove that, (i) ÐDEG = ÐEDF (ii) EF = FG.
On transversal PQ, ÐBPQ + ÐPQD = 180º 2
...(Interior angles theorem) Proof:
\ ÐBPT + ÐTPQ + ÐPQT + ÐTQD Let ÐDEG = ÐGEF = x ...(i)
= 180º ...(Angle addition property) ÐDFG = ÐGFM = y ...(ii)
\ x + x + y + y = 180 ... From (i) and (ii) (Ray EG and FG bisects ÐDEF and ÐDFM
\ 2x + 2y = 180 respectively.)
\ 2(x + y) = 180 ÐDEF = ÐDEG + ÐGEF
\ x + y = 90 ... (iii) ...(Angle addition property)
In DPTQ, =x+x
ÐTPQ + ÐTQP + ÐPTQ = 180º \ ÐDEF = 2x ...(iii)
...(Sum of the measures of all angles seg ED || seg FG
of a triangle is 180º) On transversal DF,
\ x + y + ÐPTQ = 180 ÐEDF @ ÐDFG ...(Alternate angles theorem)
\ 90 + ÐPTQ = 180 ... [From (iii)] \ ÐEDF = ÐDFG = y ...(iv)
\ ÐPTQ = 180 – 90 In transversal EM,
\ ÐPTQ = 90º
ÐDEF @ ÐGFM ...(Corresponding angles
theorem)
(9) a
using the given
information in the \ ÐDEF = ÐGFM = y ...(v)
figure find Ða, Ðb \ ÐDEF = ÐEDF ...[From (iii), (iv)]
b c 100° and Ðc.
\ y = 2x ...(vi)
70°
ÐGFM is an exterior angle of DGEF.
38 Master Key Mathematics - II (Geometry) (Std. IX)

\ ÐGFM = ÐGEF + ÐEGF ...(Remote


interior angles theorem) Points to Remember:
\ y = x + ÐEGF
q congruence of triangles:
\ 2x = x + ÐEGF (From iv)
\ ÐEGF = 2x – x Now, we introduce the concept of
\ ÐEGF = x ... (vii) congruence of two triangles.

In DEFG, ÐGEF @ ÐEGF ...[From (i), (vii)] The general idea about congruence of
triangles is that, “If two triangles are exactly of
seg EF @ seg FG ...(Converse of isosceles
the same shape and size then the triangles are
triangle theorem)
congruent to each other.”
\ EF = FG
Now consider ∆ABC and ∆PQR in the
following manner.
ProblEms For PrACtiCE P
A
(1) based on information given in the D
a
figure, prove x = a + b + c.
R b T x
× ×
c B C Q R
S ∆ABC and ∆PQR are of same shape and size
(2) The exterior angles, obtained
by producing the base of a triangle both ways therefore they are congruent.
are 104° and 136°. Find the measures of all When you place them one above the other
angles of the triangle. vertex A will coincide with vertex P
(3) If the sides of the triangle are produced in Symbolically we write,
order, prove that sum of the exterior angles so A↔P (i.e. A corresponds to P or
formed is equal to four right angles. P corresponds to A)
(4) the side bC of DABC is produced such that B↔Q
D is on ray BC. The bisector of ÐA meets bC C↔R
in point L. Prove that mÐAbC + mÐACD =
In mathematical language, such pairing of
2mÐAlC.
vertices is called a one to one correspondence
A
between the vertices of two triangles.
The above one to one correspondence is written
as ABC ↔ PQR.
There are six possible correspondences between
the vertices of the ∆ABC and ∆PQR.
B L C D
Namely,
(5) In the following figure, seg AC ^ ray CE. ABC ↔ PQR ABC ↔ RQP
mÐA : mÐb : mÐC = 3 : 2 : 1. Find mÐECD.
ABC ↔ PRQ ABC ↔ QPR
A
ABC ↔ RPQ ABC ↔ QRP
E But ∆ABC will exactly fit on ∆PQR with
ABC ↔ PQR.
∆ABC will not exactly fit on ∆PQR with
B C D
ABC ↔ QRP or ABC ↔ PRQ.
\ ∆ABC ≅ ∆PQR under ABC ↔ PQR.
anSwER Therefore, correspondence plays a very
important role in congruence of triangles.
(2) 76°, 44°, 60° (5) mÐECD = 60°
Triangles 39

Now we define congruence of two triangles. In ∆ABC and ∆PQR,


“If there exists atleast one, one to one For the correspondence
correspondence between the vertices of any ABC ↔ PQR,
two triangles such that corresponding sides If, side AB ≅ side PQ
and angles of one triangle are congruent with ∠ABC ≅ ∠PQR
corresponding sides and angles of other triangle
side BC ≅ side QR then
then these two triangles are congruent.”
∆ABC ≅ ∆PQR.
l Properties of congruent triangles:
(2) Angle-side-Angle (AsA) test: For a given one-
The following are the properties of congruent
to-one correspondence between the vertices of
triangles:
any two triangles, when two angles are included
(i) Reflexivity : Every triangle is congruent to itself
side of one triangle congruent to corresponding
i.e. ∆ABC ≅ ∆ABC.
two angles and included side of another triangle
(ii) Symmetry : If ∆ABC ≅ ∆PQR then ∆PQR ≅
then the two triangles are congruent.
∆ABC.
(iii)Transitivity : If ∆ABC ≅ ∆PQR and ∆PQR ≅
L X

∆XYZ then ∆ABC ≅ ∆XYZ.


l Test of congruence of two triangles:
We know, for congruence between two triangles,
M N Y Z

six elements (3 sides and 3 angles) of one triangle In ∆LMN and ∆XYZ,
are congruent to corresponding six elements of For the correspondence
other triangle. LMN ↔ XYZ,
But to establish or to prove the congruence of If ∠LMN ≅ ∠XYZ
two triangles, it is not necessary to know or side MN ≅ side YZ
prove the congruence of six elements.
∠MNL ≅ ∠YZX
If we are given only the congruence of some then ∆LMN ≅ ∆XYZ.
particular three elements, then the triangles will
(3) side-side-side (sss) test: For a given one-to-
be congruent to each other.
one correspondence between the vertices of any
Therefore out of six conditions of congruence
two triangles when three sides of one triangle
between two triangles, three particular conditions
are respectively congruent to corresponding
are sufficient.
three sides of another triangle, then the two
We shall state these sufficient conditions as tests
triangles are congruent.
of congruence.
Test of congruence:
D K
l

(1) side-Angle-side (sAs) test: For a given one-


to-one correspondence between the vertices
of any two triangles when two sides and the F E H G
angle included by them, of one triangle are
respectively congruent to corresponding two In ∆DEF and ∆KGH,
sides and the angle included by them, of another For the correspondence
triangle, then the two triangles are congruent. DFE ↔ KGH,
A P If, side DE ≅ side KG
side FE ≅ side HG
side DF ≅ side KH
Q R then ∆DEF ≅ ∆KGH.
B C
40 Master Key Mathematics - II (Geometry) (Std. IX)

(4) side-Angle-Angle (sAA) test: For a given one- observe the figures and state the test by which
to-one correspondence between the vertices of the triangles in each pair are congruent.
any two triangles, when a side, an angle adjacent (i) A P
to it and the angle opposite to it of one triangle
are respectively congruent to corresponding
side, an angle adjacent to it and the angle
opposite to it, of another triangle, then the two B C Q R
triangles are congruent. \ By SSS test DABC @ DPQR
R U (ii) X L

S T W V Y Z M N
In ∆RST and ∆UWV,
\ By SaS test DXYZ @ DLMN
For the correspondence
RST ↔ UWV, (iii) P S

If, side ST ≅ side WV,


∠RST ≅ ∠UWV
∠SRT ≅ ∠WUV × ×
Q R T U
then ∆RST ≅ ∆UWV. \ By aSa test DPRQ @ DSTU
(5) Hypotenuse - Side Test: For a given one-to-one
(iv) M T
correspondence between the vertices of any L P
two triangles, two right angled triangles are
congruent of the hypotenuse and a side of one
triangle are congruent to the hypotenuse and
corresponding side of the other triangle.

A P N R
\ By Hypotenuse-side test, DLMN @ DPTR

(2) observe the information shown in pairs of


triangles given below. State the test by which
B C Q R the two triangles are congruent. write the
remaining congruent parts of the triangles.
In ∆ABC and ∆PQR,
(i) A P
For the correspondence
ABC ↔ PQR,
If, ∠ABC = ∠PQR = 90°
× ×
Hypotenuse AC ≅ Hypotenuse PR B C Q R
Side AB ≅ side PQ Solution:
then ∆ABC ≅ ∆PQR. From the information given in the figure
In DABC and DPQR
PRacTIcE SET - 3.2 (Textbook Page No. 31) \ ÐABC @ ÐPQR (Given)

(1) In each of the examples given below, a pair side BC @ side QR (Given)
of triangle is shown. Equal parts of triangles \ ÐACB @ ÐPRQ (Given)
in each pair are marked with the same signs. \ DABC @ DPQR (ASA test)
Triangles 41

\ ÐBAC @ ÐQPR ... (c.a.c.t) Solution:


In DLMN and DPNM,
seg AB @ seg PQ ... (c.s.c.t)
side LM @ side PN (Given)
seg AC @ seg PR (c.s.c.t.)
side LN @ side PM (Given)
(ii) P R side MN @ side NM (Common side)
DLMN @ DPNM (SSS test)
ÐLMN @ ÐPNM
T

Q S
ÐLNM @ ÐPMN
ÐNLM @ ÐMPN
} (c.a.c.t.)

Solution:
B
From the information given in the figure. (5) In the adjoining figure,
In DPTQ and DSTR seg aB @ seg cB and
C seg aD @ seg cD.
seg PT @ seg ST (Given)
A
Prove that DaBD @ DcBD.
ÐPTQ @ ÐSTR (Vertically opposite angles)
seg TQ @ seg TR (Given) D
\ DPTQ @ DSTR (SAS test) Proof:
\ ÐTPQ @ ÐTSR and ÐTQP @ ÐTRS In DABD and DCBD,
(c.a.c.t.) side AB @ side CB (Given)
seg PQ @ seg SR (c.s.c.t.) side AD @ side CD (Given)
side BD @ side BD (Common side)
(3) From the information shown in the figure, state \ DAbD @ DCbD (SSS test)
the test assuring the congruence of DaBc and
DPQR. write the remaining congruent parts of (6) Q In the adjoining figure,
the triangles. ∠P @ ∠R and seg PQ @
A B P Q S T seg RQ then prove that
DPQT @ DRQS.
P R
Proof:
In DPQT and DRQS,
C R

Solution: ∠P @ ∠R (Given)
In DABC and DQPR, side PQ @ side RQ (Given)
ÐBAC = ÐPQR = 90º (Given) ∠Q @ ∠Q (Common angle)
Hypotenuse BC @ Hypotenuse PR (Given) \ DPQt @ DrQs (ASA test of congruency)
side AB @ side QP (Given)
\ DABC @ DQPR (Hypotenuse - side test)
ProblEms For PrACtiCE
side AC @ side QR (c.s.c.t.) (1) in DPQR, PQ = PR and ray PS ^ side Qr at
\ ÐABC @ ÐQPR and ÐACB @ ÐQRP points S. Prove ray PS bisects ÐQPr.
(c.a.c.t.) (2) In the following figure, seg BA ^ seg CA, B-D-
P C-E, seg AB @ seg FE and seg bD @ seg EC.
Prove that seg AC @ seg FD.
L
(4) as shown in the A
following figure, in
DLMn and DPnM,
LM = Pn, Ln = PM. D C
M N B
write the test which E
assures the congruence of the two triangles.
write their remaining congruent parts.
F
42 Master Key Mathematics - II (Geometry) (Std. IX)

(3) If altitudes drawn from two vertices of a Proof :


triangle to the opposite sides are equal, then In DPQS and DPRS,
prove that triangle is isosceles. side PS @ side PS (Common side)
(4) In the following figure, seg AB @ seg PB, ÐQ @ ÐR Given)
seg bC @ seg BQ, ÐAbP @ ÐCBQ. Show that ÐQPS @ ÐRPS (Construction)
DAbC @ DPbQ and seg AC @ seg PQ.
\ DPQS @ DPRS (SAA test)
P
\ side PQ @ side Pr (c.s.c.t.)
A

C Theorem - 5
l 30º - 60º - 90º Theorem.
B Q
Statement:
In a right angled triangle if two acute angles are
Points to Remember: 30º and 60º then side opposite to 30º is half of the

Theorem - 3 hypotenuse and side opposite to 60º is 3 times


the hypotenuse 2
l Isosceles Triangle Theorem.
Given : In DABC, ÐBAC = 30º, A
Statement: A
ÐABC = 90º and ÐC = 60º
If two sides of a triangle are 30º
To prove : BC = ½AC
congruent then the angles opposite 60º
to them are congruent. 3
D C
AB = AC
B C B
D
2
Given : In DABC, side AB @ side AC. construction : Take a point D on ray CB such that,
To prove : ÐABC @ ÐACB C - B - D and BC = BD.
construction : Draw bisector of ÐBAC intersecting Proof :
side BC at point D, such that B - D - C.
In DABC and DABD,
Proof :
side AB @ side AB (Common side)
In DABD and DACD,
ÐABC @ ÐABD (Each 90º)
side AB @ side AC (Given)
side BC @ side BD (Construction)
ÐBAD @ ÐCAD (Construction)
\ DABC @ DABD (SAS test)
side AD @ side AD (Common side)
ÐC @ ÐD (c.a.c.t.)
\ DABD @ DACD (SAS test)
But ÐC = 60º (Given)
\ ∠ABD @ ∠ACD (c.a.c.t.)
\ ÐD = 60º
i.e. ∠AbC @ ∠ACb (B - D - C) In DADC, ÐC = 60º, ÐD = 60º
Theorem - 4 \ ÐDAC = 60º (Remaining angle of a triangle)
\ DADC is an equilateral triangle
l converse of Isosceles Triangle Theorem.
Statement: (Q an equiangular triangle is an
equilateral triangle)
If two angles of a triangle are P
congruent then the sides opposite \ AD = CD = AC ... (i) (Sides of equilateral D)
to them are congruent. 1
BC = CD (Construction)
Given : In DPQR, ÐQ @ ÐR Q R 2
To prove : side PQ @ side PR 1 ... (ii) [From (i)]
S
\ bC = AC
construction : Draw bisector of ÐQPR intersecting 2
side QR at point S, such that Q - S - R. In DABC, ÐABC = 90º (Given)
Triangles 43

\ AC2 = AB2 + BC2 (Pythagoras theorem) A In the adjoining figure,


æ1 ö
2
seg AD is median on
\ AC2 = AB2 + çç AC÷÷÷ [From (ii)]
çè 2 ø side BC
B D C
\ AC = AB + AC2
2 21
4
Theorem - 7
\ AC – AC = AB2
2 2
1
Property
l of median drawn on hypotenuse of a
4
\ 4AC2 – AC2 right angled triangle.
= AB2
4
Statement:
\ AB = AC2
2
In a right angled triangle length of the median
3

drawn on hypotenuse is half of the hypotenuse.


4

\ Ab = AC (Taking square roots) Given : In DABC, ÐABC = 90º seg


3
A E
BD is median drawn on
2
D
Theorem - 6 hypotenuse AC.
l 45º - 45º - 90º Theorem.
B C
Statement:
In a right angled triangle, if each acute angle is 1
To prove : BD = AC
2
45º then side opposite to 45º is
1
times the construction : Take a point E on ray BD such that
hypotenuse.
2
B-D-E and BD = DE. Draw seg CE.
Given : In DPQR, ÐQ = 90º, P
Proof : In DADB and DCDE,
ÐP = ÐR = 45º 45º seg AD @ seg CD (D is midpoint of
seg AC)
To prove: PQ = QR = PR
1
Q 45º R ÐADB @ ÐCDE (Vertically opposite
Proof :
2
angles)
In DPQR, ÐP @ ÐR (Given) seg BD @ seg ED (construction)
seg PQ @ seg QR (Converse of isosceles
\ DADB @ DCDE (SAS test)
triangle theorem)
\ ÐBAD @ ÐECD ... (i) (c.a.c.t.)
\ PQ = QR ...(i)
\ seg AB @ seg CE ... (ii) (c.s.c.t.)
In DPQR, ÐQ = 90º (Given) seg AB || seg CE [Alternate angles Test]
On transversal BC,
PR2 = PQ2 + QR2 (Pythagoras theorem)
\ ÐABC + ÐECB = 180° (Interior angles
\ PR2 = PQ2 + PQ2 [from (i)] theorem)
\ 90º + ÐECB = 180°
\ PR2 = 2PQ2
\ ÐECB = 180 – 90° = 90°
\ PR = 2 PQ In DABC and DECB
1 seg AB @ seg EC [from (ii)]
\ PQ = PR ...(ii)
2 ÐABC @ ÐECB (Each measures 90º)
1 seg BC @ seg CB (Common side)
\ PQ = Qr = Pr [from (i), (ii)]
2 \ DABC @ DECB (SAS test)
\ seg AC @ seg EB ... (iii) (c.s.c.t.)
Median
1
BD = BE (Construction)
Definition: A segment joining vertex of a triangle 2
and midpoint of its opposite side is called median \ 1 [from (iii)]
bD = AC
of a triangle. 2
44 Master Key Mathematics - II (Geometry) (Std. IX)

PRacTIcE SET - 3.3 (Textbook Page No. 38) (3) P In DPQR, ÐQ = 90°, PQ = 12,
QR = 5 and QS is a median,
A S
(1) Find the values of x and y 12 Find l(QS).
using the information
x Q R
C shown in figure. Find the
50° 5
B 60° y
measure of ÐaBD and
mÐacD. Solution:
In DPQR, ÐPQR = 90º (Given)

D \ PR2 = PQ2 + QR2 (Pythagoras theorem)


Solution: \ PR2 = 122 + 52 (Given)
In DABC, side AB @ side AC (Given)
\ PR2 = 144 + 25
\ ÐABC @ ÐACB (Isosceles triangle
theorem) \ PR2 = 169
\ x = 50º (Q ÐACB = 50º) \ PR = 13 units ... (i) (Taking square roots)
In DDBC, side DB @ side DC (Given)
seg QS is median on hypotenuse PR.
\ ÐDBC @ ÐDCB (Isosceles triangle 1
theorem) \ QS = PR (In a right angled triangle
2
y = 60º the median drawn on the
\ (Q ÐDBC = 60)
hypotenuse is half of the
ÐABD = ÐABC + ÐDBC (Angle addition
property) hypotenuse)
1
\ ÐABD = 50 + 60 \ QS = × 13 [from ...(i)]
2
\ ÐaBD = 110º
\ QS = 6.5 units
ÐACD = ÐACB + ÐDCB (Angle addition
property)
\ ÐACD = 50 + 60 Important note
\ ÐacD = 110º A Statement:
(2) A The length of hypotenuse of a In DABC, seg AD, seg BE and
F G E
right angled triangle is 15. Find seg CF are medians. Medians of
D
the length of median of its a triangle are concurrent (i.e. all
B D C three medians intersect at one
B C hypotenuse.
point).
Given : In DABC, ÐABC = 90º
Point of concurrence of all three medians is called
AC = 15 units
seg BD is median on hypotenuse AC centroid.
To find : BD In the above figure, point G is the centroid.
Solution: Property : Centroid of a triangle trisects the median.
In DABC, ÐABC = 90º (Given)
AG 2 BG 2 CG 2
seg BD is median on hypotenuse AC (Given) i.e. = ; = ; =
GD 1 GE 1 GF 1
1
\ BD = AC We can also say,
2
(In a right angled triangle the median drawn
2 2 2
on the hypotenuse is half of the hypotenuse) AG = AD; BG = BE, CG = CF
3 3 3
1
\ BD = × 15 1 1 1
2 and GD = AD; GE = BE; GF = CF
\ 3 3 3
BD = 7.5 units
Triangles 45

(4) P In adjoining figure


point G is the point Points to Remember :
of concurrence of the
G medians of DPQR. Theorem - 8
If GT = 2.5, find
l Perpendicular Bisector Theorem.
Q R the lengths of PG
T
and PT. case a : Any point on the perpendicular bisector of
a segment is equidisant from its endpoints.
Solution:
In DPQR, point G is centroid (Given)
l Given: Line l is ^ bisector
P of seg AB at point M. Point
PG 2 P is any point on line l.
\ = (Centroid divides the median in
GT 1
the ratio 2:1) To prove : PA = PB
A M B
PG 2
\ =
2.5 1
Proof : Line l is ^ bisector of seg AB (Given)
\ PG = 2 × 2.5 seg AM @ seg BM ... (i)
\ PG = 5 cm ÐPMA = ÐPMB = 90° ...(ii)
In DPMA and DPMB
PT = PG + GT ( P – G – T)
side PM @ side PM (common side)
= 2.5 + 5
ÐPMA @ ÐPMB [from (ii)]
PT = 7.5 cm
side AM @ side BM [from (i)]
\ DPMA @ DPMB (SAS test)
ProblEms For PrACtiCE seg PA @ seg PB (c.s.c.t.)
\ PA = Pb
(1) in DPQR, mÐQ = 90°, seg QM is median.
PQ2 + Qr2 = 169. Find Qm. case B : Any point equidistant from the end points
of a segment lies on the perpendicular bisector of
(2) In the following figure, find the value of x and y.
A
the segment.
Given : Point P is any point equidistant from
40°
P the end points of seg AB. That is,
140° B PA = PB.
y
B x C To prove : Point P is on the perpendicular bisector
(3) In the adjoining figure, G is centroid of DAbC. of seg AB.
If AQ = 6 cm, BR = 9 cm, CG = 5 cm. construction : Take mid-point M of seg AB and
Find: (i) bG (ii) CP (iii) AG draw line PM.
A Proof : In DPAM and DPBM

P seg PA @ seg PB (Given)


P R
seg AM @ seg BM
G
(construction)
B Q C A M B seg PM @ seg PM
(common side)
anSwER \ DPAM @ DPBM
(SSS test)
(1) QM = 6.5 cm (2) x = 20
\ ÐPMA @ ÐPMB (c.a.c.t.)
(3) BG = 6 cm, CP = 7.5 cm, AG = 4 cm
But ÐPMA + ÐPMB = 180º (Linear pair axiom)
46 Master Key Mathematics - II (Geometry) (Std. IX)

ÐPMA + ÐPMA = 180º ...(Q ÐPMB = ÐPMA) i.e. ÐPQA @ ÐRQA (P-M-Q, R-N-Q)
2ÐPMA = 180º \ Ray QA bisects ÐPQR [from (iv) and
\ ÐPMA = 90º definition]
\ seg PM ^ seg AB ...(i) i.e. point A lies on bisector for of ÐPQr
But point M is the midpoint of seg AB. ... (ii)
(Construction) Theorem - 10

\ Line PM is the perpendicular bisector of seg AB. so If two sides of the triangle are not congruent then
point P is on the perpendicular bisector of seg AB. the angle opposite to the greater side is greater.
Theorem - 9 P Given : In PQR, PR > PQ
To prove : ÐPQR > ÐPRQ
l angle bisector theorem.
T construction : Take a point T
case a : If a point is on the angle bisector of an angle,
then it is equidistant from the sides of the angle. on seg PR such that PQ = PT,
Q R
Draw seg QT.
Given: (1) Ray BD bisects ÐABC
A D (2) Point P is on ray BD. Proof : In DPQT, seg PQ @ seg PT (Construction)
M P (3) seg PM ^ ray BA, B-M-A. \ ÐPQT @ ÐPTQ ... (i) (Isosceles triangle
(4) seg PN ^ ray BC, B-N-C. theorem)
B N C
To prove : PM = PN ÐPTQ is exterior angle of DQTR (Definition)
Proof : In DPMB and DPNB \ ÐPTQ > ÐTRQ (Exterior angle
side PB @ side PB (Common side) theorem)

ÐPBM @ ÐPBN (Ray BD bisects, i.e. ÐPTQ > ÐPRQ ... (ii) (P-T-R)
ÐABC) \ ÐPQT > ÐPRQ ... (iii) [from (i) and (ii)]
ÐPMB @ ÐPNB (each 90º) Point T is in the interior of ÐPQR.
\ DPMB @ DPNB (SAA test) \ ÐPQR > ÐPQT ...(iv)
\ seg Pm @ seg PN (c.s.c.t.) \ ÐPQr > ÐPrQ [from (iii) and (iv)]
case B : If the point in the plane of an angle is
Theorem - 11
equidistant from the sides of the angle, then it lies
on the angle bisector. Statement :
Given: Point A is in the interior The sum of the length of any two sides of a triangle
P is greater than the third side.
of ÐPQR.
A Given : D A B C i s a g i v e n
M seg AM ^ ray QP, Q-M-P. D
triangle
seg AN ^ ray QR, Q-N-R.
Q N R A To prove : (i) AB + AC > BC
AM = AN
To prove : Point A lies on the bisector of ÐPQR. (ii) AB + BC > AC
B C
construction : Draw ray QA. (iii) BC + AC > AB
Proof : In DAMQ and DANQ construction : Take point D on ray BA such that
AC = AD.
ÐAMQ @ ÐANQ (Each 90º)
Proof : In DACD, AC = AD (Construction)
Hypotenuse QA @ Hypotenuse QA
(Common side) \ ÐACD @ ÐADC (Isosceles D theorem)
side AM @ side AN (Given) \ ÐACD + ACB > ADC (Adding ÐACB on
DAMQ = DANQ (Hypotenuse-side L-H-S)
test) \ ÐBCD > ÐBDC ...(i) [Angle Addition
\ ÐMQA @ ÐNQA (c.a.c.t.) property and B-A-D]
Triangles 47

In DBCD, Solution:
\ ÐBCD > ÐBDC [From (i)] In DPQR, PQ = 10cm, QR = 12cm,
\ BD > BC (Side opposite to greater) PR = 8cm. (Given)
(angle is greater) \ QR > PQ > PR
\ BA + AD > BC (B-A-D) \ ÐP > ÐR > ÐQ (Angle opposite to
greater side is greater)
\ Ab + AC > bC (Q AD = AC,
\ ÐP is the greatest angle and ÐQ is the
construction)
Similarly we can prove that smallest angle of DPQR
(4) In DFan, ÐF = 80°, Ða = 40°. Find out the
Ab + bC > AC
greatest and the smallest side of the triangle.
bC + AC > Ab State the reason.
Solution:
PRacTIcE SET - 3.4 (Textbook Page No. 43)
In DFAN,
(1) Point a is on the bisector of ÐF + ÐA + ÐN = 180º (Sum of the measures
ÐXyZ. aX = 2 cm. Find aZ. of all angles of a triangle is 180º)
X A
\ 80 + 40 + N = 180° (Given)
\ ÐN = 180 – 120
Y Z
\ ÐN = 60º
Solution: \ ÐF > ÐN > ÐA
Point A lies on bisector of ÐXYZ (Given) \ AN > FA > FN (Side opposite to
\ Point A is equidistant from ray YX and greater angle is greater)
ray YZ. (Angle bisector theorem) \ Side an is greatest side and side Fn is
\ AX = AZ smallest side of DFan.
But, AX = 2cm
\ aZ = 2cm P
(5) Prove that an equilateral
triangle if equiangular.
(2) T In adjoining figure ÐRST
= 56°, seg PT ^ ray ST, Given : In DPQR,
S P
seg PR ^ ray SR and Q R PQ = QR = PR.
R seg PR @ seg PT. Find the To prove: ÐP @ ÐQ @ ÐR
measure of ÐRSP. State Proof:
the reason for your answer. In DPQR, side PQ @ side PR (Given)
Solution: \ ÐQ @ ÐR ..(i) (Isosceles triangle theorem)
In DPQR, side PR @ side QR (Given)
PR = PT (Given)
\ ÐQ @ ÐP (ii) (Isosceles triangle theorem)
\ Point P lies on bisector of ÐRST \ ÐP @ ÐQ @ ÐR [from (i) and (ii)]
(Angle bisector theorem) \ DPQR is an equiangular triangle.
1
\ ÐRSP = ÐRST
2 (6) A Prove that, if the bisector
1 of ÐBac of D aBc is
\ ÐRSP = × 56
2 perpendicular to side Bc,
then DaBc is an isosceles
\ ÐRSP= 28º triangle.
B D C
Given : In DABC, AD bisects
(3) In DPQR, PQ = 10 cm, QR = 12 cm, PR = 8 cm.
ÐBAC, B-D-C.
Find out the greatest and the smallest angle of
seg AD ^ side BC.
the triangle.
To prove : DABC is an isosceles triangle.
48 Master Key Mathematics - II (Geometry) (Std. IX)

Proof: (3) in DABC, side AB @ side bC.


In DADB and DADC If A-P-C then show that BP < congruent
\ ÐBAD @ ÐCAD (Given) sides.
side AD @ side AD (Common side) (4) In the following figure, x > y. Prove that side
\ ÐADB @ ÐADC (Each measures 90°) lm > side lN.
L
\ DADB @ DADC (ASA test)
\ side AB @ side AC (c.s.c.t.) N
\ DABC is an isosceles triangle [by definition] M y
x
P
(7) In adjoining figure, if
seg PR @ seg PQ, show
that seg PS > seg PQ.
Q
Points to Remember:
R S
Similar Triangles
Proof:
In DPQR, side PQ @ side PR (Given) Definition : For a given one-one correspondence,
\ ÐPQR @ ÐPRQ ...(i) (Isosceles triangle two triangles are said to be similar if the:
theorem) (i) corresponding sides are in proportion.
ÐPRQ is exterior angle of DPRS (Definition)
(ii) corresponding angles are congruent.
\ ÐPRQ > ÐS (Exterior angle property)
i.e. For the correspondence DEF XYZ if,
\ ÐPQR > ÐS ...(ii) [from (i)]
DE EF DF
In DPQS, ÐPQS > ÐS [from (ii) Q-R-S] (i) = =
XY YZ XZ
\ (In a triangle side opposite D
Ps > PQ (ii) ÐD @ ÐX X
to greater angle is greater)
(iii) ÐE @ ÐY and
A
(8) In adjoining figure, in DaBc, (iv)ÐF @ ÐZ E F Y Z
E
seg aD and seg BE are
then DDEF ~ DXYZ.
altitudes and aE = BD.
B C note : Converse of the definition is also true
D Prove that seg aD = seg BE
Proof: i.e. if DABC ~ DPQR then.
In DAEB and DBDA AB BC AC
(i) = = (Corresponding sides of
ÐAEB = ÐBDA = 90° (Given) PQ QR PR
similar triangles)
Hypotenuse AB @ Hypotenuse BA
(ii) ÐA @ ÐP (Corresponding
(Common side)

\
seg AE @ seg BD
DAEB @ DBDA
(Given)
(Hypotenuse side test)
(iii) ÐB @ ÐQ and
(iv) ÐC @ ÐR
} angles of similar
triangles)

\ seg BE @ seg AD [c.s.c.t.]


i.e. seg AD @ seg bE PRacTIcE SET - 3.5 (Textbook Page No. 47)
(1) If DXyZ ~ DLMn, write the corresponding
ProblEms For PrACtiCE
angles of the two triangles and also write the
(1) Prove that in a right angled triangle, hypotenuse ratios of corresponding sides.
is the longest side. Solution:
(2) in PQRS, prove that PQ + QR + RS + SP > PR + QS. DXYZ ~ DLMN (Given)
Triangles 49

XY YZ XZ
\ = = (c.s.s.t.) ProblEms For PrACtiCE
LM MN LN
ÐX @ ÐL (1) In the following figure, DCAb ~ DFDE. Find
ÐY @ ÐM
ÐZ @ ÐN } (c.a.s.t) value of m and n.

C
F
(2) In DXyZ, Xy = 4cm, yZ = 6cm, XZ = 5cm
5 2 n 13
If DXyZ ~ DPQR and PQ = 8cm then find the
length of remaining sides of DPQR. B E m D A
12
Solution:
\ DXYZ ~ DPQR (Given) (2) In the following figure, DGHK ~ DPHs
GH : HP = 6 : 5. If KH = 18 units, find KS.
XY YZ XZ
\ = = (c.s.s.t.)
PQ QR PR G
4 6 5
\ = =
8 QR PR
4 6 4 5 S
\ = and =
8 QR 8 PR K H

\ 4 × QR = 6 × 8 and 4 × PR = 5 × 8
6x8 5x8 P
\ QR = and PR =
4 4
\ QR = 12cm and PR 10cm
anSwER

(3) Draw a sketch of a pair of similar triangles. Label (1) m = 4.8; n = 5.2 (2) KS = 33 units
them. Show their corresponding angles by the
same signs. Show the lengths of corresponding PRoBLEM SET - 3 (Textbook Page No. 49)
sides by numbers in proportion.
A (1) Choose the correct alternative answer for the
D following questions.
m

8c
6c

(i) Two sides of a triangle are 5 cm and 1.5 cm.


4c
m

3c

length of third side cannot be ................. .


B C E F
12cm 6cm
(A) 3.7 cm (B) 4.1 cm (C) 3.8 cm (D) 3.4 cm
Solution:
ans. D
In the above figure,
DABC ~ DDEF (ii) In DPQR if ÐR > ÐQ then ................. .
AB BC AC (A) QR > PR (B) PQ > PR
\ = = (c.s.s.t.)
DE EF DF (C) PQ < PR (D) QR < PR
ÐA @ ÐD
ans. B
ÐB @ ÐE
ÐC @ ÐF } (c.a.s.t)
(iii) In DTPQ, ÐT = 65°, ÐP = 95° then which of the
following is a true statement?
(A) PQ < TP (B) PQ < TQ
(C) TQ < TP < PQ (D) PQ < TP < TQ
ans. B
50 Master Key Mathematics - II (Geometry) (Std. IX)

(2) A DABC is an isosceles in (4) A In the adjoining figure,


which AB = AC. seg BD point D and point E are
E D on side bC of DAbC
and seg CE are medians.
Show that BD = CE. such that BD = CE and
B C AD = AE. Show that
B D E C
DAbD @ DACE.
Proof: Proof:
1
AE = BE = AB ...(i) In DADE,
2
(CE is median on side AB) side AD @ side AE (Given)
1 \ ÐADE @ ÐAED ...(i) (Isosceles triangle
AD = CD = AC ...(ii)
2
Theorem)
(BD is median on side AC)
ÐADB + ÐADE = 180° (Linear pair
But, AB = AC ...(iii) axiom)
(Given) \ ÐADB = 180° – ÐADE ...(ii)
\ AE = BE = AD = CD ...(iv) ÐAEC + ÐAED = 180° (Linear pair axiom)
[from (i), (ii) and (iii)] \ ÐAEC = 180 – ÐAED ...(iii)
In DABD and DACE \ ÐADB @ ÐAEC ...(iv) [from (i), (ii), (iii)]
side AB @ side AC (Given) In DABD and DAEC,
ÐA @ ÐA (Common angle) side AD @ side AE ..... (Given)

side AD @ side AE [from (iv)] ÐADB @ ÐAEC ..... (from (iv))


side BD @ side CE ..... (Given)
\ DABD @ DACE [SAS test]
\ side BD @ side CE (c.s.c.t.) \ DAbD @ DACE ..... (SAS test)

ie bD = CE P
(5) In the adjoning figure,
(3) P in DPQR, if point S is on side QR
PQ > Pr and of DPQr. Prove that
s bisectors of Q R PQ + Qr + rP > 2Ps
ÐQ and Ðr S
×
Q × R intersect at S. Proof:
Show that SQ > SR. In DPQS,
Proof: PQ + QS > PS ... (i) (Sum of two sides of a
triangle is greater than the third side)
In DPQR, PQ > PR (Given)
In DPRS,
\ ÐPRQ > ÐPQR (Angle opposite to PR + SR > PS ... (ii) (Sum of two sides of a
greater side is greater) triangle is greater than the third side)
\ PQ + QS + SR + PR > 2PS (Adding (i) and
\ 2ÐSRQ > 2ÐSQR (Rays QS and RS bisect
(ii))
ÐPQR and ÐPRQ respectively)
\ PQ + Qr + rP > 2Ps [Q-S-R]
\ ÐSRQ > ÐSQR ...(i)
A
In DSQR, (6) In the adjoning figure,
b i s e c t o r o f Ð b AC
ÐSRQ > ÐSQR [from (i)]
intersects BC at point
\ sQ > sr (Side opposite to greater D. Prove that Ab > bD.
B C
angle is greater) D
Triangles 51

Proof: ÐCAE + ÐDAE = ÐCAD ... (Angle


ÐBAD @ ÐDAC ... (i) (AD bisects ÐBAC) addition property)
\ ÐDAE = ÐCAD – ÐCAE ....(iii)
ÐADB is an exterior angle of DADC
Now, ÐDAE + ÐDAE = ÐBAE – ÐBAD + ÐCAD – ÐCAE
\ ÐADB > ÐDAC (Exterior angle property)
[Adding (ii) and (ii)]
\ ÐADB > ÐBAD .... (ii) [from (i)] \ 2ÐDAE = ÐBAE – ÐBAD + ÐCAD – ÐBAE
In DABD [from (i)]
ÐADB > ÐBAD [from (ii)] \ 2ÐDAE = ÐCAD – ÐBAD ... (iv)
\ Ab > bD (In a triangle, side opposite In D BAD
to greater angle is greater) ÐBAD + ÐADB + ÐABD = 180° (sum of the
measures of all angles of a triangle is 180°)
(7) S In the adjoining figure,
\ ÐBAD + 90 + ÐB = 180
seg PT is the bisector of
\ ÐBAD = 180 – 90 – ÐB
ÐQPR. A line through R
\ ÐBAD = 90 – ÐB ... (v)
P intersects ray QP at point S.
In DDAC,
To prove : PS = PR
ÐCAD + ÐADC + ÐACD = 180° (sum of the
Q T R measures of all angles of a triangle is 180°)
\ ÐCAD + 90 + ÐC = 180
Proof:
ÐCAD = 180 – 90 – ÐC
In DPQR, ray PT bisects ÐQPR (Given)
ÐCAD = 90 – ÐC ... (vi)
\ ÐQPT @ ÐTPR ... (i)
Now, 2 ÐDAE = (90 – ÐC) – (90 – ÐB)
line PT || line RS (Given)
... [from (iv), (v), (vi)]
On transversal QS,
2 ÐDAE = 90 – ÐC – 90 + ÐB
ÐQPT @ ÐPSR ... (ii) (Corresponding
\ 2 ÐDAE = ÐB – ÐC
angles theorem)
On transversal PR, \ 1
ÐDAE = (Ðb – ÐC)
2
ÐTPR @ ÐPRS ... (iii) (Alternate angles
theorem) MCQ’s
In DPSR, ÐPSR @ ÐPRS [from (i), (ii), (ii)
(Converse of isosceles (1) In DPQR, PQ = PR and mÐP = 40° then mÐQ =
\ side PS @ side PR triangle theorem) ..................... .

\ (A) 140° (B) 40° (C) 70° (D) 80°


Ps = Pr
(2) In DABC, mÐA = 60°, mÐB = 90°, mÐC = 30°,
B AB = 3 cm. AC = ........ (By 30°–60°–90° triangle
(8) In adjoining figure,
D seg AD ^ seg bC . theorem)
E seg AE is the bisector of 3 1
(A) (B) 2 3 (C) 3 (D)
ÐCAb and C - E - D. 2 3
A C (3) If DABC ~ DPQR and mÐA = 40°, mÐB = 50°
1 then mÐR = ...........
Prove that : ÐDAE = (ÐB – ÐC)
2 (A) 70° (B) 80° (C) 90° (D) 55°
Proof: L
(4) In the adjoining figure,
ÐBAE @ ÐCAE ....(i) (Ray AE bisects ÐBAC)
DLPM @ DLPN. (By
ÐBAD + ÐDAE = ÐBAE ... (Angle ...................... test)
addition property) P
M N
\ ÐDAE = ÐBAE – ÐBAD ....(ii)
(A) SSS (B) SAS (C) ASA (D) SAA
52 Master Key Mathematics - II (Geometry) (Std. IX)

(5) The measures of angles of a triangle are in the (14) In the following figure,
ratio 2:3:4. Which of the following is the measure B
m ray AC is bisector of
3c
of an angle of this triangle? A 5 cm C ÐBAD such that AB = 3 cm,
(A) 20° (B) 30° (C) 40° (D) 90° AC = 5 cm then CD =
D
..................... .
(6) P In the adjoining figure,
what is the value of (A) 2 cm (B) 3 cm (C) 4 cm (D) 5 cm
x? (15) DABC @ DPQR and DABC is not congruent
7y 5y x to DRPQ, then which of the following is not
S Q B true?
(A) 35 (B) 45 (C) 50 (D) 60
(A) BC = PQ (B) AC = PR (C) AB = PQ (D) QR = BC
(7) D In the adjoining figure, the
(16) Length of three sides are given below. Determine
value of x is ........ .
which of them will form a triangle?
25

x (A) 65 (B) 80
(A) 6 cm, 3 cm, 2 cm (B) 4.5 cm, 8.5 cm, 4 cm
(C) 95 (D) 120
(C) 5 cm, 6 cm, 10 cm (D) 9 cm, 4 cm, 3 cm
(17) In the following figure, if DABC and DDEF are
55 40
A B C equilateral then what will be ÐC + ÐD + ÐE +
ÐF?
(8) If DABC @ DFDE and AB = 5cm, ÐB = 40° and A
ÐA = 80°, then which of the following is true? D E

(A) DF = 5cm, mÐF = 60°


(B) DE = 5cm, mÐE = 60° C B

(C) DF = 5cm, mÐE = 60° F

(D) DE = 5cm, mÐD = 40° (A) 180° (B) 140° (C) 200° (D) 240°
(9) In DPQR, ÐP = 90°. S is midpoint of side QR. If (18) For DABC, G is its centroid and D is midpoint
QR = 10cm, what is the length of seg PS? of side BC. If AD is 9 cm then GD = .......... .
(A) 10 cm (B) 5 cm (C) 20 cm (D) 7.5 cm (A) 3 cm (B) 4.5 cm (C) 6 cm (D) 18 cm
(10) In DXYZ, ÐX = 90°, ÐY = 60°. If XZ = 5 3 cm, (19) In D ABC, ÐA = ÐC = 45°, ÐB = 90°,
what is the length of seg YZ? AC = 16 2 cm. AB = ........ .
(A) 16 2 cm (B) 8 cm (C) 8 2 cm (D) 16 cm
(A) 10 cm
(B) 10 3 cm (C) 20 cm (D) 3 cm
(20) Using the information given, ÐABC = ........... .
PR 2
(11) If DPQR ~ DXYZ, = and PQ = 12 units A
XZ 3
than XY = ...................... . x
D
(A) 9 units (B) 18 units (C) 8 units (D) 12 units
x
(12) In the following figure, mÐBAC = ................... . B
y y
C
A (A) 30° (B) 60° (C) 90° (D) 120°

anSwERS
94° 126
° (1) (C) (2) (B) (3) (C) (4) (D)
D B C E
(5) (C) (6) (D) (7) (D) (8) (B)
(A) 94° (B) 54° (C) 40° (D) 44°
(9) (B) (10) (A) (11) (C) (12) (C)
(13) In DABC, if ÐB = ÐC = 45°, then which of the
(13) (B) (14) (C) (15) (A) (16) (D)
following is the longest side?
(17) (D) (18) (B) (19) (D) (20) (C)
(A) AB (B) BC (C) AC (D) All sides are equal
Triangles 53

AssiGNmENt - 3
time : 1 Hr. marks : 20

Q.1. Solve the following: 3


(1) If DPQR @ DMNS then state all pairs of congruent angles and sides.:
P
(2) In DPQR, ÐPRQ = 45°, ÐPQS = 100°, S-Q-R.
Find ÐQPR.

100° 45° R
S Q

(3) In DABC, AB = 5 cm, BC = 8 cm, AC = 10 cm. Then find the smallest and the biggest angle
of triangle.
Q.2. Solve the following: 4
(1) In DPQR, ÐQ = 90°, PQ = 12 units, QR = 5 units and seg QS is the median. Find QS.
(2) Based on the informaiton given in the figure, T
find x, y, z if T-E-M, N-M-R 100° E

x z 140°
N M R

Q.3. Solve the following: 9


(1) Prove that if two sides of a triangle are congruent then the angles opposite to them
are congruent.
P
(2) In the adjoining figure, seg PQ @ seg PR. Prove that PS > PQ.

Q R S

(3) In the adjoining figure, line AB || line CD and line PQ is its


transversal. Rays PT and QT bisects ÐBPQ and ÐPQD. A P B
Prove that ÐPTQ = 90°.

C Q D

Q.4. Solve the following: 4


(1) In the adjoining figure, seg AD ^ side BC. seg AE bisects ÐBAC such that B-D-E-C.
1 B
Prove ÐDAE = (ÐB – ÐC) D
2
E

A C
54 Master Key Mathematics - II (Geometry) (Std. IX)

4 Constructions of Triangles
QD = 8.5 cm ...(ii)

Points to Remember: PQ + PD = QD ...(Q-P-D)


∴ PQ + PR = 8.5 cm ...[From (i) and (ii)]
In previous standard we have learnt the following Analytical Figure
triangle constructions.
D D
• To construct a triangle when its three sides are l P
given.

cm
5
• To construct a triangle when its base and two
l

8.
P
adjacent angles are given. 40°
Q

cm
4.2 cm R
• To construct a triangle when two sides and the

5
8.
included angle are given.
• To construct a right angled triangle when its 40°
Q R
hypotenuse and one side is given. 4.2 cm

 Perpendicular Bisector Theorem Steps :


• Every point on (1) Draw seg QR of length 4.2 cm
the perpendicular Q (2) Draw ∠C = 40°; on this ray locate a point D such
bisector of a that QD = 8.5 cm.
segment is (3) Draw seg DR.
equidistant from
A T B
(4) Draw perpendicular bisector of seg DR, it
its end points.
intersect ray QD at point P.
• E v e r y p o i n t l
(5) Draw seg PR.
equidistant
from the end points of a segment is on the (2) Construct ∆XYZ, in which YZ = 6 cm,
perpendicular bisector of the segment. XY + XZ = 9 cm, m∠XYZ = 50°.
Solution: Analytical Figure

MASTER KEY QUESTION SET - 4


A

A
Type I : Construction of triangle, when its base, X
l
the sum of the other two sides and one of the base
m
9c

angles is given. 50°


X Y 6 cm Z
l
PRACTICE SET - 4.1 (Textbook Page No. 53)
m
9c

(1) Construct ∆PQR, in which QR = 4.2 cm,


50°
m∠Q = 40o and PQ + PR = 8.5 cm. Y
6 cm Z
Solution:
Explanation :
Explanation :
Line l is perpendicular bisector of seg AZ
Line l is perpendicular bisector of seg DR
∴ XA = XZ ...(i)
∴ PD = PR ...(i)
(Perpendicular bisector theorem)
(Perpendicular bisector theorem)
(54)
Constructions of Triangles 55

AY = 9 cm ...(ii)
XY + XA = AY ...(A-X-Y) P
∴ XY + XZ = 9 cm ...[From (i) and (ii)]
Steps :
(1) Draw seg YZ of length 6 cm A

cm
(2) Draw ∠Y = 50°; on this ray locate a point A such l

9.8
that YA = 9 cm.
(3) Draw seg AZ.
45°
(4) Draw perpendicular bisector of seg AZ, it C B
5.2 cm
intersect ray YA at point X.
Explanation :
(5) Draw seg XZ.
Line l is perpendicular bisector of seg PB
(3) Construct ∆ABC, in which BC = 6.2 cm, ∴ AP = AB ...(i)
m∠ACB = 50°, AB + AC = 9.8 cm. (Perpendicular bisector theorem)
Solution: Analytical Figure PC = 9.8 cm ...(ii)

P AP + AC = PC ...( P-A-C)
P ∴ AB + AC = 9.8 cm ...[From (i) and (ii)]
cm

A Steps :
9.8

(1) Draw seg CB of 5.2 cm


A 50° (2) Draw an ∠C = 45° at vertex C.
l C 6.2 cm B
(3) On this ray locate a point P such that
cm

CP = 9.8 cm. Draw seg PB.


9.8

50° (4) Draw perpendicular bisector of seg PB,


C B intersecting seg CP at point A.
6.2 cm
Explanation : (5) Draw seg AB.
Line l is perpendicular bisector of seg PB NOTE
∴ AP = AB ...(i) Perimeter of ∆ABC is corrected from 10 cm to 15 cm.
(Perpendicular bisector theorem)
PROBLEMS FOR PRACTICE
PC = 9.8 cm ...(ii)
AC + AP = PC (P-A-C) (1) Draw ∆ABC, where AB = 6 cm, BC = 4 cm
∴ AC + AB = 9.8 cm ...[From (i) and (ii)] and AC = 7 cm.

(4) Construct ∆ABC, in which BC = 5.2 cm, Draw perpendicular bisector of each side of
m∠ACB = 45° and perimeter of ∆ABC is 15 ∆ABC. In how many points do they intersect
cm. one another ?
Solution: Perimeter of ∆ABC = AB + BC + AC. (2) Draw ∆BAD, such that AD = 5.7 cm,
∴ AB + AC + 5.2 = 15 cm ∠BAD = 120° and AB = 4.5 cm.
∴ AB + AC = 15 – 5.2 Draw seg BM perpendicular to line AD.
P
∴ AB + AC = 9.8 cm.
(3) Construct ∆DEF, such that EF = 4.8 cm,
cm

Analytical Figure
A ∠E = 50° and DE + DF = 8.3 cm.
9.8

l
(4) Perimeter of ∆ABC is 14 cm, AB = 4.5 cm and
45°
C 5.2 cm B ∠A = 80°. Construct ∆ABC.
56 Master Key Mathematics - II (Geometry) (Std. IX)

Type II : Construction of a triangle if its base, Explanation :


difference of the other two sides and one of the Line l is perpendicular bisector of seg MR
base angles is given. ∴ PM = PR ...(i)
(Perpendicular bisector theorem)
PRACTICE SET - 4.2 (Textbook Page No. 54) QM = 2.5 cm ...(ii)
(1) Construct ∆XYZ, such that YZ = 7.4 cm, PQ = PM + QM ...(P-M-Q)
m∠XYZ = 45o and XY – XZ = 2.7 cm.
l ∴ PQ = PR + 2.5 ...[From (i) and (ii)]
Solution: Analytical Figure
X ∴ PQ – PR = 2.5 cm

X
Steps of construction :
(1) Draw a segment QR of length 6.5 cm
P (2) Draw an angle measuring 60° at vertex Q, on
cm

this ray mark a point M such that QM = 2.5 cm.


7

P 45°
2.

Y 7.4 cm Z (3) Draw seg RM.


cm

l
7

(4) Draw line l as perpendicular bisector of seg


2.

Y 45°
7.4 cm
MR intersecting ray QM at point P, Q-M-P.
Z
Draw PR.
Explanation : (3) Construct ∆ABC, such that base BC = 6 cm,
Line l is perpendicular bisector of seg PZ m∠ABC = 100° and AC – AB = 2.5 cm.
∴ XP = XZ ...(i) Solution: Analytical Figure
(Perpendicular bisector theorem)
PY = 2.7 cm ...(ii) A

XY = XP + PY ...(X-P-Y)
A
∴ XY = XZ + 2.7 cm ...[From (i) and (ii)]
∴ XY – XZ = 2.7 cm
100°
Steps of construction : B
C
2.5 cm 6 cm
(1) Draw a segment YZ of length 7.4 cm. M
(2) Draw an angle measuring 45° at vertex Y, on this l
ray mark a point P such that YP = 2.7 cm.
(3) Draw seg PZ.
100°
(4) Draw line l as perpendicular bisector of seg PZ B C
6 cm
intersecting ray YP at point X. Draw seg XZ. 2.5 cm

(2) Construct ∆PQR, such that QR = 6.5 cm, M

m∠PQR = 60° and PQ – PR = 2.5 cm. l l


Solution: Analytical Figure
P Explanation :
Line l is perpendicular bisector of seg MC
P ∴ AM = AC ...(i)
(Perpendicular bisector theorem)
BM = 2.5 cm ... (ii)
M
M
AM = AB + BM ...(A-B-M)
cm.5

cm
2

60°
2.5

Q 6.5 cm R 60° ∴ AC = AB + 2.5 cm ...[From (i) and (ii)]


Q 6.5 cm R ∴ AC – AB = 2.5 cm
Constructions of Triangles 57

Steps of construction : Explanation :


(1) Draw a segment BC of length 6 cm Line l and m are perpendicular bisector of
(2) Draw an angle measuring 100° at vertex B, seg PA and PB respectively
extend this ray below seg BC. On this lower ∴ PQ = AQ ...(i) (Perpendicular
side of this ray mark a point M such that and PR = RB ...(ii) bisector theorem)
BM = 2.5 cm.
PQ + QR + PR = 9.5 cm
(3) Draw seg CM.
∴ AQ + QR + RB = 9.5 cm
(4) Draw line l as perpendicular bisector of seg
∴ AB = 9.5 cm
CM, intersecting ray MB at point A, M-B-A.
In �PQA,
Draw seg AC.
seg PQ ≅ seg AQ ...[From (i)]
∴ ∠QPA ≅ ∠QAP
PROBLEMS FOR PRACTICE ...(Isosceles triangle theorem)
Let ∠QPA = ∠QAP = x
(1) Construct ∆LMN in which base MN = 7 cm,
∠LMN = 40° and LM – LN = 3 cm. ∠PQR is an exterior angle of �PQA
∴ ∠QPA + ∠QAP = ∠PQR
(2) Construct ∆XYZ in which base YZ = 7.4 cm ...(Remote interior angle theorem)
∠XYZ = 45° and XY – XZ = 2.9 cm
∴ x + x = 70
(3) Construct ∆LMN such that MN = 6.2 cm, ∴ 2x = 70
∠M = 50° and LN – LM = 2.4 cm. ∴ x = 35
(4) Construct ∆ABC where BC = 4.7 cm, ∴ ∠QPA = ∠QAP = 35°
∠B = 45° and AC – AB = 2.5 cm. Similarly, we can prove ∠RPB = ∠RBP = 40°.
Now, draw �PAB, with AB = 9.5 cm, ∠A = 35°
Type III : Construction of triangle of given and ∠B = 40°
perimeter and base angles. Steps of construction :
(1) Draw a seg AB of length 9.5 cm
PRACTICE SET - 4.3 (Textbook Page No. 56) (2) Draw an angle measuring 35° at vertex A and
(1) Construct ∆PQR, in which ∠Q = 70°, ∠R = 80° an angle measuring 40° at vertex B. Name the
and PQ + QR + PR = 9.5 cm. point as point P where these two rays intersect.
Solution: Analytical Figure (3) Draw line l as perpendicular bisector of seg PA
P intersecting AB at point Q.
l m
(4) Draw line m as perpendicular bisector of seg PB
intersecting AB at point R.
35° 70° 80° 40°
A Q R B (5) Construct seg PQ and seg PR.
l 9.5 cm
(2) Construct ∆XYZ, in which ∠Y = 58°, ∠X = 46°
P m and perimeter of triangle is 10.5 cm.
Solution: Analytical Figure
Z
l m

35° 40° 23° 46° 58° 29°


70° 80° P X 10.5 cm Y Q
Q R B
A

9.5 cm
58 Master Key Mathematics - II (Geometry) (Std. IX)

(3) Construct ∆LMN, in which ∠M = 60°,


l Z
m
∠N = 80° and LM + MN + NL = 11cm.
Solution: Analytical Figure
L
l m

23° 46° 58° 29° 30° 60° 80° 40°


P X Y Q P M 11 cm N Q
10.5 cm

L
Explanation :
l m
Line l and m are perpendicular bisector of
seg PZ and QZ respectively
∴ PX = ZX ...(i) (Perpendicular
ZY = QY ...(ii) bisector theorem) 80° 40°
30° 60°
XY + YZ + XZ = 10.5 cm ...(Given) Q
P M N
∴ XY + QY + PX = 10.5 cm ...[From (i), (ii)] 11 cm
∴ PQ = 10.5 cm ...(P - X - Y - Q) Explanation :
Line l and m are perpendicular bisector of
In �PXZ,
seg PL and seg LQ respectively
seg PX ≅ seg ZX ...[From (i)]
∴ MP = ML ...(i) (Perpendicular
∴ ∠XPZ ≅ ∠XZP bisector theorem)
and NL = NQ ...(ii)
...(Isosceles triangle theorem)
LM + MN + NL = 11 cm ...(Given)
Let ∠XPZ = ∠XZP = x
∴ MP + MN + NQ = 11 cm ...[From (i) and (ii)]
∠ZXY is an exterior angle of �PXZ
∴ PQ = 11 cm ...(P - M - N - Q)
∴ ∠ZXY = ∠XPZ + ∠XZP
...(Remote interior angle theorem) In �PML,

∴ 46 = x+x ∴ seg MP ≅ seg ML ...[From (i)]

∴ 46 = 2x ∠MPL ≅ ∠MLP
...(Isosceles triangle theorem)
∴ x = 23
Let ∠MPL = ∠MLP = x
∴ ∠XPZ = ∠XZP = 23°
∠LMN is an exterior angle of �PML
Similarly, we can prove ∠YQZ = ∠YZQ = 29°.
∴ ∠LMN = ∠MPL + ∠MLP
Now, draw �ZPQ, with PQ = 10.5 cm, ∠P = 23°
...(Remote interior angle theorem)
and ∠Q = 29°
∴ 60 = x+x
Steps of construction :
∴ 60 = 2x
(1) Draw a seg PQ of length 10.5 cm
∴ x = 30
(2) Draw an angle measuring 23° at vertex P and
an angle measuring 29° at vertex Q. Name the ∴ ∠MPL = ∠MLP = 30°
point of intersection of these rays as point Z. Similarly, we can prove ∠NQL = ∠NLQ = 40°.
(3) Draw line l as ⊥ bisector of Seg PZ, intersecting Now, draw �LPQ, with PQ = 11 cm, ∠P = 30°
seg PQ, at point X. and ∠Q = 40°
(4) Draw line m as ⊥ bisector of Seg QZ, intersecting Steps of construction :
seg PQ at point Y. (1) Draw a seg PQ = 11 cm.
(5) Draw seg XZ and seg YZ. (2) Draw an angle measuring 30° at vertex P and an
angle measuring 40° at vertex Q. Name point of
intersection of these rays as point L.
Constructions of Triangles 59

(3) Draw line 'l' as ⊥ bisector of seg PL, intersecting (2) Construct ∆ABC, in which ∠B = 70°,
seg PQ, at point M. ∠C = 60°. AB + BC + AC = 11.2 cm
(4) Draw line as ⊥ bisector of seg QL, intersecting Solution: Analytical Figure
seg PQ at point N. l
A
m
(5) Draw seg LM and seg LN.

35° 70° 60° 30°


PROBLEMS FOR PRACTICE P B 11.2 cm C Q

(1) Construct ∆PQR with perimeter 9.5 cm and


each of base angle is 80°. Write the type of A m
∆PQR. l

(2) Construct ∆ABC such that ∠B = 70°,


∠C = 60° and AB + BC + CA = 10.5 cm.

(3) Construct ∆XYZ whose perimeter is 12 cm 35° 70° 60° 30°


and ∠Y = 70° and ∠Z = 80°. P B C Q
11.2 cm
PROBLEM SET - 4 (Textbook Page No. 56)
(1) Construct ∆XYZ, in which XY + XZ = 10.3 cm, Explanation :
YZ = 4.9 cm. ∠XYZ = 45° Line l and m are perpendicular bisector of
Solution: seg AP and AQ respectively
P ∴ BP = BA ...(i) (Perpendicular
Analytical Figure and CQ = CA ...(ii) bisector theorem)
AB + BC + AC = 11.2 cm ...(Given)
P
X
cm

l
∴ BP + BC + CQ = 11.2 cm ...[From (i), (ii)]
.3
cm

l
10

X ∴ PQ = 11.2 cm ...(P - B - C - Q)
.3
10

In �ABP,
45° 45° seg BP ≅ seg BA ...[From (i)]
Y Z Y 4.9 cm Z
4.9 cm ∴ ∠BPA ≅ ∠BAP
...(Isosceles triangle theorem)
Explanation :
Let ∠BPA = ∠BAP = x
Line l is perpendicular bisector of seg PZ
∠ABC is an exterior angle of �ABP
∴ XP = XZ ...(i)
∴ ∠ABC = ∠BPA + ∠BAP
(Perpendicular bisector theorem)
...(Remote interior angle theorem)
PY = 10.3 cm ...(ii)
∴ ∴ 70 = x+x
XY + XP = PY ...( P-X-Y)
∴ 70 = 2x
∴ XY + XZ = 10.3 cm ...[From (i) and (ii)]
∴ x = 35
Steps of construction :
∴ ∠BPA = ∠BAP = 35°
(1) Draw a seg YZ of length 4.9 cm
Similarly, we can prove ∠CAQ = ∠CQA = 30°.
(2) Draw an angle measuring 45° at vertex Y, on this
Now, draw �APQ, with PQ = 11.2 cm, ∠P = 35°
ray locate point P such that YP = 10.3 cm
and ∠Q = 30°
(3) Draw seg PZ. Draw line as ⊥ bisector of seg PZ
Steps of construction :
intersecting ray YP at point X.
(1) Draw seg PQ of length 11.2 cm
(4) Draw seg XZ.
60 Master Key Mathematics - II (Geometry) (Std. IX)

(2) Draw an angle measuring 35° and 30° at vertex (3) Taking C as a centre and radius 6.4 draw an arc
P and vertex Q respectively. Name the point of intersecting arc. Name point of intersection as
intersection as point A. point A.
(3) Draw line l and line m as ⊥ bisector of seg AP (4) Draw seg AB and seg AC.
and seg AQ respectively. Line l intersects PQ at (4) Construct ∆PQR, in which PQ – PR = 2.4 cm,
point B and line m intersects PQ at point C. QR = 6.4 cm and ∠PQR = 55°
(4) Draw seg AB and seg AC. Solution: Analytical Figure
(3) The perimeter of a triangle is 14.4 cm P
and ratio of lengths of its sides is 2 : 3 : 4.
Construct the triangle. P
Solution:
T
Explanation :

cm
Let the required triangle be ∆ABC.

4
55°

2.
Q 6.4 cm R
AB + BC + AC = 14.4 ...[Given]
AB : BC : AC = 2:3:4 ...[Given] l

Let the common multiple be x


∴ 2x + 3x + 4x = 14.4
T
∴ 9x = 14.4

cm
14.4

4
x =

2.
9 55°
Q 6.4 cm R
∴ x = 1.6
l
∴ AB = 2x = 2 × 1.6 = 3.2 cm;
∴ BC = 3x = 3 × 1.6 = 4.8 cm. Explanation :

∴ AC = 4x = 4 × 1.6 = 6.4 cm Line l is perpendicular bisector of seg TR

Analytical Figure ∴ PT = PR ...(i)


(Perpendicular bisector theorem)
A QT = 2.4 cm ...(ii)

A PQ = PT + QT ...( P-T-Q)
6.
3.2 cm

∴ PQ = PR + 2.4 ...[From (i) and (ii)]


cm

∴ PQ – PR = 2.4 cm
B C Steps of construction :
6.4 4.8 cm
cm (1) Draw a seg QR of length 6.4 cm.
(2) Draw an angle measuring 55° at vertex
Q, on this ray, locate a point T such that
3.2 cm

l(QT) = 2.4 cm. Draw seg RT.


(3) Draw line l as ⊥ bisector of seg RT intersecting
ray QT at point P.
B 4.8 cm C (4) Draw seg PR.

Steps of construction :
(1) Draw seg BC of length 4.8 cm
(2) Taking B as a centre and arc length 3.2 draw an
arc.
61 Master Key Geometry (Std. IX)

ASSIGNMENT - 4
Time : 1 Hour Marks : 20

Q.1. Solve : (6)

(1) Draw a line l and take any point P on it. Draw line m perpendicular to line l at point P.
(2) Draw seg AB of length 7 cm. Take point P such that AP = 3 cm and A-P-B. Draw a perpendicular
to seg AB through the point P.
(3) Draw a circle of radius 3 cm and centre O. Take any point P on the circle, draw perpendicular to
seg OP at point P.

Q.2. Solve : (6)

(1) Construct ∆PQR such that PQ – PR = 2.4 cm, QR = 6.4 cm and ∠PQR = 55°
(2) Construct ∆ABC, such that BC = 6 cm, ∠ABC = 100° and AC – AB = 2.5 cm.

Q.3. Solve : (8)

(1) Perimeter of ∆ABC is 15 cm, BC = 5.2 cm, ∠ACB = 45°. Construct ∆ABC.
(2) Construct �ABC whose perimeter is 12 cm, ∠B = 60°, ∠C = 70°.

vvv
5 Quadrilaterals

∠BCA ≅ ∠DAC ....[From (ii)]


Points to Remember: ∴ ∆ABC ≅ ∆CDA
...(ASA Test of congruency)
 Parallelogram :
side AB ≅ side CD
• Definition : A quadrilateral is called a ...(c.s.c.t.)
parallelogram if its D C side BC ≅ side DA
opposite sides are ∠ABC ≅ ∠CDA ...(c.a.c.t.)
parallel. Similarly, we can prove
A B
In ABCD, ∠DAB ≅ ∠DCB
side AB side DC
and side AD side BC. Theorem - 2 :
R
∴ ABCD is a parallelogram. "Diagonals of a S
M
• Properties of Parallelogram : parallelogram bisect
each other."
(i) Opposite sides are congruent P Q
Given:
(ii) Opposite angles are congruent
(i) PQRS is a parallelogram.
(iii) Adjacent angles are supplementary.
(ii) Diagonals PR and QS intersect each
(iv) Diagonals bisect each other.
other at point M.
Theorem - 1 : To Prove: (i) seg PM ≅ seg RM
(ii) seg QM ≅ seg SM
Opposite sides and opposite angles of a
parallelogram are congruent. Proof : PQRS is a parallelogram ...(Given)
Given : ABCD is a parallelogram. side PQ side RS
...(Opposite sides of a parallelogram)
To Prove : (i) side AB ≅ side CD
on transversal PR,
(ii) side BC ≅ side AD
∠RPQ ≅ ∠PRS
(iii) ∠ABC ≅ ∠CDA
...(Alternate angles theorem)
(iv) ∠DAB ≅ ∠BCD.
D C ∴ ∠MPQ ≅ ∠MRS ...(i) (P - M - R)
on transversal QS,
∠SQP ≅ ∠QSR
A B
...(Alternate angle theorem)
Construction : Draw diagonal AC.
∴ ∠MQP ≅ ∠MSR ...(ii) (Q - M - S)
Proof : ABCD is a parallelogram.
In ∆PMQ and ∆RMS,
∴ side AB side CD
∠MPQ ≅ ∠MRS ...[From (i)]
and side BC side AD
side PQ ≅ side RS ...(Opposite sides of
on transversal AC,
a parallelogram are congruent)
∠BAC ≅ ∠DCA ...(i) (Alternate
∠MQP ≅ ∠MSR ...[From (ii)]
∠BCA ≅ ∠DAC ...(ii) angles theorem)
∴ ∆PMQ and ∆RMS
In ∆ABC and ∆CDA, ...(ASA Test of congruency)
∠BAC ≅ ∠DCA ...[From (i)] ∴ seg PM ≅ seg RM ...(c.s.c.t.)
side AC ≅ side CA ...(Common side) seg QM ≅ seg SM
(62)
Quadrilaterals 63

∠A = (3x + 12) ∠B = (2x – 32)


MASTER KEY QUESTION SET - 5
= 3 (40) + 12 = 2 (40) – 32
PRACTICE SET - 5.1 (Textbook Page No. 62) = 120 + 12 = 80 – 32
∴ ∠A = 132° ∴ ∠B = 48°
(1) Diagonals of a parallelogram WXYZ intersect
each other at point O. If ∠XYZ = 135° then what ∴ ∠C = ∠A and ∠D = ∠B ...(Opposite angles
is the measure of ∠XWZ and ∠YZW? of a parallelogram are congruent.)
If l(OY) = 5 cm then find l(WY) = ? ∴ ∠C = 132° and ∠D = 48°
Solution: W Z
(3) Perimeter of a parallelogram is 150 cm. One
O of its sides is greater than the other side by
25 cm. Find lengths of all sides.
X Y
Solution: D C
WXYZ is a parallelogram ...(Given)
∠XYZ ≅ ∠XWZ ...(Opposite angles of a A B
parallelogram are congruent) ABCD is a parallelogram.
But ∠XYZ = 135° Perimeter of ABCD = 150 cm.
∴ ∠XWZ = 135° Let BC = x cm.
∴ ∠YZW + ∠XYZ = 180° ∴ AB = (x + 25) cm
...(Adjacent angles of a parallelogram
AB = CD and BC = AD ...(Opposite sides
are supplementary)
of parallelogram are congruent.)
∴ ∠YZW + 135 = 180
∴ CD = x + 25 cm and AD = x cm
∴ ∠YZW = 180 – 135
Perimeter of ABCD = AB + BC +CD +AD
∴ ∠YZW = 45°
∴ 150 = x + 25 + x + x + 25 + x.
l (OY) = ½ l (WY) ...(Diagonals of
∴ 150 – 50 = 4x
parallelogram bisect each other.)
∴ 100 = 4x
∴ 5 = ½ l (WY)
∴ 100 = x
∴ l(WY) = 10 cm
4
(2) In a parallelogram ABCD, if ∠A = (3x + 12)°; ∴ x = 25
∠B = (2x – 32)° then find the value of x and then AB = CD = x + 25 = 25 + 25 = 50 cm
find the measures of ∠C and ∠D.
BC = AD = x = 25 cm
Solution: A (3x + 12)° D
(4) If the ratio of measures of two adjacent
angles of a parallelogram is 1 : 2, find the
(2x – 32)°
B C measures of all angles of the parallelogram.
ABCD is a parallelogram ...(Given)
Solution: S R
∴ ∠A + ∠B = 180° ...(Adjacent angles
PQRS is a
of a parallelogram are supplementary)
parallelogram.
∴ (3x + 12) + (2x – 32) = 180 P Q
∠P : ∠Q = 1 : 2
∴ 5x – 20 = 180
Let the common multiple be x.
∴ 5x = 180 + 20
∴ ∠P = x and ∠Q = 2x
∴ 5x = 200
∴ ∠P + ∠Q = 180° ...(Adjacent angles
∴ x = 40 of a parallelogram are supplementary)
∴ x + 2x = 180
∴ 3x = 180
64 Master Key Mathematics - II (Geometry) (Std. IX)

180 ∴ ∠B = 110°
∴ x = = 60
3
∴ ∠RAB + ∠R = 180° ...(Adjacent angles
∠P = x = 60° of a parallelogram are supplementary)

∠Q = 2x = 2 × 60 = 120° ∴ ∠RAB + 110 = 180
∴ ∠R = ∠P and ∠S = ∠Q ...(Opposite angles ∴ ∠RAB = 180 – 110
of a parallelogram are congruent)
∴ ∠RAB = 70°
∴ ∠R = 60° and ∠S = 120° ∴ ∠BCR = ∠RAB ...(Opposite angles of a
(5) Diagonals of a parallelogram intersect each parallelogram are congruent)
other at point O. If AO = 5, BO = 12 and ∴ ∠BCR = 70°
AB = 13 then show that ABCD is a rhombus.
D C (7) In following figure, ABCD is a
O parallelogram. Point E is on the ray AB such
To Prove: ABCD is a that BE = AB then prove that line ED bisects
rhombus. A B seg BC at point F.
2 2 2 2 A B E
Proof : AO + BO = 5 + 12 = 25 + 144
∴ AO2 + BO2 = 169 ...(i)
2
AB = 13 = 169 2
...(ii) F

In ∆AOB, AB2 = AO2 + BO2 D C


...[From (i) and (ii)] Proof : ABCD is a parallelogram ...(Given)
∴ ∠AOB = 90° ...(Converse of Pythagoras AB = CD ...(i) (Opposite side of a
theorem) parallelogram are congruent.)
∴ Diagonals of ABCD are perpendicular AB = BE ...(ii) (Given)
to each other at point O. ∴ CD = BE ...(iii) [From (i) and (ii)]
Also, ABCD is a parallelogram ...(Given) side AB side CD
But, we know that a parallelogram in ...(Definition of a parallelogram)
which diagonals are perpendicular is a ∴ side AE side CD ...(A - B - E)
rhombus On transversal BC,
∴ ABCD is a rhombus. ∴ ∠DCF ≅ ∠EBF ...(iv)
(6) In following diagrams, PQRS and ABCR (Alternate angles theorem)
are two parallelograms. If ∠P = 110° then find In ∆DCF and ∆EBF
measures of all angles of ABCR. side CD ≅ side BE ...[From (iii)]
P
110° Q ∠DFC ≅ ∠EFB
B C ...(Vertically opposite angles)
∠DCF ≅ ∠EBF ...[From (iv)]
S A R
Solution: ∴ ∆DCF ≅ ∆EBF ...(SAA Test of congruency)
PQRS is a parallelogram ...(Given) ∴ seg CF ≅ seg BF ...(c.s.c.t)
∴ ∠R = ∠P ...(Opposite angles of a ∴ line ED bisects seg BC at point F.
parallelogram are congruent)
But, ∠P = 110° ...(Given) PROBLEMS FOR PRACTICE
∴ ∠R = 110°
(1) Prove that two opposite vertices of a
ABCR is a parallelogram ...(Given) parallelogram are equidistant from the
∴ ∠B = ∠R ...(Opposite angles of a diagonal not containing these vertices.
parallelogram are congruent)
Quadrilaterals 65

(2) In the adjoining figure, LAXM is a


∴ ∠ABD ≅ ∠CDB ...(c.a.c.t.)
parallelogram. Point I is the midpoint of
diagonal LX. PQ is a line passing through point ∴ Side AB Side CD ...(i)
I. The points P and Q P (By Alternate angles test)
a r e th e po in ts o f L A Similarly, we can prove that,
intersection of sides ∴ side AD side BC ...(ii)
AL and MX I
In ABCD,
respectively. Prove
M X
side AB side CD ...[From (i)]
that Q
seg PI ≅ seg IQ. side AD side BC ...[From (ii)]

(3) Prove that in a parallelogram, the angle ∴ ABCD is a parallelogram.


bisectors of two adjacent angles meet at right ...(By definition)
angles.
(4) In a parallelogram ABCD, diagonal Theorem - 4 :
AC ⊥ diagonal BD intersect at point M. Statement : If both the pairs of opposite angles
AM = 3 cm, BM = 4 cm, AB = 7.5 cm and of a quadrilateral are congruent then it is a
BC = 5 cm. Find the perimeter of ABCD and parallelogram. P Q
also the lengths of AC and BD. Given: In PQRS,
ANSWERS ∠SPQ ≅ ∠SRQ

(4) Perimeter of ABCD = 25 cm, AC = 6 cm, ∠PSR ≅ ∠PQR


S R
BD = 8 cm.
To Prove: PQRS is a parallelogram.
Proof : Let,
Points to Remember: ∠SPQ = ∠SRQ = x° ...(i) (Given)
 Test for Parallelogram : ∠PSR = ∠PQR = y° ...(ii)
• Following theorems will help us to prove a In PQRS,
quadrilateral is a parallelogram. ∠SPQ + ∠PQR + ∠QRS
+ ∠PSR = 360° ...(Angle sum property
Theorem - 3 : of a quadrilateral)
∴ x + y + x + y = 360
Statement : If pairs of opposite sides of a
quadrilateral are congruent then that quadrilateral ∴ 2x + 2y = 360
is a parallelogram. ∴ 2 (x + y) = 360
A B ∴ x + y = 180
Given : In ABCD,
(i) side AB ≅ side CD ∴ m∠SPQ + m∠PSR = 180°
(ii) side AD ≅ side BC ...[From (i) and (ii)]
D C
To Prove : ABCD is a parallelogram. ∴ side PQ side SR ...(iii)
(By Interior angles test)
Construction : Draw diagonal BD.
Similarly, we can prove,
Proof : In ∆ABD and ∆CDB,
∴ side PS side QR ...(iv)
side AB ≅ side CD ...(Given)
In PQRS,
side AD ≅ side BC ...(Given)
side PQ side SR ...[From (iii)]
side BD ≅ side BD ...(Common side)
side PS side QR ...[From (iv)]
∴ ∆ABD ≅ ∆CDB
...(By SSS test of congruency) ∴ PQRS is a parallelogram.
...(By definition)
66 Master Key Mathematics - II (Geometry) (Std. IX)

Theorem - 5 : ∠LMK ≅ ∠NKM ...(i)


...(Alternate angles theorem)
Statement : If the diagonals of a quadrilateral bisect
each other, then the quadrilateral is a parallelogram. ∴ In ∆KLM and ∆MNK
Given: In PQRS, P Q seg LM ≅ seg KN ...(Given)
(i) Diagonal M ∠LMK ≅ ∠NKM ...[From (i)]
PR and seg KM ≅ seg KM ...(Common side)
diagonal ∴ ∆KLM ≅ ∆MNK
QS S R ...(By SAS test of congruency)
intersect at M. ∴ ∠LKM ≅ ∠NMK ...(c.a.c.t.)
(ii) seg PM ≅ seg RM ∴ seg LK seg MN ...(ii)
(iii) seg SM ≅ seg QM (By Alternate angles test)
To Prove: PQRS is a parallelogram. In LMNK,
Proof : In ∆PMQ and ∆RMS, side LM side KN ...(Given)
seg PM ≅ seg RM ...(i) (Given) side LK side MN ...[From (ii)]
∠PMQ ≅ ∠RMS ∴ LMNK is a parallelogram.
...(Vertically opposite angles) ...(By definition)
seg QM ≅ seg SM ...(Given)
∴ ∆PMQ ≅ ∆RMS, ...(By SAS test of PRACTICE SET - 5.2 (Textbook Page No. 67)
congruency)
∴ ∠PQM ≅ ∠RSM ...(c.a.c.t.)
(1) A D In adjoining figure,
ABCD is a
∴ ∠PQS ≅ ∠RSQ ...(Q-M-S) P Q parallelogram,
∴ side PQ side SR ...(i) P and Q are
(By Alternate angles test) B C midpoints of side AB
Similarly, we can prove and side CD
side PS side QR ...(ii) respectively.
In PQRS, Prove that: APCQ is a parallelogram.
side PQ side SR ...[From (i)] Proof: side AB side CD ...(Opposite sides of a
parallelogram are parallel)
side PS side QR ...[From (ii)]
∴ side AP side CQ ... (i) (A-P-B,C-Q-D)
∴ PQRS is a parallelogram.
...(By definition) AB = CD ...(Opposite sides of
parallelogram are congruent)
Theorem - 6 : 1 1
∴ AB = CD
Statement : A quadrilateral is a parallelogram if 2 2
1
a pair of opposite sides is parallel and congruent. ...(Multiplying through out by )
2

Given: In LMNK, ∴ AP = CQ ...(ii) [ P and Q are midpoints
L M
(i) side LM side KN of sides AB and CD respectively]
(ii) side LM ≅ side KN In APCQ,
To Prove: LMNK is a side AP side CQ [From (i)]
K N
parallelogram. side AP ≅ side CQ [From (ii)]
Construction: Draw diagonal MK. ∴ APCQ is a parallelogram
Proof : In LMNK, ...(A quadrilateral is a parallelogram,
side LM side KN ...(Given) if a pair of opposite sides is
On transversal MK, parallel and congruent)
Quadrilaterals 67

(2) Using opposite angles test for parallelogram, GP = PH ...(vi) [From (ii), (v)]
prove that every rectangle is a parallelogram. In GEHF, seg EP ≅ seg FP ...[From (i)]
Given: ABCD is a rectangle. seg GP ≅ seg PH ...[From (vi)]
ABCD is a D C
To Prove: ∴ GEHF is parallelogram.
parallelogram. ...(If diagonals of a quadrilateral bisect
each other then it is a parallelogram.
Proof: A B
(4) Prove that quadrilateral formed by the
ABCD is a rectangle ...(Given) intersection of angle bisectors of all angles of
∴ ∠A = ∠B = ∠C = ∠D = 90° ...(i) a parallelogram is a rectangle.
(Angles of a rectangle) Given: ABCD is a parallelogram.
In ABCD, rays AR, BR, CP, DP are bisectors of ∠A,
∠A ≅ ∠C ...[From (i)] ∠B, ∠C, ∠D respectively.
∠B ≅ ∠D ...[From (i)] P-S-D, P-Q-C, R-Q-B, R-S-A.
∴ ABCD is a parallelogram  B
A P
...(If both the pairs of opposite angles of
a quadrilateral are congruent then it is a Q
parallelogram) S

 ∆
(3) In adjoining figure, D
D
 R ∆

G is the point of C
To Prove: PQRS is a rectangle.
concurrence of
Proof: Let,
medians of ∆DEF. G
Rays AR,
Ta k e p o i n t H o n E P ∠SAD = ∠SAB = a°
F BR, CP, DP
ray DG such that ∠QBA = ∠QBC = b°
bisects ∠A,
H
D-G-H and DG = GH, ∠QCB = ∠QCD = c° ∠B, ∠C, ∠D
then prove that ∠SDC = ∠SDA = b° respectively
GEHF is a parallelogram.
In ∆ASD
Proof: Let the point of intersection of seg EF and
seg GH be P. ∠SAD + ∠SDA + ∠ASD = 180°
G is the centroid of ∆DEF ...(Sum of the measures
of all angles of a triangle is 180°)
∴ seg DP is the median
∴ a + d + ∠ASD = 180
∴ P is the midpoint of seg EF.
∴ ∠ASD = 180° – (a + d) ...(i)
∴ EP = FP ...(i)
centroid ABCD is a parallelogram ...(Given)
1
Now, GP = PD ...(ii) divides the
3 ∴ ∠A + ∠D = 180° ...(Adjacent angles
2 median in the
DG = PD ...(iii) of a parallelogram are supplementary)
3 ratio 2 : 1
∴ ∠SAD + ∠SAB + ∠SDA + ∠SDC = 180°
But, DG = GH ...(iv) ...(Given) ...(Angle addition property)
2
∴ GH = PD ...[From (iii) and (iv)] ∴ a+a+d+d = 180
3
2 ∴ 2a + 2d = 180
∴ GP + PH = PD
3 ...(G-P-H) ∴ 2 (a + d) = 180
1 2 180
∴ PD + PH = PD ...[From (ii)] a+d =
3 3 ∴ 2
2 1
∴ PH = PD – PD ∴ a+d = 90 ...(ii)
3 3
1 Now, ∠ASD = 180 – 90 [From (i), (ii)]
∴ PH = PD ...(v)
3
68 Master Key Mathematics - II (Geometry) (Std. IX)

∴ ∠ASD = 90° ...(iii)


PROBLEMS FOR PRACTICE
Now, ∠ASD ≅ ∠PSR
...(Vertically opposite angles) (1) In ∆ABC, seg AB ≅ seg AC. D
∴ ∠PSR = 90° [From (iii)] Ray AF bisects ∠DAC. A °
° F
Similarily, we can prove ∠SPQ = 90°, Ray CF ray BA.
∠PQR = 90°, ∠SRQ = 90° Prove that ABCF is a
In PQRS, parallelogram B C
∠P = ∠Q = ∠R = ∠S = 90° (2) If diagonal of a parallelogram bisects one of
the angles of the parallelogram, it also bisects
∴ PQRS is a rectangle ...(By definition)
the second angle.
(5) In adjoining A P (3) In the adjoining figure,
B D C
figure, If points ABCD is a parallelogram
Q P
P, Q, R, S are on ∠DAB = 60°. If the bisector
t h e si d es o f S AP and BP of ∠A and ∠B

parallelogram C respectively meet at point 
such that D R P on side CD, prove that P A B
AP = BQ = CR = DS then prove that PQRS is is midpoint of side CD.
a parallelogram.
To Prove: PQRS is a parallelogram
Proof: ABCD is a parallelogram. ...(Given) Points to Remember:
∴ AD = BC ...(Opposite sides of
 Rectangle :
parallelogram are equal.) A B
A parallelogram
∴ AS + DS = BQ + QC ...(A-S-D, B-Q-C)
∴ in which each
∴ AS = CQ ...(i) ( DS = BQ) angle is a right
In ∆PAS and ∆RCQ angle is called a
C D
side AP ≅ side CR ...(Given) rectangle.
∠A ≅ ∠C ...(Opposite angles of a In parallelogram ABCD,
parallelogram are congruent) ∠A = ∠B = ∠C = ∠D = 90°.
side AS ≅ side CQ [From (i)]
Properties of a Rectangle :
∴ ∆PAS ≅ ∆RCQ
(i) Diagonals bisect each other
...(By SAS test of congruency)
(ii) Diagonals are congruent.
∴ side PS ≅ side RQ ...(ii) (c.s.c.t.)
Similarly, we can prove, NOTE
side PQ ≅ side RS ...(iii) Every rectangle is a parallelogram
In PQRS,
Theorem - 7 :
side PQ ≅ side RS ...[From (iii)]
Statement : Diagonals of a rectangle are congruent.
side PS ≅ side RQ ...[From (ii)]
Given : ABCD is a rectangle.
∴ PQRS is a parallelogram.
...(A quadrilateral is a parallelogram if seg BD and seg AC are its diagonals.
its opposite sides are congruent.) To Prove: Diagonal AC ≅ diagonal BD
A D

B C
Quadrilaterals 69

Proof : ABCD is a rectangle. ...(Given) Proof : PQRS is a rhombus. ...(Given)


∴ ∠ABC = ∠DCB = 90° ...(i) ∴ It is a parallelogram

(Angles of a rectangle) ...( Every rhombus is a parallelogram)
In ∆ABC and ∆DCB, ∴ seg PM ≅ seg RM ...(i)
side AB ≅ side DC seg SM ≅ seg QM ..(ii)
...(Opposite sides of rectangle) ...(Diagonals of a parallelogram
∠ABC ≅ ∠DCB ...[From (i)] bisect each other)
side BC ≅ side BC ...(Common side) In ∆PMS and ∆PMQ
seg SM ≅ seg QM [From (ii)]
∴ ∆ABC ≅ DCB
...(By SAS test of congruency) side PS ≅ side PQ ...(sides of a rhombus)
∴ Diagonal AC ≅ diagonal BD ...(c.s.c.t.) seg PM ≅ seg PM ...(Common side)
∴ ∆PMS ≅ ∆PMQ
Theorem - 8 : ...(By SSS test of congruency)
Statement : Diagonals of a square are congruent. ∴ ∠PMS ≅ ∠PMQ ...(iii) (c.a.c.t.)
Hint : Using the concept of theorem 7, prove this ∠PMS + ∠PMQ = 180°
theorem. ...(Angles forming linear pair)
• Rhombus : A D ∴ ∠PMS + ∠PMS = 180 ...[From (iii)]
A parallelogram ∴ 2 ∠PMS = 180
having all sides ∴ ∠PMS = 90°
congruent is called a B C ∴ Diagonal PR ⊥ diagonal QS
rhombus.
• Square :
In ABCD,
A quadrilateral is called a square if all its sides
seg AB ≅ seg BC ≅ seg CD ≅ seg AD are congruent and each angle is a right angle
∴ ABCD is a rhombus. In ABCD,
Properties of a Rhombus : seg AB ≅ seg BC ≅ seg CD ≅ seg AD.
(i) Opposite angles are congruent. ∠A = ∠B = ∠C = ∠D = 90° A B
(ii) Diagonals bisect each other at right ∴ ABCD is a square.
angles.
Properties of a square :
NOTE
(i) Diagonals are congruent. D C
Every rhombus is a parallelogram.
(ii) Diagonals bisect each other at right
Theorem - 9 : angles.
Statement : Diagonals of a rhombus are NOTE
perpendicular bisectors of each other. (i) A parallelogram whose adjacent sides are
Given : PQRS is a rhombus. congruent and one angle is a right angle, is
Diagonal PR and diagonal QS intersect at the a square.
point M. (ii) A rectangle with adjacent sides congruent is
To Prove: (i) seg PM ≅ seg RM a square.
P (iii)If in a rhombus, one of its angle is a right angle
(ii) seg SM ≅ seg QM then it is a square. Every square is a rhombus.
S Q (iii) diagonal PR ⊥ diagonal QS
M Theorem - 10 :
Statement : Diagonals of a square are perpendicular
R
bisectors of each other.
70 Master Key Mathematics - II (Geometry) (Std. IX)

Given : ABCD is a square. PRACTICE SET - 5.3 (Textbook Page No. 69)
Diagonal AC and diagonal BD intersect at M.
(1) Diagonals of a rectangle ABCD intersect at
To Prove: (i) seg AM ≅ seg CM
point O. If AC = 8 cm then find BO and if
(ii) seg BM ≅ seg DM ∠CAD = 35° then find ∠ACB.
(iii) Diagonal AC ⊥ diagonal BD A 35° D
Solution:
A D O
M
B C

B C ABCD is a rectangle. ...(Given)

Proof : ABCD is a square ...(Given) ∴ BD = AC ...(Diagonals of a rectangle


are congruent)
∴ ABCD is a rhombus.
∴ But, AC = 8 cm ...(Given)
...( Every square is a rhombus)
∴ BD = 8 cm ...(i)
∴ seg AM ≅ seg CM
1 ...(Diagonals of rectangle
seg BM ≅ seg DM BO =
2
BD
bisect each other)
∴ Diagonal AC ⊥ diagonal BD 1
BO = × 8
...(Diagonals of a rhombus are ∴ 2 ...[From (i)]
perpendicular bisectors of each other) ∴ BO = 4 cm
side AD side BC...(Opposite sides of a
Theorem - 11 : rectangle are parallel)
On transversal AC,
Statement : Diagonals of a rhombus bisect its
opposite angles. ∴ ∠ACB ≅ ∠CAD
D C ...(Alternate angles theorem)
Given : ABCD is a
rhombus. But, ∠CAD = 35° ...(Given)
seg AC is a diagonal. ∴ ∠ACB = 35°
A B
(2) In a rhombus PQRS if PQ = 7.5 then find QR.
To Prove: (i) ∠BAC ≅ ∠DAC
If ∠QPS = 75° then find the measure ∠PQR
(ii) ∠BCA ≅ ∠DCA and ∠SRQ. S R
Proof :
Solution:
In ∆ABC and ∆ADC
side AB ≅ side AD ... (sides of a P
75°
Q
side BC ≅ side DC rhombus)
side AC ≅ side AC ...(Common side) PQRS is a rhombus. ...(Given)

∴ ∆ABC ≅ ∆ADC...(SSS Test of congruency) ∴ QR = PQ ...(Sides of a rhombus are equal)

∴ ∠BAC ≅ ∠DAC But, PQ = 7.5 ...(Given)

and ∠BCA ≅ ∠DCA ...(c.a.c.t.) ∴ QR = 7.5


∴ ∠SRQ ≅ ∠QPS ...(Opposite angles of
Theorem - 12 : a rhombus are congruent)
Statement : Diagonals of a square bisects its But, ∠QPS = 75° ...(Given)
opposite angles. ∠SRQ = 75°

Hint : Use the concept used in the above theorem PQRS is a parallelogram
to prove the above statement. ...(Every rhombus is a parallelogram)
Quadrilaterals 71

∴ ∠PQR + ∠SPQ = 180° In �AMB, ∠AMB = 90°


...(Adjacent angles of a parallelogram ...(Diagonals of a rhombus are
are supplementary) perpendicular to each other)
∴ ∠PQR + 75 = 180
∴ AB2 = AM2 + BM2 ...(Pythagoras theorem)
∴ ∠PQR = 180 – 75
= 102 + 10.52 ...[From (i) and (ii)]
∴ ∠PQR = 105°
∴ = 100 + 110.25
(3) Diagonals of square IJKL intersect at point M. ∴ AB2 = 210.25 =
21025 5 × 5 × 29 × 29
=
Find the measures of ∠IMJ, ∠JIK and ∠LJK 100 10 × 10
L K 5 × 29 145
Solution: ∴ AB =
10
=
10
M
...(Taking square roots)
∴ AB = 14.5 cm
I J
IJKL is a square. ...(Given) ∴ Each side of rhombus ABCD is 14.5 cm

Diagonals of a square are perpendicular ∴ Perimeter of rhombus ABCD = 4 × 14.5


to each other. ∴ Perimeter of rhombus ABCD = 58 cm
∴ ∠IMJ = 90°
(5) State with reasons whether following
Diagonals of a square bisects the opposite statements are 'True' or 'False'.
angles
(i) Every parallelogram is a rhombus.
1
∴ ∠JIK = ∠JIL Ans. False. All sides of rhombus are congruent
2
whereas in parallelogram opposite sides are
1
∴ ∠JIK = × 90° = 45° congruent.
2
(ii) Every rhombus is a rectangle.
... (Angle of a square)
Ans. False : Each angle of a rectangle is right angle,
1
Also, ∠LJK = ∠IJK whereas in rhombus opposite angles are
2
congruent, they need not be right angles always.
1
∴ ∠LJK = × 90° = 45° (iii) Each rectangle is a parallelogram.
2
... (Angle of a square) Ans. True : In rectangle, both pairs of opposite sides
are parallel. This is true for parallelograms also.
(4) Diagonals of a rhombus are 20 cm and 21 cm (iv) Each square is a rectangle.
respectively, then find the side of rhombus and
Ans. True : Each angle of square is a right angle, which
its perimeter. D C is true for rectangle also.
Solution: (v) Each square is a rhombus.
M
ABCD is a Ans. True : All sides of a square are congruent which
rhombus A B is true for rhombus also.
Diagonals AC (vi) Every parallelogram is a rectangle.
and BD intersect at point M. Ans. False : Each angle of rectangle is a right angle,
AC = 20 cm; BD = 21 cm which is not true for a parallelogram.
Diagonals AC and BD are bisectors of
each other
PROBLEMS FOR PRACTICE
1 1
∴ AM = × AC = × 20 = 10 cm ...(i)
2 2 (1) Prove that diagonals of a square divide it into
1 1 four congruent triangles.
BM = × BD = × 21 = 10.5 cm ...(ii)
2 2 (2) PQRS is a rhombus. O be any point on
diagonal QS. Prove that seg OP ≅ seg OR.
72 Master Key Mathematics - II (Geometry) (Std. IX)

(3) TAMS is a square.


PRACTICE SET - 5.4 (Textbook Page No. 71)
T A
seg AW ≅ seg AN.
W (1) In IJKL, side IJ side KL, ∠I = 108° ∠K = 53°
Prove that
then find the measures of ∠J and ∠L.
seg SW ≅ seg SN. I J
Solution: 108° ?°

S N M

Points to Remember: L ?° 53° K


In IJKL, side IJ side KL ...(Given)
 Trapezium : On transversal JK,
A quadrilaterial is said to be a trapezium, if ∴ ∠K + ∠J = 180°
only one pair of opposite sides is parallel. ...(Interior angles theorem)
A B
In ABCD, ∴ 53 + ∠J = 180
side AB side DC ∴ ∠J = 180 – 53
∴ ABCD is a ∴ ∠J = 127°
trapezium. D C On transversal IL,
 Properties of a Trapezium. ∴ ∠I + ∠L = 180°
In a trapezium line segment joining the ...(Interior angles theorem)
midpoints of non-parallel sides is : ∴ 108 +∠L = 180
(i) Parallel to its parallel sides and ∴ ∠L = 180 – 108
(ii) half the sum of the length of its parallel sides. ∴ ∠L = 72°
In trapezium PQRS, sides PS side QR point
(2) In ABCD, side BC side AD,
A and B are midpoints of seg PQ and seg SR
side AB ≅ side DC. If ∠A = 72° then find the
respectively, then
measures of ∠B, and ∠D.
(a) seg AB seg QR seg PS
1 Construction : Draw seg CE side AB such that
(b) AB = × (PS + QR) A-E-D. B C
2 P S
Solution:
A B
72°
A E D
Q R
side AB side CE ...(i) (Construction)
 Isosceles Trapezium : side BC side AD ...(Given)
A B
A trapezium in ∴ side BC side AE ...(ii) (A-E-D)
which non AECB is a parallelogram
parallel sides ...[From (i), (ii) and by definition)
are congruent is D C side AB ≅ side CE ...(iii)
called an isosceles trapezium. (Opposite sides of
In ABCD, a parallelogram are congruent)
side AB side DC Also, side AB ≅ side CD ...(iv) (Given)
side AD ≅ side BC In ∆CED, side CE ≅ side CD
...[From (iii), (iv)]
∴ ABCD is an Isosceles trapezium.
∴ ∠CED ≅ ∠CDE ...(v)
NOTE
side CE side AB
The base angles of an isosceles trapezium are
On transversal AE,
congruent.
Quadrilaterals 73

∠CED ≅ ∠BAD ...(vi) ∴ ∠DCE = ∠ABC


∴ ∠CDE ≅ ∠BAD ...(vii) ...[From (vi) (vii)]
(Corresponding angles theorem) ∴ ∠ABC ≅ ∠DCB ...(E - B - C)
... [From (v) (vi)]
∠BAD = 72° ...(Given) PROBLEMS FOR PRACTICE
∠CDE = 72° ...[From (vii)]
(1) PQRS is a isosceles S R
∴ ∠D = 72°
trapezium. PQ RS
side BC side AD and ∆RQE is an
On transversal AB, equilateral triangle.
∠A + ∠B = 180° Find measures of all P E Q

...(Interior angles theorem) angles of the trapezium.


ɺ ġġġġġġġġ72 + ∠B = 180 (2) The measures of the angles of a quadrilateal
ɺ ∠B = 180 – 72 taken in order are as 1 : 2 : 3 : 4. Prove that it is
a trapezium.
∴ ∠B = 108°
(3) ABCD is a trapezium in which side AB side
(3) In ABCD , side BC < side AD, side BC side CD and side AD ≅ side BC. Prove that diagonal
AD and if side BA ≅ side CD. AC ≅ diagonal BD.
Then prove that ∠ABC ≅ ∠DCB. ANSWERS
Construction : Draw seg AE side DC and
extend side BC such that E-B-C (1) 60°, 60°, 120°, 120°
B C
E
Proof :
Points to Remember:
D
A Theorem - 13 :
side AE side DC ...(i) (Construction)
 Theorem of midpoint of two sides of a
side BC side AD ...(Given)
triangle. (Midpoint theorem)
∴ side EC side AD ...(ii) (E-B-C)
Statement : The line A
∴ AECD is a parallelogram
segment joining the
...[From (i), (ii) and by definition] Q R
midpoints of any two P
∴ side AE ≅ side DC ...(iii) (Opposite sides
sides of a trianlge is
of a parallelogram are congruent)
parallel to the third side
side AB ≅ side DC ...(iv) (Given) B C
and is half of it.
In ∆ABE, side AB ≅ side AE
Given : In ∆ABC, P and Q are the midpoints of
...[From (iii), (iv)]
sides AB and AC respectively.
∠AEB ≅ ∠ABE
To prove : (i) seg PQ seg BC
...(Isosceles triangle theorem) 1
(ii) PQ = BC
∴ ∠AEC ≅ ∠ABE ...(v) (E - B - C) 2
∠AEC + ∠DCE = 180° ...(vi) Construction : Take a point R on ray PQ such that
...(Adjacent angles of a parallelogram P - Q - R and seg PQ ≅ seg QR. Draw
are supplementary) seg CR.
Also, ∠ABE + ∠ABC = 180° ...(vii) Proof : In ∆AQP and ∆CQR,
(Linear pair angles) seg AQ ≅ seg CQ

∠AEC + ∠DCE = ∠ABE + ∠ABC ( Q is midpoint of side AC)
...[From (vi), (vii)] ∠AQP ≅ ∠CQR
...(Vertically opposite angles)
74 Master Key Mathematics - II (Geometry) (Std. IX)

seg PQ ≅ seg QR ...(Construction) ∴ PQRB is a parallelogram


∴ ∆AQP ≅ ∆CQR ...[From (i), (ii) and by definition]
...(By SSS test of congruency) ∴ side PB ≅ side QR ...(iii)
∴ seg AP ≅ seg CR ...(i) (c.s.c.t.) (Opposite sides of a parallelogram)
Also, ∠PAQ ≅ ∠RCQ ...(c.a.c.t.) But, seg PB ≅ seg AP ...(iv)

( P is midpoint of seg AB)
∴ ∠BAC ≅ ∠RCA
∴ ∴ seg QR ≅ seg AP ...(v)
...( B - P - A and A - Q - C)
...[From (iii) and (iv)]
∴ seg BA seg CR
...(By Alternate angles test) ∴ In ∆APQ and ∆QRC,
∴ seg AP ≅ seg QR ...[From (v)]
∴ seg BP seg CR ...(ii) ( A - P - B)
seg AP ≅ seg BP ...(iii) ∠AQP ≅ ∠QCR ...(Corresponding

( P is midpoint of side AB) ∠PAQ ≅ ∠RQC angles theorem)
∴ seg BP ≅ seg CR ...(iv)[From (i), (iii)] ∴ ∆APQ ≅ ∆QRC
PBCR is a parallelogrem ...[From (ii), ...(By SAA test of congruency)
(iv) and a quadrilateral is a parallelogram ∴ seg AQ ≅ seg QC ...(c.s.c.t)
if a pair of opposite sides is parallel and
congruent.]
PRACTICE SET - 5.5 (Textbook Page No. 73)
∴ seg PR seg BC ...(By definition)

∴ seg PQ seg BC ...( P - Q - R) (1) In adjoning figure,
A
X , Y, Z a r e t h e
1
PQ =
2
PR ...(v) (Construction) midpoints of side AB, Z
X
But, PR = BC ...(vi) side BC and side AC
of ∆ABC respectively. B Y C
(Opposite sides of a parallelogram)
AB = 5 cm, AC = 9 cm
∴ PQ =
1
BC and BC = 11 cm. Find
...[From (v) and (vi)]
2 the length of XY, YZ, XZ.

Theorem - 14 : Solution:
A In ∆ABC,
 Converse of Midpoint theorem
Statement : If a line drawn X, Y, and Z are the midpoints of sides
P Q AB, BC and AC respectively. ...(Given)
through the midpoint of one
side of a triangle is parallel 1
XZ = BC ...(Midpoint theorem)
2
to second side then it bisects
B C 1
the third side. R = × 11
2
Given : In ∆ABC, P is the midpoint of side AB. ∴ XZ = 5.5 cm
Line PQ side BC 1
XY = AC ...(Midpoint theorem)
Line PQ intersects side AC in point Q. 2
To prove : seg AQ ≅ seg QC =
1
×9
2
Construction : Draw seg QR side AB and
∴ XY = 4.5 cm
let seg QR intersect side BC in point R.
1
Proof : seg PQ side BC ...(Given) YZ = AB ...(Midpoint theorem)
2
∴ seg PQ side BR ...(i) (B - R - C) 1
= ×5
seg QR side AB (Construction) 2
∴ YZ = 2.5 cm
∴ seg QR side PB ...(ii) (A - P - B)
Quadrilaterals 75

(2) In adjoining S L R sides AB, AC, BC respectively.


figure, PQRS ∴ FE =
1
BC ...(i)
and MNRL are 2 (Midpoint
N
M 1 theorem)
rectangles. If ∴ DE = AB ...(ii)
point M is the P Q
2
1
midpoint of side ∴ FD = AC ...(iii)
2
PR then
1 But, ∆ABC is equilateral
prove that, (i) SL = LR, (ii) LN = SQ.
2 ∴ AB = BC = AC ...(iv)
Proof : PQRS and MNRL are rectangles. (Sides of an equilateral triangle)
∠PSR = ∠MLR = 90° ∴ FE = DE = FD ...[From (i), (ii), (iii), (iv)]
...(Angles of a rectangle)
∴ ∆DEF is equilateral ...(By definition)
But, this is pair of corresponding angles
on transversal SR. (4) In adjoining figure, P

∴ seg ML seg PS ...(i) seg PD is median of M


N
(Corresponding angles test.) ∆PQR. Point T is the T
midpoint of seg PD. Q R
In ∆RPS, D
Produced QT
M is midpoint of seg PR ...[Given PM 1
intersects PR at M. Show that = .
and seg ML seg PS and from (i)] PR 3
∴ L is midpoint of seg RS ...(ii) [Hint : Draw DN QM.]
...(Converse of midpoint theorem) Construction : Draw a line passing through point D
∴ SL = LR parallel to seg QM,
Again ∠PQR = ∠MNR = 90° such that P - M - N - R
...(Angles of rectangle) Proof : In ∆PDN,
But, this is pair of corresponding angles Point T is the midpoint of seg PD
On transversal QR. ...(Given)
∴ seg MN seg PQ ...(iii) and seg TM side DN ...(i)
...(Corresponding angles test.) (Construction, Q - T- M)
In ∆RPQ, ∴ Point M is midpoint of seg PN ...(i)
Point M is midpoint ... [Given and ...(Converse of midpoint theorem)
of seg RP from (iii)] In ∆MQR,
seg MN seg PQ
Point D is the midpoint of seg QR
∴ Point N is midpoint of seg QR ...(iv) ∴
...( PD is median)
(Converse of midpoint theorem)
and seg DN side QM ...(Construction)
In ∆QRS, L and N are midpoints of
∴ Point N is midpoint of seg MR ...(ii)
sides SR and QR respectively.
...(Converse of midpoint theorem)
...[From (ii) (iv)]
1 PM = MN ...[From (i)]
LN = SQ ...(Midpoint theorem)
2 MN = NR ...[From (ii)]
(3) ∆ABC is an equilateral A ∴ PM = MN = NR ...(iii) [From (i), (ii)]
traingle. Point F, D and E PR = PM + MN + NR ...(P - M - N - R]
are midpoints of side F E ∴ PR = PM + PM + PM ...[From (iii)]
AB, side BC, side AC
∴ PR = 3 PM
respectively. Show that
∆FED is an equilateral B D C ∴
1
=
PM
3 PR
triangle.
PM 1
In ∆ABC, F, E, D are the midpoints of i.e. =
Proof : PR 3
76 Master Key Mathematics - II (Geometry) (Std. IX)

PROBLEMS FOR PRACTICE Line passing through midpoint of one side


and parallel to the other side bisects the
(1) In the adjoining S A R third side.
figure, PQRS is a
parallelogram in B
∴ point G is the midpoint of side AF.
which A is midpoint O C ∴ AG = GF
of side SR and point But GF = 2 GD ...(Construction)
B is a point on P Q
∴ AG = 2 GD
1
diagonal PR such that RB = 4 PR. AG 2
∴ = i.e. AG : GD = 2 : 1
GD 1
Also, seg AB when produced meets side QR at
point C. Prove that C is midpoint of side QR.
(2) In the adjoining A
PROBLEM SET - 5 (Textbook Page No. 73)
figure, point X is Z
(1) Choose correct alternative answer and fill in
midpoint of side BC,
the blanks.
seg XZ side AB, B X Y C
seg YZ seg AX. (i) If all pairs of adjacent sides of a quadrilateral
1 are congruent then it is called ................. .
Prove that YC = BC.
4 (A) rectangle (B) parallelogram
(3) Points A, B, C and D are midpoints of sides (C) trapezium (D) rhombus
PQ, QR, RS and SP respectively. Prove that Ans. (D)
ABCD is a parallelogram.
(ii) If the diagonal of a square is 12 2 cm then the
perimeter of the square is ................. .
 Additional Information :
(A) 24 cm (B) 24 2 cm (C) 48 cm (D) 48 2
You know the property that the point of
Ans. (C)
concurrence of medians of a triangle divides the
(iii) If opposite angles of a rhombus are (2x)°and
medians in the ratio 2 : 1. Proof of this property is
(3x – 40)° then value of x is ................. .
given below. A
(A) 100° (B) 80° (C) 160° (D) 40°
Ans. (D)
Given : seg AD E
and seg BE are the (2) Adjacent sides of a rectangle are 7 cm and
G
medians of ∆ABC 24 cm. Find the length of its diagonal.
which intersect at B C D C
D
point G.
F
To prove : AG : GD = 2 : 1
Construction : Take point F on ray AD such that Solution: A B
G-D-F and GD = DF ABCD is a rectangle.
Proof : Diagonals of BGCF bisect each other AB = 24 cm
...(Given and construction) and BC = 7 cm
∴ BGCF is a parallelogram In ∆ ABC, ∠ABC = 90°
∴ seg BE seg FC ...(Angle of a rectangle)
Now point E is the midpoint of side AC ∴ AC2 = AB2 + BC2...(Pythagoras theorem)
of ∆AFC ...(Given) = 242 + 72
seg EB seg CF = 576 + 49
∴ AC2 = 625
Quadrilaterals 77

∴ AC = 25 ...(Taking square roots) Perimeter of ABCD = AB + BC + CD + AD


The length of the diagonal of the ∴ 112 = 3x + 4x + 3x + 4x
∴ ...(From (ii), (iii) and given)
rectangle is 25 cm.
∴ 14x = 112
S R
(3) If diagonal of a square is ∴ x =8
13 cm then find its side. ∴ AB = 3x = 3 × 8 = 24 cm
BC = 4x = 4 × 8 = 32 cm
Solution: ∴
P Q The length of sides of the parallelogram
PQRS is a square are 24 cm, 32 cm, 24 cm, 32 cm
PR = 13 cm
(5) Diagonals PR and QS S R
PQ = QR = RS = PS ...(i) of a rhombus PQRS
(Sides of a square) are 20cm and 48cm M
In ∆ PQR, ∠PQR = 90° respectively. Find the
...(Angle of a square) length of side PQ.
P Q
∴ PQ2 + QR2 = PR2...(Pythagoras theorem) Solution:
∴ PQ2 + PQ2 = 132 ...[From (i) and given] PQRS is a rhombus
∴ 2PQ 2
= 169 Diagonals PR and QS intersect at point M.
169 PR = 20cm and QS = 48 cm
∴ PQ2 =
2 Diagonals of a rhombus are bisectors of
13
∴ PQ = ...(Taking square root) each other.
2 1 1
13 × 2 ∴ PM = PR and QM = QS
∴ PQ = 2 2
2× 2 1 1
∴ PM = × 20 and QM = × 48
13 2 2 2
∴ PQ = cm ...(ii) ∴ PM = 10cm and QM = 24cm
2
In ∆PMQ, ∠PMQ = 90º ... (Diagonals of rhombus
13 2
∴ The length of each side of square is are perpendicular to each other)
2
∴ PQ2 = PM2 + QM2 ...(Pythagoras theorem)
(4) Ratio of two adjacent sides of a parallelogram
= 102 + 242
is 3 : 4, and its perimeter is 112 cm. Find the
= 100 + 576
length of its each side.
A D ∴ 2
PQ = 676
∴ PQ = 26 cm ...(Taking square roots)
∴ Length of side PQ is 26 cm.

Solution: B C (6) Diagonals of a rectangle PQRS are intersecting


ABCD is a parallelogram in point M. If ∠QMR = 50º then find the
measure of∠MPS.
AB : BC = 3 : 4
S R
Perimeter of ABCD = 112cm.
AB = CD and BC = AD ... (i) (opposite sides M
50º
of a parallelogram are congruent)
AB : BC = 3 : 4 ...(Given) P
Solution: Q
∴ Let the common multiple be x.
∠PMS = ∠QMR
∴ AB = 3x and BC = 4x ... (ii)
...(Vertically opposite angles)
∴ CD = 3x and AD = 4x [from (i) and (ii)] ... (iii) But, ∠QMR = 50º ...(Given)
78 Master Key Mathematics - II (Geometry) (Std. IX)

∴ ∠ PMS = 50º ...(i) (Opposite sides


PR = QS ...(ii) ∴ side AP ≅ side CR ...(iii) of parallelogram
(Diagonals of a rectangle are congurent) side AP ‖ side CR ...(iv) are parallel and
congruent)
PM = 1 PR ...(iii) (Diagonals of a
2 rectangle bisect In BCRQ,
1
SM = QS ...(iv) each other) side BQ‖ side CR ...[From (ii), (iv)]
2
∴ PM = SM ...(v) side BQ ≅ side CR ...[From (i), (iii))]
[From (ii), (iii) and (iv)] ∴ BCRQ, is a parallelogram
In ∆MPS, side PM ≅ side SM ...[From (v)] ...(A quadrilateral is a parallelogram if a pair
of opposite sides is parallel and congruent)
∴ ∠MPS ≅ ∠MSP
...(Isosecles triangle theorem) (8) In the following figure, ABCD is a trapezium.
AB DC. Point P and Q are midpoints of
Let ∠MPS = ∠MSP = x
side AD and side BC respectively.
∠MPS + ∠MSP + ∠PMS = 180
Prove that: (1) PQ AB. (2) PQ = 1 (AB + CD)
...(Sum of the measures of all angles B 2
Solution:
A
of a triangle is 180º)
∴ x + x + 50 = 180 ...[From (vi) and given] P Q

∴ 2x = 180 – 50
∴ 2x = 130 D C E

∴ x = 65 Construction: Draw seg AQ and extend it to


intersect side DC at point E such that D - C - E
∴ ∠MPS = 65º
Proof: ABCD is a trapezium ... (Given)
(7) In the adjacent figure, if side AB || side DC
seg AB seg PQ, seg AB ≅ seg PQ, ∴ side AB || side DE ...(∴ D – C – E)
seg AC seg PR, seg AC ≅ seg PR then prove that, On transversal AE,
seg BC seg QR seg BC ≅ seg QR. ∠BAE ≅ ∠DEA
A P ...(Alternate angles theorem)
∴ ∠BAQ ≅ ∠CEQ ...(i)
B Q ...(∴ A – Q – E and D – C – E)
Proof : In ΔABQ and ΔECQ,
In ABQP, seg BQ ≅ seg CQ
C R
side AB side PQ ...(∴ Q is midpoint of seg BC)
...(Given)
side AB ≅ side PQ ∠AQB ≅ ∠CQE
∴ ABQP is a parallelogram ...(Vertically opposite angles)
...(A quadrilateral is a parallelogram, if a pair ∠BAQ ≅ ∠CEQ ...[From (i)]
of opposite sides is parallel and congruent) ∴ ΔABQ ≅ ΔECQ
(Opposite sides of ...(SAA test of congruency)
∴ side AP ≅ side BQ ...(i)
a parallelogram ∴ seg AB ≅ seg CE ...(ii)
∴ side AP ‖ side BQ ...(ii) ...(c.s.c.t)
are congruent) seg AQ ≅ seg EQ ...(iii)
In ACRP,
In ΔADE,
side AC‖ side PR
...(Given) P is the midpoint of seg AD ...(Given)
side AC ≅ side PR
and Q is the midpoint of seg AE ...[From (iii)]
∴ ACRP is a parallelogram
seg PQ seg DE ...(By Midpoint theorem)
...(A quadrilateral is a parallelogram if a pair
of opposite sides is parallel and congruent) ∴ seg PQ seg DC ...(D - C - E)
Quadrilaterals 79

∴ But seg DC seg AB ...(Given) MCQ’s


∴ seg PQ seg AB
1 (1) In parallelogram PQRS, if adjacent sides are
and PQ = DE ...(By Midpoint theorem) 8 cm and 5 cm then what will be its perimeter?
2
(A) 13 cm (B) 26 cm
...(∴ D - C - E)
1
∴ PQ = (CE + DC)
2 (C) 3 cm (D) 52 cm
1
∴ PQ = (AB + DC) ...[From (ii)] (2) KLMN is a square. What is m∠NMK?
2
(A) 45° (B) 60°
(9) In the following diagram, ABCD is a
(C) 90° (D) 30°
trapezium. AB || DC. Points M and N are
midpoints of diagonal AC and diagonal BD (3) In ∆ABC, M and N are midpoint of sides
respectively. AB and AC respectively. If MN = 7.2 cm, then
BC = ........ .
Prove that MN|| AB. C
D (A) 3.6 cm (B) 10.8 cm
Construction:
(C) 21.6 cm (D) 14.4 cm
Draw seg DM and P M N Q
(4) In a rhombus ABCD, if m∠ACB = 40°, then
extend it to meet
A T B m∠ADB = ........
side AB at point T
such that A – T – B (A) 70° (B) 45°
Proof: seg DC || seg AB ... (Given) (C) 50° (D) 60°
On transversal AC, (5) Diagonals necessarily bisect opposite angles in
∠DCA ≅ ∠CAB a ........ .
...(Alternate angles theorem) (A) rectangle (B) parallelogram
∴ ∠DCM ≅ ∠MAT ...(i) (C) trapezium (D) square

( A – M – C and A – T – B) B) (6) PQRS is rectangle.
S R
In ΔDMC and ΔTMA, OP = ........ .
∠DCM ≅ ∠MAT

5 cm
...[From (i)] (A) 13 cm
O
side CM ≅ side AM (B) 6.5 cm
...(∴ M is the midpoint of seg AC) (C) 26 cm P 12 cm Q

∠DMC ≅ ∠TMA (D) 17 cm


...(Vertically opposite angles) (7) EFGH is a rhombus with ∠FEG = 45°. What
∴ ΔDMC ≅ ΔTMA is ∠EFG?
...(ASA test of congruency) (A) 45° (B) 60°
∴ seg DM ≅ seg TM ...(ii) (C) 75° (D) 90°
...(c.s.c.t.)
seg DC ≅ seg TA ...(iii) (8) Diagonals of a quadrilateral bisect each other. If
In ΔDTB, ∠A = 45°, then ∠B = ........ .
M and N are midpoints of sides DT and (A) 115° (B) 120°
DB respectively ...[From (ii) and Given] (C) 125° (D) 135°
∴ seg MN || seg TB ...(Midpoint Theorem) (9) Side of a square is 4 cm. Length of its diagonal
∴ seg MN || seg AB ...(A – T – B) is ........ .
(A) 4 cm (B) 8 cm
(C) 4 2 cm (D) 8 2 cm
80 Master Key Mathematics - II (Geometry) (Std. IX)

(10) One side of a parallelogram is 4.8 cm and (17) In the adjoining figure, A
3 MN = 3 cm. BP = ........ .
other side is times the first side, what is the
2
perimeter of the parallelogram? (A) 6 cm M N
(A) 12 cm (B) 24 cm (B) 1.5 cm

(C) 36 cm (D) 48 cm (C) 3 cm

(11) KLMN is a parallelogram in which (D) 9 cm B P C


KN = 8 cm. Diagonals meet at point P. KP = 6 cm, (18) ABCD is a rectangle with m∠BAC= 32°. What
NP = 4 cm. What is the perimeter of ∆MPL? is ∠DBC ?
(A) 18 cm (B) 9 cm (A) 32° (B) 58°
(C) 27 cm (D) 15 cm (C) 90° (D) 45°
(12) In ∆PQR , X is midpoint of side PQ. (19) ABCD is a rhombus with ∠ABC = 56°.
seg XZ side QR, seg XY side PR and PQ = ∠ACD = ........ .
15 cm, YZ = ? (A) 28° (B) 56°
(A) 15 cm (B) 20 cm (C) 62° (D) 124°
(C) 7.5 cm (D) 30 cm (20) In the adjoining figure, A D
z
(13) WXYZ is a parallelogram value of y = ........ . A B C D i s a x

(A) 12 (B) 15 W Z parallelogram. What is


28° y
60° the sum of x, y and z?
(C) 6 (D) 7 B C
x (A) 140° (B) 150°
10 4 y
X Y (C) 168° (D) 180°

(14) Measures of opposite angles of a parallelogram ANSWERS


are (60 – x)° and (3x – 4)°. Value of x = ........ .
(1) (B) (2) (A) (3) (D) (4) (C)
(A) 16 (B) 32
(5) (D) (6) (B) (7) (D) (8) (D)
(C) 28 (D) 17
(9) (C) (10) (B) (11) (A) (12) (C)
(15) The diagonals of a rectangle ABCD meet at point
(13) (D) (14) (A) (15) (C) (16) (D)
O. If ∠BOC = 44°, then ∠OAD = ........ .
(17) (C) (18) (B) (19) (C) (20) (D)
(A) 22° (B) 136°
(C) 68° (D) 90°
(16) In the adjoining figure, A
∠PQR = ........ . 50°
(A) 50° P Q
(B) 70°
(C) 90° 60° 70°
(D) 60° B R C
Quadrilaterals 81

ASSIGNMENT - 5
Time : 1 Hr. Marks : 20

Q.1. Attempt the following : (2)


(1) ABCD is a rectangle. AB = 7 cm, BC = 24 cm. Find AC.
(2) ABCD is a parallelogram. ∠A = x°, ∠B = (3x + 20)°, find the value of x.

Q.2. Attempt the following : (4)


(1) Adjacent sides of a parallelogram are in the ratio 1 : 2. Find the length of each side of this
parallelogram if its perimeter is 36 cm.
(2) PQRS is a rhombus. If PQ = 7.5 cm. Find QR. If ∠QPS = 75° then find ∠PQR, ∠SRQ.

Q.3. Attempt the following : (6)


P
(1) In ∆PQR, seg PO is median. T is midpoint
M
of median PO. seg QT is extended to meet
T
PM 1
side PR at point M. Prove : = Q O R
PR 3
D

(2) In the adjoining figure, G is the centroid


of ∆DEF. H is a point on ray DG such that
G
D - G - H and DG = GH.
Prove that GEHF is a parallelogram. E F

Q.4. Attempt the following : (8)


(1) ABCD is a trapezium in which side AB side DC. A B

P and Q are the midpoints of sides AD and BC respectively.


P Q
1
Prove seg PQ side AB and PQ = (AB + DC)
2 D C

D C
(2) In the adjoining diagram ABCD is a trapezium.
AB DC. Point M and N are midpoints of the Q
M N
diagram AC and diagonal BD respectively.
A T B
Prove that MN AB

vvv
82 Master Key Mathematics - II (Geometry) (Std. IX)

6 Circle
• The union of a circle and its interior is called the
Points to Remember: circular region.
A • If point P is the centre of the circle and 'r' is the
 Circle : [Definition]
radius of the circle and if
• Circle is the set of all points in P
d(P, A) = r, then point A is on the circle.
a given plane which are at a D
constant distance from a fixed B d(P, A) < r, then point A is in the interior of the
point. C circle. and d(P, A) > r, then point A is in the
exterior of the circle.
The fixed point is called the centre.
 Circles in a plane.
• The fixed (constant) distance is called radius
of the circle. In the above figure, point P is the (1) Concentric circles :
centre and seg PA, Seg PB, seg PC and seg PD Circles having the same centre P A
are the radii. but different radii are called B
B R
 Terms related to circle : concentric circles.
Q
(1) Chord : The segment In the adjoining figure, the two circles are
O
joining any two points A concentric circles as they have same centre P
of the circle is called a and different radii PA and PB.
P
S
chord. In the adjoining l (2) Intersecting circles :
figure, seg AB is the Coplanar circles having two points in common
chord. are called intersecting circles.
(2) Diameter : The chord passing through the centre In the adjoining
of the circle is called the diameter. figure, circles A
• Diameter is the longest chord of the circle. with centres P &
Q are intersecting
P Q
• Diameter is twice the radius of the circle.
• In the above figure seg RS is the diameter. circles as they
have two points B
(3) Secant : A line in the plane of the circle
A a n d B i n
intersecting the circle at two distinct points is
common between them.
called a secant.
(3) Touching Circles : Coplanar circles having one
• Secant always contains a chord.
point in common are called touching circles.
• In the above figure, line PQ is a secant.
(4) Tangent : A line in the plane of the circle
touching the circle at one and only one point is
P P
called a tangent. A B
B A
• The point at which tangent touches the circle is
called the point of contact.
Figure (i) Figure (ii)
• In the above figure, line l is a tangent and A is
Internally touching circles Externally touching circles
the point of contact.
 Points in the plane of a circle. In figure (i), circles with centres A and B are
touching each other internally at point P.
• A circle divides a plane into three disjoint
regions. These three regions are the circle itself, In figure (ii), circles with centres A and B are
its exterior and its interior. touching each other externally at point P.
(82)
Circle 83

(4) Congruent circles : Circles having equal radii side PC ≅ side PD


are called congruent circles. ...(Radii of same circle)
∴ �PCM ≅ �PDM ...(By SSS test)
∴ ∠PMC ≅ ∠PMD ...(c.a.c.t.)

A B ∴ ∠PMC = ∠PMD ...(i)


O P
∠PMC + ∠PMD = 180°
...(Angles forming a linear pair)
∴ ∠PMC + ∠PMC = 180° ...[From (i)]
In above figure, circles with centre O and P
are congruent as their radii OA and PB are ∴ 2 ∠PMC = 180
congruent. 180
∴ ∠PMC =
2
Theorem - 1 : ∴ ∠PMC = 90°
Statement : The perpendicular drawn from the ∴ seg PM ⊥ chord CD
centre of a circle to a chord bisects the chord.
Given : In a circle with MASTER KEY QUESTION SET - 6
centre P, seg PE
P PRACTICE SET - 6.1 (Textbook Page No. 79)
⊥ chord AB such
that A - E - B. A B (1) Distance of chord AB from the centre of a circle
E is 8 cm. Length of the chord AB is 12 cm. Find
To Prove : seg AE ≅ seg BE
the diameter of the circle.
Construction : Draw seg PA and seg PB.
Given :
Proof : In �PEA and �PEB
(i) A circle with centre 'O' O
∠PEA and ∠PEB = 90° ...(Given)
(ii) seg OM is ⊥ to the
hypotenuse PA ≅ hypotenuse PB A
chord AB, A - M - B M
B
...(Radii of same cicle)
(iii) OM = 8 cm; AB = 12 cm.
side PE ≅ side PE ...(Common side)
To find : Diameter of the circle.
∴ �PEA ≅ �PEB
Solution :
...(By hypotenuse side test)
Seg OM ⊥ Chord AB ...(Given)
∴ side AE ≅ side BE ...(c.s.c.t.)
1
AM = AB
2
Theorem - 2 :
...(Perpendicular drawn from the centre of
Statement : The segment joining midpoint of a the circle to the chord bisects the chord.)
chord and the centre of a circle is perpendicular to 1
∴ AM = × 12 = 6 cm
the chord. 2
In �OMA, ∠OMA = 90° ...(Given)
Given : In a circle with
∴ OA2 = OM2 + AM2 ...(Pythagoras theorem)
centre P, M is
P
= 82 + 62
the midpoint of
= 64 + 36
chord CD. C D
M OA = 100
2
To Prove : seg PM ⊥ chord CD
∴ OA = 10 cm ...(Taking square roots)
Construction : Draw seg PC and seg PD.
i.e. Radius of the circle is 10 cm
Proof : In �PCM and �PDM,
Diameter = 2 × radius
side PM ≅ side PM ...(Common side)
= 2 × 10
side CM ≅ side DM ...(Given)
∴ Diameter of the circle is 20 cm
84 Master Key Mathematics - II (Geometry) (Std. IX)

(2) Diameter of a circle is 26 cm and length of the ∴ AM2 = 1150 – 900


chord of a circle is 24 cm. Find the distance of ∴ AM2 = 256
the chord from the centre.
∴ AM = 16 cm ...(Taking square roots)
Given :
Seg PM ⊥ chord AB ...(Given)
(i) A circle with centre 'P' P 1
AM = × AB ...(Perpendicular drawn from
2
and diameter 26 cm. the centre of the circle to

(ii) Length of chord C D the chord bisects the chord.)
M
CD = 24 cm ∴ 16 = 1 × AB
2
(iii) seg PM ⊥ chord CD, C - M - D
∴ AB = 16 × 2
To find : PM
∴ AB = 32 cm
Solution :
Diameter of the circle = 26 cm ...(Given) (4) Radius of a circle with centre O is 41 units.
Diameter 26 Length of a chord PQ is 80 units, find the
Radius = 2 = 2 distance of the chord from the centre of the
∴ Radius of the circle = 13 cm circle.
∴ PC = 13 cm Given :
Seg PM ⊥ chord CD ...(Given) (i) A circle with centre 'O' O
1 and radius 41 cm.
∴ CM = 2 CD ...(Perpendicular drawn from
(ii) seg OM ⊥ chord PQ, P Q
the centre of the circle to M
the chord bisects the chord.) P - M - Q
1
CM = 2 × 24 = 12 cm (iii) l(PQ) = 80 cm

To find : l(OM)
In �PMC, ∠PMC = 90° ...(Given)
Solution : OP = 41 cm ...(Radius of the circle)
∴ PC2 = PM2 + CM2 ...(Pythagoras theorem)
∴ Seg OM ⊥ chord PQ ...(Given)
∴ 132 = PM2 + 122
1
∴ 169 – 144 = PM2 ∴ PM = PQ ...(Perpendicular drawn from
2
the centre of the circle to
∴ PM = 25
2
the chord bisects the chord.)

∴ PM = 5 cm ...(Taking square roots) 1
∴ PM = × 80 = 40 cm
2
(3) Radius of the circle is 34 cm and the distance
of a chord from the centre is 30 cm. Find the In �OMP, ∠OMP = 90° ...(Given)
length of the chord. ∴ OP2 = OM2 + PM2 ...(Pythagoras theorem)
Given : ∴ 412 = OM2 + 402
P
(i) A circle with centre 'P' ∴ 1681 = OM2 + 1600
and radius 34 cm. A B ∴ 1681 – 1600 = OM2
M
(ii) seg PM ⊥ chord AB, A - M - B ∴ OM2 = 81
(iii) l(PM) = 30 cm ∴ OM = 9 cm ...(Taking square roots)
To find : l(AB)
(5) Centre of two circles is
Solution : PA = 34 cm ...(Radius of the circle)
O. Chord AB of bigger
In �PMA, ∠PMA = 90° ...(Given) circle intersects the O
∴ PA = PM + AM ...(Pythagoras theorem)
2 2 2
smaller circle in points
A P M Q B
342 = 302 + AM2 P and Q.
1156 = 900 + AM2 Show that AP = BQ.
Circle 85

To prove : AP = BQ
PROBLEMS FOR PRACTICE
Construction : Draw seg OM ⊥ chord AB, A - M - B.
(1) The radius of a circle with
Proof : In the smaller circle,
centre P is 25 cm and the P
Seg OM ⊥ Chord PQ ...(Construction)
length of the chord is
∴ PM = MQ ...(i)
48 cm. Find the distance A
(Perpendicular drawn from the centre M B
of the chord from the
of the circle to the chord bisects the chord.)
centre.
In the bigger circle,
Seg OM ⊥ Chord AB ...(Construction) (2) A chord of length 30 cm
O
∴ AM = MB is drawn at a distance of
...(Perpendicular drawn from the centre 8 cm from the centre of
to the chord bisects the chord.) the circle. Find the P M Q
∴ AP + PM = MQ + QB radius of the circle.
...(A - P - M, M - Q - B) (3) In the adjoining figure, C
∴ AP + PM = PM + QB ...[From (i) ] O is the centre of the N
∴ AP = QB circle. D
O
Y
AB = 16 cm CD = 14 cm.
(6) If a diameter of a circle C
D Seg OM ⊥ seg AB and
bisects two chords of Q A M B
ON ⊥ seg CD. If
the circle then those
O OM = 6 then find seg ON.
two chords are parallel
to each other. (4) If chord PQ of a circle have length equal to the
A P B
Given : radius, then find the distance of the chord from
X
(i) A circle with centre 'O' the centre of the circle in terms of radius.

(ii) Diameter XY intersects chord AB and chord CD


at points P and Q respectively ANSWERS
(iii) AP = BP and CQ = DQ (1) 7 cm (2) 17 cm (3) 51 cm
To Prove : chord AB chord CD. (4) Distance of chord PQ from the centre is
3
r
2
Proof : AP = BP ...(Given)
∴ P is the midpoint of seg AB
∴ seg OP ⊥ chord AB ...(Segment joining Points to Remember:
centre of the circle and the midpoint of the
chord, is perpendiculars to the chords.)  Properties of Congruent Chords
i.e. ∠OPB = 90° ...(i)
Theorem - 3 :
CQ = DQ ...(Given)
Statement : Congruent chords of a circle are
∴ Q is the midpoint of seg CD equidistant from the centre of the circle.
∴ seg OQ ⊥ chord CD ...(Segment joining Given : D
centre of the circle and the midpoint of the N
(i) A circle with
chord, is perpendiculars to the chords.) C
centre 'P'
i.e. ∠OQC = 90° ...(ii) P
(ii) Chord AB ≅ chord
∠OPB + ∠OQC = 90 + 90 = 180 CD.
...[Adding (i) and (ii)] A M B
(iii) seg PM ⊥ chord
i.e. ∠QPB + ∠PQC = 180° ...(Q - O - P)
AB, A - M - B
∴ chord AB chord CD
(iv) seg PN ⊥ chord CD, C - N - D
...(Interior Angles Test)
86 Master Key Mathematics - II (Geometry) (Std. IX)

To Prove : seg PM ≅ seg PN PRACTICE SET - 6.2 (Textbook Page No. 82)
Construction : Draw seg PA and seg PC.
(1) Radius of circle is 10 cm. There are two chords
Proof : AM = 1 AB...(i) (Perpendicular drawn of length 16 cm each. What will be the distance
2 from the centre of the of these chords from the centre of the circle?
circle to the chord
CN = 1 CD ...(ii)
N D
Given : C
2 bisects the chord.)
(i) A circle with centre P
But, AB = CD ...(iii) ...(Given) P
and radius 10 cm.
B
∴ AM = CN ...(iv) [From (i), (ii) and (iii)] (ii) chord AB ≅ chord CD
M
∴ In �PMA and �PNC (iii) AB = CD = 16 cm. A
∠PMA ≅ ∠PNC ...(Each 90°) (iv) seg PM ⊥ chord AB, A - M - B
Hypotenuse PA ≅ Hypotenuse PC (v) seg PN ⊥ chord CD, C - N - D.
...(Radii of same circle) To find : PM and PN
side AM ≅ side CN ...[From (iv)] Solution : PA = 10 cm ...(Radius of the circle)
∴ �PMA ≅ �PNC ...(Hypotenuse - side test) seg PM ⊥ chord AB ...(Given)
∴ seg PM ≅ seg PN ...(c.s.c.t.) 1
∴ AM = AB ...(Perpendicular drawn
2
from the centre of the circle
Theorem - 4 :
to the chord bisects the chord)
Statement : Chords which are equidistant from the 1
centre of a circle are congruent. S ∴ AM = × 16 = 8 cm
2
N
Given : In �PMA, ∠PMA = 90° ...(Given)
R
(i) A circle with A ∴ PA2 = PM2 + AM2 ...(Pythagoras theorem)
centre 'A' ∴ 102 = PM2 + 82
(ii) seg AM ⊥ chord PQ, P Q ∴ 100 – 64 = PM2
M
P - M - Q
∴ PM2 = 36
(iii) seg AN ⊥ chord RS, R - N - S
∴ PM = 6 cm ...(Taking square roots)
(iv) AM = AN
chord AB ≅ chord CD ...(Given)
To Prove : chord PQ ≅ chords RS
∴ PM = PN ...(In a circle, congruent chords
Construction : Draw seg AP and seg AS. are equidistant from the centre)
Proof : In �AMP and �ANS ∴ PN = 6 cm
∠AMP = ∠ANS ...(Each 90°)
(2) In a circle with radius 13 cm, two equal chords
Hypotenuse AP ≅ Hypotenuse AS
are at a distance of 5 cm from the centre. Find
...(Radii of same circle)
the lengths of the chords.
seg AM ≅ seg AN ...(Given) D
N
Given : C
�AMP ≅ �ANS ...(Hypotenuse - side test)
(i) A circle with centre P
seg PM ≅ seg SN ...(i) (c.s.c.t.) P
and radius 13 cm.
PM = 1 PQ ...(ii) (Perpendicular drawn
B
(ii) chord AB ≅ chord CD
2 from the centre of the M
(iii) seg PM ⊥ chord AB, A
SN = 1 RS ...(iii) circle to the chord A - M - B.
2
bisects the chord.)
1 1 (iv) seg PN ⊥ chord CD, C - N - D.
∴ PQ = RS ...[From (i), (ii), (iii)]
2 2 (v) PM = PN = 5 cm
∴ PQ = RS To find : l(AB) and l(CD)
i.e. chord PQ ≅ chord RS Solution : PA = 13 cm ...(Radius of the circle)
Circle 87

In �PMA, ∠PMA = 90° ...(Given)


PROBLEMS FOR PRACTICE
∴ PA = PM + AM ...(Pythagoras theorem)
2 2 2

In the adjoining figure P


(1) A
132 = 52 + AM2 B
chord AB chord CD of C
∴ 169 – 25 = AM2 a circle wih centre O and
Q D
O
∴ AM2 = 144 radius 5 cm. Such that
∴ AM = 12 cm ...(Taking square roots) AB = 6 cm and CD = 8 cm.
∴ PM ⊥ chord AB ...(Given) If OP ⊥ AB, OP ⊥ CD determine PQ
1
∴ AM = AB ...(Perpendicular drawn
2 (2) A chord AB of a circle is at a distance of 6 cm
from the centre of the circle
from the centre of a circle whose radius is 6 cm
to the chord bisects the chord)
less than the chord AB. Find the length of the
1
∴ 12 = × AB chord and radius of the circle.
2
∴ l(AB) = 24 cm (3) The radius of a circle is 16 cm. The midpoint
∴ AB = CD ...(Given) of a chord of a circle lies on the diameter
∴ l(CD) = 24 cm
perpendicular to the chord and its distance
from one end of the diameter is 3 cm. Find the
(3) Seg PM and seg PN are congruent chords of a length of he chord.
circle with centre C. Show that the ray PC is
the bisector of ∠NPM. N (4) In the adjoining figure,
P
P is the centre. seg AB is
Given : B
C smaller than the sum of
(i) A circle with centre C
side PA and side PB by A M B
(ii) chord PM ≅ chord PN P
A M 4 cm. If the perimeter of
To prove : ray PC bisects ∠NPM ∆PAB is 144 cm then find the length of seg PM.
Construction :
Draw seg CA ⊥ chord PM, P - A - M ANSWERS
and seg CB ⊥ chord PN, P - B - N (1) PQ = 1 cm
Proof : (2) length of the chord is 16 cm and radius is 10 cm
chord PM ≅ chord PN ...(Given) (3) The length of the chord is 2 87 cm
∴ CA = CB ...(i) (4) PM = 12 cm
(In a circle, congruent chords
are equidistant from the centre of the circle)
In �PAC and �PBC Points to Remember:
∠PAC ≅ ∠PBC ...(Each 90°)
 Incircle of a Triangle
Hypotenuse PC ≅ Hypotenuse PC
• A circle which touches all three sides of a triangle
...(Common side)
is called incircle of a triangle.
seg CA ≅ seg CB ...[From (i)]
• Centre of incircle is called 'INCENTRE' of the
∴ �PAC ≅ �PBC ...(Hypotenuse - side test)
triangle.
∴ ∠CPA ≅ ∠CPB ...(c.a.c.t.)
• As incentre is equidistant from all three sides, it
∴ ray PC bisects ∠BPA lies on the angle bisectors of all three angles of
i.e. ray PC bisects ∠NPM a triangle.
...(P - A - M, P - B - N) • While constructing incircle it is sufficient to draw
any two angle bisectors to get the incentre.

( Angle bisectors of all three angles of a � are
concurrent)
88 Master Key Mathematics - II (Geometry) (Std. IX)

 Circumicircle of a Triangle
• A circle which passes through all three vertices A
of a triangle is called circumcircle of the triangle.
• Centre of circumcircle is called 'CIRCUMENTRE'
of the triangle.
• As circumcentre is equidistant from all three
vertices, it lies on the perpendicular bisectors
of all three sides of a triangle.
I
• While constructing circumcircle it is sufficient
to draw the perpendicular bisectors of any two
sides. 50°


10

( Perpendicular bisectors of three sides of a P C
B 6.4 cm
triangle are concurrent)
Steps of construction :
 Position of Incentre and circumcentre
(1) Draw seg BC of length 6.4 cm
Type of Equila- Isosceles Scalene (2) Draw an angle of 100° at vertex B and an angle
triangle teral
of 50° at vertex C, name the point of intersection
(1) Position Interior Interior Interior as point A.
of incen- of a tri- of a tri- of a tri-
tre angle angle angle (3) Draw bisectors of ∠ABC and ∠ACB which meet
at point I.
(2) Posi- Interior Interior or Interior
tion of of a tri- exterior or exte- (4) Draw ray IP ⊥ to side BC, B - P - C.
circum- angle or on the rior or (5) Taking 'I' as centre and seg IP as a radius draw
centre side of a on the a circle.
triangle side of a
triangle. (2) Construct �PQR such that ∠P = 70°, ∠R = 50°,
QR = 7.3 cm. and construct its circumcircle.
 Important to know :
Solution : In ∆PQR, ∠P + ∠Q + ∠R = 180°
(1) For an equilateral triangle, angle bisector is also ...(sum of measures of all
perpendicular bisector. angles of a triangle is 180°)
(2) For an equilateral triangle, incentre and ∴ 70 + ∠Q + 50 = 180
circumcentre is one and the same. ∴ ∠Q = 180 – 120
(3) For an equilateral triangle, ratio of circumradius ∴ ∠Q = 60°
and inradius is 2 : 1. Analytical Figure
P

PRACTICE SET - 6.3 (Textbook Page No. 86)


(1) Construct �ABC such that ∠B = 100°, O
BC = 6.4 cm and ∠C = 50° and construct its
60° 50°
incircle. Q 7.3 cm R
Solution : Analytical Figure
A

I
50°

10

B P 6.4 cm C
Circle 89

Steps of construction :
P (1) Construct �XYZ, using lengths of all three sides.
m (2) Draw bisectors of ∠XYZ and ∠XZY.
(3) Name the point of intersection of angle bisectors
as point I.

O
(4) Draw IP ⊥ side YZ, Y - P - C
(5) Taking 'I' as the centre and seg IP as the radius
draw the circle.
60° 50° (4) In �LMN, LM = 7.2 cm, ∠M = 105°, MN =
Q 7.3 cm R
6.4 cm, then draw �LMN and construct its
circumcircle.
Solution : Analytical Figure

Steps of construction : L
l
(1) Draw seg QR of length 7.3 cm p
q

7.2 cm
(2) Draw an angle of 60° at vertex Q and an angle
of 50° at vertex R, name the point of intersection
as point P. 105°
6.4 cm
(3) Draw perpendicular bisectors of seg QR and M N

seg PR, let they intersect at point O.


(4) Taking 'O' as a centre and seg OP as a radius, L
draw a circle.
(3) Construct ∆XYZ such that XY = 6.7 cm, p
YZ = 5.8 cm, XZ = 6.9 cm. Construct its incircle. q
7.2 cm

Solution : Analytical Figure


X

105°
6.9
cm

cm

6.4 cm
6. 7

X M N
I

Steps of construction :
(1) Draw �LMN as per the given measurements.
Y 5.8 cm Z
(2) Draw line p as perpendicular bisector of side
cm

MN.
6.9
6.7

cm

(3) Draw line q as perpendicular bisector of side


LM.
(4) Name the point of intersection of line p and line
I q as point 'O'
(5) Taking point 'O' as centre and seg OL as a radius,
draw a circle.

Y P Z
5.8 cm
90 Master Key Mathematics - II (Geometry) (Std. IX)

Construct �DEF such that DE = EF = 6 cm,


PROBLEM SET - 6 (Textbook Page No. 86)
(5)
∠F = 45° and construct its circumcircle.
(1) Choose correct alternative answer and fill in
Solution : Analytical Figure
the blanks.
D
l
(i) Radius of a circle is 10 cm and distance of a chord

6 cm
O m from the centre is 6 cm. Hence the length of the
chord is .......... .
45° (A) 16 cm (B) 8 cm (C) 12 cm (D) 32 cm
6 cm
Ans. (A)
E F

D (ii) The point of concurrence of all angle bisectors


of a triangle is called the .......... .
l (A) centroid (B) circumcentre
(C) incentre (D) orthocentre
Ans. (C)
6 cm

O m
(iii) The circle which passes through all the vertices
of a triangle is called .......... .
(A) circumcircle (B) incircle
45° (C) congruent circle (D) concentric circles
E 6 cm F
Ans. (A)
(iv) Length of a chord of a circle is 24 cm. If distance
of the chord from the centre is 5 cm, then the
Steps of construction : radius of that circle is .......... .
(A) 12 cm (B) 13 cm (C) 14 cm (D) 15 cm
(1) Draw �DEF as per the given measurements.
Ans. (B)
(2) Draw perpendicular bisector of seg EF, name
the line as line l. (v) The length of the longest chord of the circle with
radius 2.9 cm is .......... .
(3) Draw perpendicular bisector of seg DF, name
(A) 3.5 cm (B) 7 cm (C) 10 cm (D) 5.8 cm
the line as line m.
Ans. (D)
(4) Name the point of intersection of line l and line
m as point 'O'. (vi) Radius of a circle with centre O is 4 cm. l(OP) =
4.2 cm, say where point P will lie.
(5) Taking point 'O' as centre and seg OD as a radius
(A) on the centre (B) inside of the circle
draw the circle.
(C) outside of the circle (D) on the circle
Ans. (C)
PROBLEMS FOR PRACTICE
(vii) The lengths of parallel chords which are on
(1) Draw the circumcircle of ∆PMT in which opposite sides of the centre of a circle are 6 cm
PM = 5.4 cm, ∠P = 60°, ∠M = 70° and 8 cm. If radius of the circle is 5 cm, then the
distance between these chords is .......... .
(2) Construct ∆DCE, such that DC = 7.9 cm,
(A) 2 cm (B) 1 cm (C) 8 cm (D) 7 cm
∠C = 135°, ∠D = 20° and draw its circumcircle. r
Ans. (D) D

(3) Construct any right angle triangle and draw


its incircle. (2) Construct incircle q
cm

and circumcircle of
7.5

Construct ∆ SRP such that RP = 6 cm, ∠R = 75°


7.5

cm

(4) an equilateral O
and ∠P = 55° and draw its incircle. �DSP with side 7.5
cm. Measure the S M P
(5) Construct the circumcircle and incircle at an 7.5 cm
radii of both the
equilateral ∆XYZ with side 7.3 cm Analytical Figure
circles and find the
Circle 91

ratio of radius of circumcircle to the radius of Steps of construction :


r
incircle.
D (1) Draw �NTS as per the given measurement.
(2) Draw bisectors of NTS and NST intersecting at
point I.
(3) Draw IM ⊥ to seg TS, T - M - S.
q

m
(4) Taking point I as a centre and IM as a radius
c draw a circle.
7.5

7.5
c
(5) Draw line l and line m as perpendicular

m
O
bisectors of side TS and side TN, intersecting
at point 'O'.
(6) Taking point 'O' as a centre and seg ON as a
S M P radius draw the circle. Q
7.5 cm
(4) In the adjoining figure, C R S
is the centre of the circle. P
r Seg QT is diameter,
C
Circumradius = 3.8 cm CT = 13 cm, CP = 5, find the
Inradius = 1.9 cm length of chord RS.
Circumradius : Inradius = 3.8 cm : 1.9 cm = 2 : 1. Construction : Draw seg CR. T
Steps of construction : Solution : CT = CR. ...(Radius of the same circle)

(1) Draw �DSP as per given measurements. ∴ CR = 13 ...( CT = 13, given)
(2) Draw line r and line q as bisectors of sides In �CPR, ∠CPR = 90° ...(Given)
PS and DP respectively. Name the point of ∴ CR = CP + RP
2 2 2
...(Pythagoras theorem)
intersection of line r and line q as point O.
13 = 5 + RP
2 2 2

(3) Draw a circle taking 'O' as a centre and OD as


∴ 169 – 25 = RP2
a radius.
∴ RP2 = 144
(4) Draw a circle taking 'O' as a centre and OM as
a radius. ∴ RP = 12 units ...(Taking square roots)
(3) Construct �NTS in which NT = 5.7 cm, seg CP ⊥ chord RS, ...(Given)
TS = 7.5 cm and ∠NTS = 110° and draw incircle 1
RP = RS ...(Perpendicular drawn
2
and circumcircle of it. l from the centre of the circle
Analytical Figure N to the chord bisects the chord)
1
m
∴ 12 = × RS
2
5.7 cm

I ∴ RS = 24 units B C
110° E
(5) In the adjoning figure, P
7.5 cm
T M S
is the centre of the circle. D
N M A
chord AB and chord CD P
l
intersect on the diameter
N at a point E.
m If ∠AEP ≅ ∠DEP then
prove that AB = CD.
5.7 cm

O
Construction : Draw seg PM ⊥ chord AB, A - M - B.
Draw seg PN ⊥ chord CD, C - N - D.
I
Proof : In �PME and �PNE,
110°

side PE ≅ side PE ...(Common side)
T M S
∠PME ≅ ∠PNE ...(Each 90°)
7.5 cm
∠PEM ≅ ∠PEN
...(Given, A - M - E, D - N - E)
92 Master Key Mathematics - II (Geometry) (Std. IX)

∴ �PME ≅ �PNE ...(SAA test) (4) The perpendicular drawn from the centre of a
∴ seg PM ≅ seg PN ...(c.s.c.t.) circle to a chord ........ the chord.
i.e. chord AB and chord CD are equidistant (A) parallel (B) trisects
from the cenre P.
(C) bisects (D) equal
∴ chord AB ≅ chord CD.
(5) If the radius of the circle is 7.6 cm then its
...(In a circle, if chords are equidistant from
diameter is ......... .
the centre then they are congruent.)
(A) 13.2 cm (B) 7.6 cm
(6) In the adjoining figure, C (C) 3.8 cm (D) 15.2 cm
CD is a diameter of the
(6) In the adjoining figure,
circle with centre O.
OM ⊥ AB.
Diameter CD is O O
perpendicular to chord OM = 3, AM = 4,
AB at point E. Show that A E
B then OA = ........?
A B
∆ABC is an isosceles (A) 25 (B) 5
M
D
triangle. (C) 2.5 (D) 10
To prove : ∆ABC is an isosceles triangle. (7) Using the given P

12 cm
Proof : Diameter CD ⊥ chord AB ...(Given) information, find QR.

20
cm
∴ seg OE ⊥ chord AB ...(C - O - E - D) (A) 6 (B) 32 R
Q M
∴ AE = BE ...(i) (Perpendicular drawn (C) 8 (D) 42
from the centre of the circle (8) In the adjoining figure,
to the chord bisects the chord.) OP ⊥ MN and MN = 24 O
In �AEC and �BEC, then MP = ........?
side CE ≅ side CE ...(Common side) (A) 24 (B) 6
M P N
∠AEC ≅ ∠BEC ...(Each 90°) (C) 12 (D) 4
side AE ≅ side BE ...[From (i)] (9) In the adjoining figure, chord AB ≅ chord CD.
∴ �AEC ≅ �BEC ...(SAS test) PM ⊥ AB, PN ⊥ CD, D
∴ side AC ≅ side BC ...( c.s.c.t.) then what is the relation
between PM and PN ? N
∴ �ABC is an isosceles triangle
(A) PM > PN P
...(Definition) C
(B) PM < PN
(C) PM ⊥ PN A B
M
MCQ’s (D) PM = PN
(10) How many circles can be drawn through two
(1) The segment joining any two points on the circle
give points?
is called ........ .
(A) one (B) two
(A) tangent (B) radius
(C) none (D) infinite
(C) chord (D) none of the above
(11) In a circle, chords which are equidistant from
(2) Diameter is ........ of radius.
the centre are ........ .
(A) half (B) twice
(A) parallel (B) congruent
(C) thrice (D) one-fourth
(C) intersecting (D) all of above
(3) Two circles having same centre but different
(12) The point of intersection of the perpendicular
radii are called ........ circles.
bisectors of a trianlge is called ........ .
(A) concentric (B) congruent
(A) centroid (B) incentre
(C) intersecting (D) touching
(C) circumcentre (D) orthocentre
Circle 93

(13) Incircle of a triangle should touch ........ sides of (19) Using the information R
S
a triangle. given in the figure, what
(A) 1 (B) 3 is m∠ROS?
P
(C) 2 (D) none (A) 30° (B) 45° 60°
O

(14) For a right angle triangle, the circumcentre lies (C) 60° (D) 90°
on ........ of the triangle. Q
(A) on the hypotenuse (B) in the interior (20) Using the information
(C) in the exterior (D) on the vertex given in the figure, find
A
(15) For a scalene triangle, the incentre lies ........ of the m∠ADB.
the triangle. (A) 80° (B) 90°
(C) 60° (D) 70°
B C
(A) in the exterior (B) on the side D
(C) in the interior (D) none of the above
(16) Radius of the circle with centre O is 7 cm,
l (OP) = 8 cm. Then the position of point P is ANSWERS
........ of the circle.
(1) (C) (2) (B) (3) (A) (4) (C)
(A) in the exterior (B) in the interior
(5) (D) (6) (B) (7) (B) (8) (C)
(C) on the circle (D) on the centre
(9) (D) (10) (D) (11) (B) (12) (C)
(17) In which triangle is the circumcentre and (13) (B) (14) (A) (15) (C) (16) (A)
incentre the same point?
(17) (B) (18) (D) (19) (C) (20) (B)
(A) isosceles (B) equilateral
(C) scalene (D) right angled
(18) If the radius of the circle is 3.9 cm, then the length
of the longest chord is ........ .
(A) 3.9 cm (B) 9.3 cm
(C) 1.3 cm (D) 7.8 cm
Circle 94

ASSIGNMENT - 6
Time : 1 Hr. Marks : 20

Q.1. Attempt the following : (2)


(1) The radius of a circle is 7 2 cm. Find the length of larges t chord of that circle.

(2) The radius of a circle with centre O is 6 cm. OP = 6.3 cm and OQ = 5.8 cm. State with reason
which point lies in the interior of the circle.

Q.2. Attempt the following : A (4)


(1) In the adjoining figure,
AP = 34, AM = 30, AM ⊥ PQ. P
M Q
Find the length of chord PQ. C
B
O
(2) In a circle with centre O,
diameter AB and diameter A
CD are given. Prove that AD = BC D

Q.3. Attempt the following : (6)


(1) Prove that perpendicular drawn from the centre of a circle to the chord bisects the chord.

(2) If a line intersects two concentric


circles with centre A in points P, Q, R A

and S respectively. Prove that PQ = RS.


P Q R S

Q.4. Attempt the following : (8)


(1) Construct the circumcircle of ∆PQR in which QR = 6.5 cm, ∠Q = 125°, PQ = 4.4 cm
(2) Construct the incircle of right angled ∆ABC, such that AB = 5 cm, BC = 7 cm, ∠B = 90°

vvv
Co-ordinate Geometry 95

7 Co-ordinate Geometry
q Introduction :
To determine co-ordinates of a point in a
A pair of numbers which represents the position quadrant.
of a point in a plane of two mutually perpendicular Y
lines is called an ordered pair.
3
The two mutually perpendicular lines are called B P
2
co-ordinate axes.
1
Q
q Origin, Axes and Quadrants : X’ X
o A
The position of a point in a plane is determined –3 –2 –1 1 2 3 4
–1
with reference to two fixed mutually perpendicular
–2
number lines, called as co-ordinate axes. R S
–3
Y
4 Y

IInd Quadrant 3 Ist Quadrant To determine co-ordinates of a point in a


2 quadrant we have to draw a perpendicular to each
1 of the axis.
X’ X Let us understand with an example. Point P is
o
–3 –2 –1 1 2 3 4 5 in the Ist quadrant . From point P we have drawn
–1 a ^ to X-axis at which intersects X-axis at point A
–2 which represents ‘1’.
IIIrd Quadrant IVth Quadrant
–3 \ X co-ordinate of point P is 1.
Y’
From point P we have drawn a ^ to Y axis which
l The horizontal number line is called X-axis intersects Y-axis at point B which represents
represented as XX’. ‘2’.
l The vertical number line is called Y-axis \ Y co-ordinate of point P is 2.
represented as YY’. \ P (1, 2)
l The point of intersection of these two axes is
called the ‘origin’ and represented as ‘O’. Points to Remember:
l Ray OX is positive direction and ray OX’ is
negative direction of X-axis. q Coordinates of points in a plane :

l Ray OY is positive direction and ray OY’ is


When we write co-ordinates of a point, the first
negative direction of Y-axis. number is x-coordinate and the second number is
y-coordinate.
The co-ordinate axes divide the plane of the paper
x coordinate is also called ‘Abscissa’.
into four regions, XOY, X’OY, X’OY’ and XOY’. Each
of this region is called ‘Quadrant’. y coordinate is also called ‘Ordinate’.

Region XOY is called Ist Quadrdant. (1) If a point is in the first quadrant then its both
Region X’OY is called IInd Quadrdant. the coordinates are positive. e.g. P(1,2).
Region X’OY’ is called IIIrd Quadrdant. (2) If a point is in the second quadrant then its
x coordinate is negative and y coordinate is
Region XOY’ is called IVth Quadrdant.
positive. e.g. Q(–2,1).

(95)
96 Master Key Mathematics - II (Geometry) (Std. IX)

(3) If a point is in the third quadrant then its both MAster KeY QuestIOn set - 7
the coordinates are negative. e.g. R(–3, –2).
(4) If a point is in the fourth quadrant then its PrACtICe set - 7.1 (Textbook Page No. 93)
x coordinate is positive and y coordinate is (1) state in which quadrant or on which axis do
negative S(2, –2). the following points lie.
X-coordinate Y-coordinate A (–3, 2), B (–5, –2), K (3.5, 1.5), D (2, 10),
I Quadrant + +
e (37, 35), F (15, –18), G (3, –7), H (0,–5),
II Quadrant – +
M (12, 0), n (0, 9), P (0, 2.5), Q (–7, –3)
III Quadrant – –
IV Quadrant + – solution:

q Coordinates of points on the Axes. A(–3, 2) IInd Quadrant G (3, –7) IV quadrant

(1) y coordinate of any point on x axis is ‘0’. B (–5, –2) IIIrd Quadrant H (0, –5) y-axis.

(2) x coordinate of any point on y axis is ‘0’. K (3.5, 1.5) Ist Quadrant M (12, 0) x-axis.

(3) Coordinates of the origin are (0, 0). D (2, 10) Ist Quadrant N (0, 9) y-axis.
E (37, 35) Ist Quadrant P (0, 2.5) y-axis.
Y F (15, –18) IVth Quadrant Q (–7, –3) IIIrd quadrant .

3 (2) In which quadrant the following points lie?


2 B (i) Whose both coordinates are positive.
1 solution:
C A
X’ X Ist quadrant.
o
–3 –2 –1 1 2 3 4
–1 (ii) Whose both coordinates are negative.
–2 D solution:
–3 IIIrd quadrant.
Y (iii) Whose x coordinate is positive and y coordinate
For point A, x coordinate is 3 and y coordinate is negative.
is 0. solution:
\ A(3, 0) IVth quadrant.
For point B, x coordinate is 0 and y coordinate (iv) Whose x coordinate is negative and y coordinate
is 2. is positive.
\ B (0, 2) solution:
For point C, x coordinate is –1 and y coordinate IInd quadrant.
is 0. (3) using cartesian coordinate system, plot the
\ C (–1, 0) following points.
For point D, x coordinate is 0 and y coordinate L(–2, 4), M(5, 6), n(–3, –4), P(2, –3),
is –2. Q(6, –5), s(7, 0), t(0, –5).
\ D (0, –2)
Co-ordinate Geometry 97

solution:
scale : 1cm = 1 unit on
Y M both axes.
6

L 4

1
X’ S X

–8 –7 –6 –5 –4 –3 –2 –1 O 1 2 3 4 5 6 7 8
–1

–2

–3
P
N –4

T
–5
Q
–6
Y’

Problems for Practice AnsWers

(1) from the given coordinates, state to which


(1) A - I, B - III, M - II, S - IV, Q - I, T - III
quadrant do the following points belong:
(2) (i) II quadrant (ii) III quadrant (iii) on negative
A(8, 9), B(–4, –1), M(–6.2, 4.3),
y-axis (iv) on Positive x-axis.
s(5, –1), Q(7, 3), t(–2, –2)
(2) in which quadrant or on which axes do the
following points lie if:
(i) x < 0 and y > 0
(ii) Both the coordinates are negative
(iii) x = 0 and y < 0
(iv) y = 0 and x > 0
98 Master Key Mathematics - II (Geometry) (Std. IX)
q equation of a line parallel to Axis:
(1) A line parallel to X-axis.
scale : 1cm = 1 unit on
both axes.
Y

2
A B
1
X’ X

0
–6 –5 –4 –3 –2 –1 1 2 3 4 5 6 7 8 9 10
–1

–2

–3
D C
Y’

Observe the line AB and coordinates of all the


points marked on line AB. Points to Remember :
We can see that Y coordinate of all points on line
AB is constant i.e. 1. (1) Equation of a line parallel to X axis is y = b where
Now observe line CD and coordinates of all the b Î R.
points marked on line CD. (2) If b > 0 then line will be parallel to X-axis above
Here also, we can see that Y coordinate of all the X-axis.
points on line CD are constant. i.e. –3. (3) If b < 0 then line will be parallel to X-axis below
If we observe carefully then both these lines are X-axis.
parallel to X axis. (4) Equation of X axis is y = 0.
\ Equation of line AB is y = 1
\ Equation of line CD is y = – 3

q A line parallel to y-axis.

P Y
R

X’ X
O
–6 –5 –4 –3 –2 –1 1 2 3 4 5 6
–1

–2

–3 S
Q Y’
Co-ordinate Geometry 99

Observe the line PQ and coordinates of all the (2) Write the equation of the line parallel to the Y-
points marked on line PQ. axis at a distance of 7 units from it to its left.
We can see that X coordinate of all points on
solution:
line PQ is constant i.e. –5.
Now, observe the line RS and coordinates of Equation of the required line is x = –7.
all the points marked on RS. (3) Write the equation of the line parallel to the X-
Here also, we can see that X coordinate of all axis at a distance of 5 units from it and below
points on line RS is constant i.e. 3. the X-axis.
If we observe carefully then both the lines are solution:
parallel to Y-axis.
\ Equation of line PQ is X = –5. Equation of the required line is y = –5.
\ Equation of line RS is X = 3. (4) Point Q(–3, –2) lies on a line parallel to the
Y-axis. Write the equation of the line and draw
its graph.
Points to Remember: solution:
(1) Equation of a line parallel to Y-axis is x = a where scale : 1cm = 1 unit on
Y both axes.
a Î R.
4
(2) If a > 0 then line will be parallel to y-axis and
on right side of y-axis. 3
(3) If a < 0 then line will be parallel to y-axis and
on left side of y-axis. 2
(4) Equation of y-axis is X = 0.
1
PrACtICe set - 7.2 (Textbook Page No. 97) X’ X

–4 –3 –2 –1 O 1 2 3 4
(1) On a graph paper plot points A(3, 0) B(3, 3) and
C(0, 3). Join A, B and B, C. What is the figure –1
formed? Q (–3, –2)
solution: –2

scale : 1cm = 1 unit on –3


Y both axes.
4
–4
C (0, 3) B (3, 3) Y’
3

2 Equation of the required line is x = –3.


(5) Y-axis and line x = –4 are parallel lines. What is
1 the distance between them?
X’ (3, 0) A X note: In the text book X-axis is corrected to
O Y-axis.
–4 –3 –2 –1 1 2 3 4
solution:
–1
Equation of y axis is x = 0.
Other line’s equation is x = –4.
–2
Distance between these two lines = 0 – (–4) = 0 + 4
–3 = 4 units.
(6) Which of the equations given below have
–4 graphs parallel to the X-axis, and which ones
Y’
have graphs parallel to the Y-axis?
Square ABCO.
100 Master Key Mathematics - II (Geometry) (Std. IX)

(i) x=3 (iii) x + 6 = 0


solution: solution:
It is parallel to y-axis. x + 6 = 0 i.e. x = – 6. It is parallel to y-axis.
(ii) y – 2 = 0 (iv) y = – 5
solution: solution:
y – 2 = 0 i.e. y = 2. It is parallel to x-axis. It is parallel to x-axis.

(7) On a graph paper, plot the points A(2, 3) B(6, –1) and C(0, 5). If those points are collinear then draw the
line which includes them. Write the co-ordinates of the points at which the line intersects the X-axis and
the Y-axis.

scale : 1cm = 1 unit on


Y both axes.
6

5 C (0, 5)

3
A (2, 3)

1
X’ S X

–8 –7 –6 –5 –4 –3 –2 –1 O 1 2 3 4 5 6 7 8
–1 B (6, –1)

–2

–3

–4
Y’

solution:
Yes, the points are collinear.
Coordinates of points of intersection of the line and x axis is (5, 0)
Coordinates of point of intersection of the line and y axis is (0, 5)
(8) Draw the graphs of the following equations on the same system of co-ordinates. Write the co-ordinates
of their points of intersection.
x + 4 = 0; y – 1 = 0; 2x + 3 = 0; 3y – 15 = 0
solution:
(i) x+4=0 \x=–4
(ii) y–1=0 \y=1
–3
(iii) 2x + 3 = 0 \ 2x = –3 \ x = \ x = –1.5
2
15
(iv) 3y – 15 = 0 \ 3y = 15 \ y = \y=5
3
Co-ordinate Geometry 101

scale : 1cm = 1 unit on


Y both axes.
6
3y – 15 = 0
5

2
y–1 =0
1
X’ X

–8 –7 –6 –5 –4 –3 –2 –1 O 1 2 3 4 5 6 7 8
–1

–2

–3
2x + 3 = 0
x+4 =0

–4

T
–5

–6
Y’

Point of intersection of lines (a) When x = 1 (b) When x = 2


x + 4 = 0 and y – 1 = 0 is (–4, 1)
y=2–x y=2–x
Point of intersection of lines
\ y=2–1=1 \ y=2–2=0
x + 4 = 0 and 3y – 15 = 0 is (–4, 5)
(c) When x = 3
Point of intersection of lines
y=2–x
2x + 3 = 0 and y – 1 = 0 is (–1.5, 1)
\ y=2–3=–1
Point of intersection of lines
(ii) 3x – y = 0
2x + 3 = 0 and 3y – 15 = 0 is (–1.5, 5) \ y = 3x
(9) Draw the graphs of the equations given solution:
below. x 1 2 –1
(i) x+y=2 y 3 6 –3
solution: (x, y) (1, 3) (2, 6) (–1, –3)
x + y = 2 i.e. y = 2 – x (a) When x = 1 (b) When x = 2
x 1 2 3 y = 3x y = 3x
y 1 0 –1 \ y=3×1=3 \ y=3×2=6
(x, y) (1, 1) (2, 0) (3, –1)
102 Master Key Mathematics - II (Geometry) (Std. IX)

(c) When x = – 1 (a) When x = 1 (b) When x = 2


y = 3 × (–1) y = 1 – 2x y = 1 – 2x
\ y=–3 \ y = 1 – (2 × 1) \ y = 1 – (2 × 2)
(iii) 2x + y = 1 \ y=1–2 \ y=1–4
\ y = 1 – 2x \ y = –1 \ y = –3
(c) When x = 3
solution:
y = 1 – 2x
x 1 2 3
\ y = 1 – (2 × 3)
y –1 –3 –5
\ y=1–6
(x, y) (1, –1) (2, –3) (3, –5)
\ y=–5

scale : 1cm = 1 unit on


Y both axes.
7

1
X’ X

–6 –5 –4 –3 –2 –1 1 2 3 4 5 6

–1
x

–2
+
y
=
2

–3
2x
+

–4
0

y=
y=

1

3x

–5

–6
Y’

Problems for Practice (i) Parallel to X-axis and 5 units below it.
(ii) Parallel to Y-axis and 1 unit on the right of it.
(1) Draw the graphs of the following equations: (iii) Parallel to X-axis and 7 units above it.
(i) y=–2 (ii) 3(x + 2) = 2x – 4 (iv) Parallel to Y-axis and 3 units on the left of it.
(iii) 2y + 7 = 0 (3) Draw the graphs of the following equations:
(2) Write the equations of the following lines: (i) x + 2y = 0 (ii) 2x – 3y = 0 (iii) 3x – 2y = 0
Co-ordinate Geometry 103

(4) Draw the graph of the following equation 3x + scale : 1cm = 1 unit on
2y = 0. find the co-ordinates of the point where Y both axes.
4
the graph intersects the Y-axis.
(5) Draw the graphs of the following equations 2x + y P
3
= – 10 and 2x – 3y = 6 on the same graph paper.
Q
2
AnsWers
1
(2) (i) y = – 5 (ii) x = 1 (iii) y = 7 (iv) x = –3
X’ M X
PrOBLeM set - 7 (Textbook Page No. 98) –4 –3 –2 –1 O 1 2 3 4

(1) choose the correct alternative answer for each –1 T R


of the following questions.
S –2
(i) What is the form of co-ordinates of a point on
the X-axis. –3
(A) (b, b) (B) (0, b) (c) (a, 0) (d) (a, a)
–4
Ans. (C)
Y’
(ii) Any point on the line y = x is of the form .....
(A) (a, a) (B) (0, a) (c) (a, 0) (d) (a, –a)
(i) Write the coordinates of point Q and point R.
Ans. (A) Ans. Q(–2, 2) and R(4, –1)
(iii) What is the equation of X-axis? (ii) Write the coordinates of point T and point M.
(A) x = 0 (B) y = 0 (c) x + y = 0 (d) x = y Ans. T(0, –1) and M(3, 0)
Ans. (B) (iii) Which point lies in the IIIrd quadrant?
(iv) In which quadrant does point (–4, –3) lies? Ans. Point S
(A) First (B) Second (c) Third (d) Fourth (iv) Which are the points whose x and y coordinat
Ans. (C) are equal.
Ans. Origin O(0, 0)
(v) What is the nature of the line which includes the
points (–5, 5), (6, 5), (–3, 5), (0, 5)? (3) Without plotting the following points, state
in which quadrant or on which axis does the
(A) Passing through the origin
following point lie.
(B) Parallel to Y-axis (c) Parallel to X-axis
(i) (5, –3) (ii) (–7, –12)
(d) None of the above
(iii) (–23, 4) (iv) (–9, 5)
Ans. (c)
(v) (0, –3) (vi) (–6, 0)
(vi) Which of the points P(–1, 1), Q (3, –4), R (1, –1)
Ans.
S(–2, –3), T (–4, 4) lie in the fourth quadrant?
(i) Fourth quadrant (ii) Third quadrant
(A) P and T (B) Q and R
(iii) Second quadrant (iv) Second quadrant
(c) Only S (d) P and R
(v) Y-axis (vi) X-axis
Ans. (B)
(4) Plot the following points on the one and the
(2) observe the following graph and answer the
same co-ordinate system.
following questions.
A (1, 3), B (–3, –1), c (1, –4), d (–2, 3), E (0, –8),
F (1, 0)
104 Master Key Mathematics - II (Geometry) (Std. IX)
Ans.
scale : 1cm = 1 unit on
Y
both axes.
5

D
4

A
3

1
X’ F X

–8 –7 –6 –5 –4 –3 –2 –1 O 1 2 3 4 5 6 7 8
–1
B

–2

–3

–4
C

–5

–6

–7

E
–8
Y’

Ans.
(5) In the adjoining figure line LM is parallel to
Y scale : 1cm = 1 unit on
Y-axis. both axes.
4
(i) What is the distance between line LM and
Y-axis? 3 L
Equation of Y-axis is x = 0
2 P
Equation of line LM is x = 3.
distance between line LM and Y-axis = 3 – 0 = 3. 1
(ii) Write the coordinates of points P, Q and R. X’ R X
Ans. P(3, 2) Q(3, –1) R(3, 0) –4 –3 –2 –1 O 1 2 3 4
(iii) Find the difference between x - coordiante of –1 Q
point L and point M.
Ans. L(3, 3) M(3, –3) The x-coordiante of point L is 3 –2

and the x-coordinate of point M is 3.


–3 M
\ Difference between x-coordinates = 3 – 3 = 0
–4
Y’
Co-ordinate Geometry 105

(6) How many lines are there which are parallel (10) For the equation, –3x + 4y = 12, the value of
to X- axis and having a distance 5 units? y-coordinate is 3 then the value of x-coordinate
Ans. There can be two lines. is .................. .
(i) y = 5 (ii) y = –5 (A) 0 (B) 1 (c) 2 (d) 3
(7) if a is any real number then find the distance (11) In which quadrant or on which axis will (x, y) lie,
between Y-axis and x = a. if x < 0 and y = 0.
Ans. distance between Y-axis and a line is |a| with (A) I quadrant (B) II quadrant
equation x = a. (c) negative X-axis (d) positive X-axis
MCQ’s 7
(12) For the given line 2y = 3x + 2 if x = then y = ...
(1) If x coordinate is positive and y coordiante is 3
2 9 4 5
negative the point lies in ..................... . (A) (B) (c) (d)
9 2 5 4
(A) I quadrant (B) III quadrant (13) The equation of a line is 5x – 4y = P. If x = 2 and
(c) II quadrant (d) IV quadrant y = 0 then P = .............. .
(2) If x coordinate of a point is zero, then the point (A) 5 (B) 10 (c) –10 (d) –5
lies on ..................... . (14) The equation of a given line is 8x – 3y + 4 = 0. If
(A) IV quadrant (B) X-axis y = –4 then x = .............. .
(c) Y-axis (d) Origin (A) –1 (B) –2 (c) –3 (d) –4
(3) In III quadrant both the coordinates are ............... . (15) The equation of a line is 2x – 5y – 7= 0. If y = 0 then
(A) x -positive, y-negative x = .............. .
(B) x -negative, y-negative (A) 2.5 (B) 2 (c) 3 (d) 3.5
(c) x -positive, y-positive 4y – 12
(16) If x = and the value of y = 3 then the
(d) x -negative, y-positive 3
(4) If the coordinates of a point are (0, a) where value of x = .............. .
a = 6, then the point belongs to ..................... . (A) 3 (B) 2 (c) 1 (d) 0

(A) I quadrant (B) X-axis (17) For the given equation 3x + 2y = 4 the value of y
(c) II quadrant (d) Y-axis is .............. .
4 + 3x 3x – 4
(5) The point of intersection of the lines y = –3 and (A) (B)
2 2
x = 4 is ..................... . 4 – 3x
(c) (d) None of above
(A) (–3, 4) (B) (2, 3) (c) (4, –3) (d) (4, 3) 2
(6) For the equation x – 2y = 0 4 – 3x
(18) If y = and x = 1, then y = .............. .
2
When x = –2, the coordinate of the point is .........
(A) 2 (B) 3 (c) 0.5 (d) 4
(A) (2, 1) (B) (–2, 3) (c) (–2, 2) (d) (–2, 4)
10 – 7y
(7) The equation of a line passing through –3 on the (19) For the given equation x = if y = 1 then
5
x-axis and parallel to y-axis is ..................... . x = ................. .
(A) x = –3 (B) x = 3 (c) y = –3 (d) y = 3 (A) 0.7 (B) 0.6 (c) 1.6 (d) 0.8
(8) On simplifying the equation 3 (x + 1) = 2x – 3 we (20) If a + b = –1 and –a + b = 3 Then adding both the
get ..................... . equations, the value of b is .................... .
(A) –1 (B) 1 (c) –2 (d) 2
(A) x = –4 (B) x = 4 (c) x = 6 (d) x = –6
(9) The point of intersection of the lines AnsWers
y + 4 = 2y + 7 and 2x + 4 = 3x+ 1 is ..................... .
(1) (d) (2) (c) (3) (B) (4) (d) (5) (c)
(A) (3, –3) (B) (–3, 3) (c) (2, 1) (d) (1, 2)
(6) (A) (7) (A) (8) (d) (9) (A) (10) (A)
(11) (c) (12) (B) (13) (B) (14) (B) (15) (d)
(16) (d) (17) (c) (18) (c) (19) (B) (20) (B)
106 Master Key Mathematics - II (Geometry) (Std. IX)

assiGNmeNt - 7
time : 1 Hr. marks : 20

Q.1. Attempt the following: (2)


(1) From the given co-ordinates, state to which quadrant do the following points belong:
A(–4, –1) B(5, –8)
(2) In which quadrant or on which axis will (x, y) lie if x > 0 and y > 0
Q.2. Attempt the following: (4)
(1) (i) Line PQ is parallel to y-axis and passing through the point (–4, 3). Write the equation
of the line PQ.
(ii) Line AB is parallel to x-axis and passing through the point (6, 7). Write the equation
of line AB.
(2) Given below are some points with their co-ordinates. Answer the following questions.
Q(–3, 2), R(5, 1), S(0, –4)
(i) What is the y co-ordinate of point S.
(ii) What is the x co-ordinate of point Q.
(iii) What is the y co-ordinate of point R.
Q.3. Attempt the following: (6)
(1) Write the co-ordinates of the following points in ordered pairs.
(i) Point B : x co-ordinate is 4 and y co-ordinate is –5.
(ii) Point F : y co-ordinate is 6 and x co-ordinate is 0.
(iii) Point O : origin.
(2) draw x axis and y axis on a graph paper and plot the following points.
A(1.5, 2.5) B(0, –3) c(4, 0) P(–4, –3) Q(0, 6) R(2, 5)
Q.4. Attempt the following: (8)
(1) draw the graphs of the following equations on the same graph paper.
(i) 2y + 1 = y + 3 (ii) 3(x + 1) = 2x – 3
(2) draw the graphs of the following equations on the same graph paper.
(i) 2x – 3y = 5 (ii) 2x – 5y – 7 = 0
Trigonometry 107

8 Trigonometry

Master Key QuestioN set - 8


Points to Remember:
P Practice set - 8.1 (Textbook Page No. 104)
In DPQR, PQR = 90º (1) In the figure, R is the right angle of DPQr.
\ side PR is hypotenuse. Write the following ratios.
Let R = q. (i) sin P P R
\ With respect to q, q
Q R (ii) cos Q
side PQ is the opposite side and (iii) tan P
side QR is the adjacent side. (iv) tan Q
Q
Now we shall study three trigonometric ratios, Solution:
viz sine (sin), cosine (cos) and tangent (tan). Opposite side of P
(i) sin P =
Opposite side Adjacent side Hypotenuse
sine = ; cosine =
Hypotenuse Hypotenuse QR
\ sin P =
Opposite side PQ
and tangent =
Adjacent side Adjacent side of Q
(ii) cos Q =
So with respect to above figure, Hypotenuse
QR
Opposite side of q PQ \ cos Q =
sinq = = PQ
Hypotenuse PR
Opposite side of P
Adjacent side of q QR (iii) tan P =
cosq = = Adjacent side
Hypotenuse PR
QR
\ tan P =
Opposite side of q PQ PR
tanq = =
Adjacent side of q QR Opposite side of Q
(iv) tan Q =
Note: Adjacent side
Cosecant, secant and cotangent are three PR
\ tan Q =
more ratios in trigonometry. These ratios are QR
multiplicative inverse of sine, cosine and
tangent ratios. (2) In the right angled DXYZ, XYZ = 90º and a,
1 1 b, c are the lengths of the sides as shown in the
i.e. cosec q = , sec q = and
sin q cos q figure. Write the following ratios,
1 (i) sin X Y b X
cot q =
tan q (ii) tan Z
(iii) cos X
With respect to figure given above a
c
(iv) tan X
PR PR QR
cosec q = , sec q = and cot q =
PQ QR PQ Z

(107)
108 Master Key Mathematics - II (Geometry) (Std. IX)

Solution: Adjacent side of a QR


Opposite side of X YZ a cos a = =
(i) sin X = = = Hypotenuse PR
Hypotenuse XZ c
Opposite side of a PQ
Opposite side of Z XY b tan a = =
(ii) tan Z = = = Adjacent side of a QR
Adjacent side of Z YZ a
Opposite side of q QS
Adjacent side of X XY b (ii) sin q = =
(iii) cos X = = = Hypotenuse PS
Hypotenuse XZ c
Opposite side of X YZ a Adjacent side of q PQ
(iv) tan X = = = cos q = =
Adjacent side of X XY b Hypotenuse PS
Opposite side of q QS
(3) In right angled DLMN, LMN = 90º tan q = =
Adjacent side of q PQ
L = 50º and N = 40º,
Write the following ratios.
ProbLeMs for Practice
(i) sin 50º L
(ii) cos 50º 50º (1) In the following figure, DXYZ is a right angled
(iii) tan 40º triangle with Y = 90º. Write all trigonometric
ratio of X and Z.
(iv) cos 40º 40º
M N Y
Solution:
Opposite side of L MN
(i) sin 50º = =
Hypotenuse LN
Z X
Adjacent side of L LM
(ii) cos 50º = = (2) In the following figure, write tan x, cos(90 – y),
Hypotenuse LN sin y, tan(90 – x), sin(90 – y), cos x.
Opposite side of N LM B
(iii) tan 40º = =
Adjacent side of N MN

Adjacent side of N MN x
(iv) cos 40º = = A y C
Hypotenuse LN

(4) In the figure, PQR = 90º, PQS = 90º,


D
PrQ = a and QPs = q.
P (3) in DUVW, mUVW = 90º, UV = 6, UW = 8.
Find trigonometric ratios of W.
q
U

a 8
R S 6
Q
Write the following trigonometric ratios.
V W
(i) sin a, cos a, tan a
(ii) sin q, cos q, tan q
Solution: aNsWers
Opposite side of a PQ
(i) sin a = = YZ XY YZ
Hypotenuse PR (1) (i) sin X = ; cos X = ; tan X =
XZ XZ XY
Trigonometry 109

XY YZ XY q For additional information:


(ii) sin Z = ; cos Z = ; tan Z =
XZ XZ YZ 1
(1) sin q × cosec q = 1 i.e. sin q =
cosec q
AB AD
(2) (i) tan x = (ii) cos(90 – y) = 1
BC AC (2) cos q × sec q = 1 i.e. cos q =
sec q
AD BC
(iii) sin y = (iv) tan(90 – x) = 1
AC AB (3) tan q × cot q = 1 i.e. tan q =
cot q
CD BC (4) sec q = cosec (90 – q)
(v) sin(90 – y) = (vi) cos x =
AC AC
(5) cosec q = sec(90 – q)
3 7 3
(3) (i) sin W = ; cos W = ; tan W = (6) tan q = cot (90 – q)
4 4 7
(7) cot q = tan(90 – q)

q Trigonometric Ratios of special angles:


Points to Remember: (i) In DPQR, P = 30º, P

q Relation between Trigonometric ratios of Q = 90º and R = 60º, 30º


complementary angles: PR = 2a 2a

X DPQR is 30º-60º-90º
triangle 60º
90–q Q R
1
QR = × PR (Side opposite to 30º)
2
q
Y Z 1
\ QR = × 2a = a
In DXYZ, XYZ = 90º 2
\ seg XZ is hypotenuse 3
PQ = × PR (Side opposite to 60º)
Z = q; X = 90 – q (Remaining angle) 2
With respect to q 3
\ PQ = × 2a = 3 a.
Opposite side is XY, Adjacent side is YZ 2
With respect to (90 – q) (A) Trigonometric ratios of angle measuring 30º
Opposite side is YZ QR a 1
(i) sin 30º = = =
Adjacent side is XY PR 2a 2
XY YZ PQ 3a 3
(i) sin q = (ii) cos q = (ii) cos 30º = = =
XZ XZ PR 2a 2
XY YZ QR a 1
(iii) tan q = (iv) sin(90 – q) = (iii) tan 30º = = =
YZ XZ PQ 3a 3
XY YZ
(v) cos(90 – q) = (vi) tan(90 – q) = (B) Trigonometric ratios of angle measuring 60º
XZ XY
PQ 3a 3
Three relations: (i) sin 60º =
PR
=
2a
=
2
(i) sin q = cos(90 – q) ... [From (i) and (v)] QR a 1
(ii) cos 60º = = =
(ii) cos q = sin(90 – q) ... [From (ii) and (iv)] PR 2a 2
XY YZ 3a
(iii) tan q × tan(90 – q) = × =1 PQ
YZ XY (iii) tan 60º = = = 3
QR a
... [From (iii) and (vi)]
110 Master Key Mathematics - II (Geometry) (Std. IX)

(ii) In DABC, A q Trigonometric Identity:


A
A = C = 45º, 45º
a
B = 90º, AC = a
DABC is 45º-45º-90º 45º
B C
triangle. q
B C
1
\ AB = BC = × AC (Side opposite to 45º) In DABC, B = 90º, C = q
2
AB
1 a \ sin q = ...(i)
\ AB = BC = ×a= AC
2 2 BC
and cos q = ...(ii)
(C) Trigonometric ratios of angle measuring AC
45º: In DABC, ∠B = 90º
AB a a 1 1 \ AB2 + BC2 = AC2 ... [Pythagoras theorem]
(i) sin 45º = = ÷a= × =
AC 2 2 a 2
Dividing throughout by AC2
BC a a 1 1
(ii) cos 45º = = ÷a= × = AB2 BC2 AC2
AC 2 2 a 2 + =
AC2 AC2 AC2
AB a a
(iii) tan 45º = = ÷ =1 \ (sin q)2 + (cos q)2 = 1 ... [From (i) and (ii)]
BC 2 2
Note:
(D) Trigonometric ratios of angle measuring 0º (sinq) 2 means square of sinq which is
and 90º: conventionally written as sin2q.
(i) sin 0º = 0 Hence above identity can be written as
(ii) cos 0º = 1 sin2q + cos2q = 1

sin 0º 0
(iii) tan 0º = = =0
cos 0º 1 Practice set - 8.2 (Textbook Page No. 112)
(iv) sin 90º = 1 (1) In the following table one of the trigonometric
(v) cos 90º = 0 ratio is given. Using this find remaining
trigonometric ratios.
sin 90º 1
(vi) tan 90º = = = Not defined
cos 90º 0 11 1 3
sin q
We summarise all the values calculated in (A), 61 2 5
35 1
(B), (C) and (D) in the following table. cos q
37 3
0º 30º 45º 60º 90º 21 8 1
tan q 1
1 1 3 20 15 2 2
sin 0 1
2 2 2 Solution:
3 1 1 (i) Method I A
cos 1 0
2 2 2
1 Not
tan 0 1
3
3 defined q
B C
In the above adjoining, in DABC
B = 90º and C = q
Trigonometry 111

BC 72 × 2
\ cos q = ... (i) (By definition) =
AC 372
35 144
But cos q = ... (ii) (Given) sin2q =
37
372
BC 35
\ = ... [From (i) and (ii)] 12
AC 37 \ sin q =
37
Let the common multiple be k (k ¹ 0)
\ BC = 35k; AC = 37k sinq 12 35
tanq = = ÷
cosq 37 37
In DABC, B = 90º ... (Given)
\ AC2 = AB2 + BC2 ... (Pythagoras theorem) 12 37
\ tanq = ×
37 35
\ (37k)2 = AB2 + (35k)2
\ (37k)2 – (35k)2 = AB2 12
\ tanq =
35
\ AB2 = (37k + 35k) (37k – 35k)
\ AB2 = 72k × 2 k 11
(ii) sin q = ... (Given)
2
\ AB = 144k 2
61
\ AB = 12 k ... (Taking square roots) sin2q + cos2q = 1 ... (Trigonometric identity)
AB æ 11 ö÷2
sin q = \ çç ÷÷ + cos2q = 1
AC çè 61 ÷ø
12k 112
\ sin q = \ cos2q = 1 –
37k 612
12 612 – 112
\ sin q = \ cos2q =
37 612
AB (61 + 11)(61 – 11)
tan q = \ cos2q =
BC 612
12k 72 × 50
\ tan q = =
35k 612
12 3600
\ tan q = cos2q =
35 612
60
(i) Method II \ cos q = ... (Taking square roots)
61
sin q + cos q = 1
2 2
... (Trigonometric identity)
2 sinq
æ 35 ö tan q =
\ sin q + çç ÷÷÷ = 1
2
cosq
çè 37 ÷ø
11 60
352 = ÷
\ sin2q = 1 – 61 61
372
11 61
372 – 352 = ×
\ sin2q = 61 60
372
11
(37 + 35) (37 – 35) tan q =
\ sin2q = 60
372
112 Master Key Mathematics - II (Geometry) (Std. IX)

(iii) In the adjoining figure, A 1


(v) cos q =
3
in DABC, B = 90º
sin q + cos2q = 1
2
... (Trigonometric identity)
C = q æ 1 ö2
q sin2q + çç ÷÷÷ = 1
B C çè 3 ÷ø
tan q = 1 ... (i) (Given) 1
AB \ sin2q + =1
\ tan q = ... (ii) (By definition) 3
BC 1
\ sin2q = 1 –
AB 1 3
\ = [From (i) and (ii)]
BC 1 3–1
\ sin2q =
Let the common multiple be k (k ¹ 0) 3
2
\ AB = k, BC = k \ sin2q =
3
In DABC, B = 90º
\ AC2 = AB2 + BC2 2
... (Pythagoras theorem) \ sin q = ... (Taking square roots)
3
\ AC = k + k 2 2 2

\ AC2 = 2k2 sinq


\ tan q =
\ AC = 2k ... (Taking square roots) cosq
2 1
AB k 1 tan q = ÷
sin q = = = 3
AC 2 3
2k
BC k 1 2 3
cos q = = = = ×
AC 2 3 1
2k
2
(iv) sin q =
1 \ tanq =
2 1
sin q + cos2q = 1
2
... (Trigonometric identity) (vi) In the adjoining figure, A
2
æ1ö
\ çç ÷÷÷ + cos2q = 1 in DABC,
çè 2 ÷ø
B = 90º,
1
+ cos q = 1 2
C = q q
4 B C
1 21
\ cos q = 1 –2
tan q = ... (i) (Given)
4 20
4–1 3 AB
\ cos2q = = tan q = ... (ii) (By definition)
4 4 BC
3 AB 21
\ cos q = ... (Taking square roots) \ = ... [From (i) and (ii)]
2 BC 20
sinq Let the common multiple be k. (k ¹ 0)
tan q = \ AB = 21k and BC = 20k.
cosq
1 In DABC, ABC = 90º
3
= ÷
2 2 \ AC2 = AB2 + BC2 ... (Pythagoras theorem)
1 2 1 = (21k)2 + (20k)2
\ tan q = × =
2 3 3 = 441k2 + 400k2
1 AC2 = 841k2
\ tanq =
3 \ AC = 841k 2 = 29k ... (Taking square roots)
Trigonometry 113

AB 21k 21 3 4
sin q = = = \ tan q = ÷
AC 29k 29 5 5
3 5
BC 20k 20 tan q = ×
cos q = = = 5 4
AC 29k 29
3
\ tan q =
A 4
(vii) In the adjoining figure,
A
in DABC, (ix) In the adjoining figure,
B = 90º, in DABC,
C = q q B = 90º,
B C
8 C = q q
tan q = ... (i) (Given) B C
15 1
tan q = ... (i) (Given)
AB 2 2
tan q = ... (ii) (By definition)
BC AB
tan q = ... (ii) (By definition)
AB 8 BC
\ = ... [From (i) and (ii)]
BC 15 AB 1
Let the common multiple be k. \ = ... [From (i) and (ii)]
BC 2 2
\ AB = 8 k and BC = 15 k. Let the common multiple be k. (k ¹ 1)
In DABC, B = 90º
AB = k ; BC = 2 2 k.
\ AC2 = AB2 + BC2 ... (Pythagoras theorem)
In DABC, B = 90º
\ AC2 = (8 k)2 + (15 k)2
\ AC2 = AB2 + BC2 ... (By Pythagoras theorem)
= 64 k2 + 225 k2
= k2 + (2 2 k)2
AC2 = 289 k2
2
= k2 + 8 k2
\ AC = 289 k = 17 k
AC2 = 9 k2
... (Taking square roots)
AB 8k 8 \ AC = 3k ... (Taking square roots)
sin q = = =
AC 17k 17 AB k 1
sin q = = =
BC 15k 15 AC 3k 3
cos q = = =
AC 17k 17 BC 2 2k 2 2
cos q = = =
3 AC 3k 3
(viii) sin q =
5
(2) Find the values of:
sin q + cos2q = 1
2
... (Trigonometric identity)
(i) 5 sin 30º + 3 tan 45º
æ 3 ö÷2
çç ÷ + cos2q = 1 Solution:
çè 5 ÷÷ø
5 sin 30º + 3 tan 45º
9 æ1ö
+ cos q = 1 = 5 × ççç ÷÷÷ + 3 × (1)
2
25 è 2 ÷ø
9
\ cos2q = 1 – 5
25 = +3
2
25 – 9 16
\ cos2q = = 5+6
25 25 =
2
4
\ cos q = ... (Taking square roots) 11
5 =
2
sin q 11
tan q = \ 5 sin 30º + 3 tan 45º =
cos q 2
114 Master Key Mathematics - II (Geometry) (Std. IX)

4
(ii) tan260º + 3 sin260º 1 1
5 = +
2 4
Solution:
4 2+1
tan260º + 3 sin260º =
5 4
4 æ 3 ö÷2 3
= × ( 3 ) + 3 ´ ççç ÷÷÷
2 =
5 çè 2 ø 4
4 3
= ×3+3× 3
\ cos245º + sin230º =
5 4 4
12 9 48 + 45 93
= + = = (vi) cos60º × cos30º + sin60º × sin30º
5 4 20 20
Solution:
4 93
\ tan260º + 3 sin260º = cos60º × cos30º + sin60º × sin30º
5 20
1 3 3 1
(iii) 2sin30º + cos0º + 3sin90º = ´ + ´
2 2 2 2
Solution:
3 3
2sin30º + cos0º + 3sin90º = +
æ1ö 4 4
= 2 × ççç ÷÷÷ + (1) + 3 × (1)
è 2 ø÷ 2 3
=
= 1+1+3 4
= 5 3
=
\ 2sin30º + cos0º + 3sin90º = 5 2

tan60 \ cos60º × cos30º + sin60º × sin30º =


3
(iv)
sin60 + cos60 2
Solution: 4
(3) If sinq = then find cosq
tan60 5
Solution:
sin60 + cos60
sin2q + cos2q = 1 ... (Trigonometric identity)
æ 3 1ö
= ( 3 ) ¸ ççç + ÷÷÷÷ æ 4 ö÷2
çè 2 2ø
\ ççç ÷÷ + cos2q = 1
æ 3 + 1 ö÷ è 5 ÷ø
= ( 3 ) ¸ ççç ÷÷ 16
çè 2 ÷ø \ cos2q = 1 –
25
2
= 3 ´ 25 – 16
3 +1 \ cos2q =
25
2 3 9
=
3 +1 \ cos2q =
25
\ tan60 2 3 3
= \ cosq = ... (Taking square roots)
sin60 + cos60 3 +1 5

15
(v) cos245º + sin230º (4) If cosq = then find sinq
17
Solution: Solution:
2
cos 45º + sin 30º 2
sin2q + cos2q = 1 ... (Trigonometric identity)
æ 1 ö÷2 æ 1 ö÷2 æ 15 ö2
= çç ÷ + çç ÷ sin 2 q + çç ÷÷÷ = 1
çè 2 ÷÷ø çè 2 ÷÷ø çè 17 ÷ø
Trigonometry 115

225 (iv) What is the value of


cos 28º
=?
\ sin2q + =1
289 sin 62º
(a) 2 (b) – 1 (c) 0 (d) 1
2 225
\ sin q = 1 – ans. (d)
289
289 – 225 (2) In right angled D TSU,
\ sin2q =
T
289 TS = 5, S = 90º, SU = 12

5 cm
64 then find sin T, cos T, tan T.
\ sin2q = Similarly find sin U, cos
289
U, tan U.
S 12 cm U
8
\ sin q = ... (Taking square roots)
17 Solution:
In D UST, S = 90º
ProbLeMs for Practice \ US2 + ST2 = UT2 ... (Pythagoras theorem)
\ 122 + 52 = UT2
3
(1) If cos q = , find other trigonometric ratios \ 144 + 25 = 169 = UT2
2
of q.
\ UT = 13 ... (Taking square roots)
(2) Find the value of 2tan245º + cos230º – sin260º
sin T = =
US 12
45
(3) If sin q = , find cos q. UT 13
53
tan60º – tan30º cos T =
ST
=
5
(4) Find the value of .
1 + tan60º . tan30º
UT 13

Find the value of: tan T = =


US 12
(5)
2(cos245º + tan260º) – 6(sin245º – tan230º)
ST 5

sin U = =
ST 5
aNsWers
UT 13

cos U = =
US 12
1 1
(1) sin q = ; tan q = UT 13
2 3
28 tan U =
ST
=
5
(2) 2 (3)
53
US 12

(4)
1
(5) 6 (3) In right angled D YXZ, X Y
3 = 90º, XZ = 8 cm, yZ = 17 17
cm, find sin y, cos y, tan y,
cm

ProbLeM set - 8 (Textbook Page No. 113) sin Z, cos Z, tan Z.


Solution:
X 8 cm Z
(1) Choose the correct alternative answers for the
following multiple choice questions. In D YXZ, X = 90º
(i) Which of the following statement is true? XZ = 8 cm; YZ = 17 cm
(a) sin q = cos (90 – q) (b) cos q = tan (90 – q) \ YZ2 = XY2 + XZ2 ... (Pythagoras theorem)
(c) sin q = tan (90 – q) (d) tan q = tan (90 – q) \ 172 = XY2 + 82
ans. (a) \ 289 – 64 = XY2
(ii) Which of the following is the value of sin 90º? \ XY2 = 225
1 \ XY = 15 cm ... (Taking square roots)
(a) (b) 0 (c) (d) 1
3
2
sin Y = =
2 XZ 8
ans. (d)
YZ 17
(iii) 2 tan 45º + cos 45º – sin 45º = ?
cos Y = =
XY 15
(a) 0 (b) 1 (c) 2 (d) 3 YZ 17
ans. (c)
116 Master Key Mathematics - II (Geometry) (Std. IX)

(5) Fill in the blanks:


tan Y = =
XZ 8
XY 15 (i) sin20º = cos º
sin Z =
XY
=
15 (ii) tan30º × tan º=1
(iii) cos40º = sin º
YZ 17
cos Z = =
XZ 8
ans. (i) 70º (ii) 60º (iii) 50º
YZ 17
tan Z = = MCQ’s
XY 15
XZ 8
(1) Trigonometry is applicable only in a .................
(4) In the adjoining figure, L
in D LMN, N = q, (a) Equilateral triangle
(b) Right angled triangle
M = 90º cos q = . Find
24
θ (c) Obtuse angled triangle
remaining trigonometric M
25
N (d) acute angled triangle
ratios of q.
(2) For any acute angle of a right angled triangle
Also, find (sin q)2 + (cos q)2. there are ................. trigonometric ratios.
Solution: (a) one (b) three (c) four (d) six
In D LMN, M = 90º
(3) In the given figure, P
\ cos N = (By definition)
MN
LN
cos P = ...........
i.e. cos q = [Q N = q] ... (i)
MN
Q R
LN
But cos q = (Given) ... (ii) PQ PQ PR QR
24
25 (a) (b) (c) (d)
QR PR PQ PR
\ = ... [From (i) and (ii)]
MN 24
LN 25 (4) tanθ = ......................
Let the common multiple be k, (k ¹ 0) adjacent side adjacent side
\ MN = 24k, LN = 25k (a) (b)
Opposite side Hypotenuse
In D LMN, M = 90º ... (Given)
Hypotenuse Opposite side
\ LN2 = LM2 + MN2 [Pythagoras theorem) (c) (d)
\ (25k)2 = LM2 + (24k)2 Opposite side adjacent side

\ 576k2 = LM2 + 625k2 (5) In Δ XYZ, mY = 90° X


\ 625k2 – 576k2 = LM2 then for Z, what is
\ LM2 = 49k2 the trigonometric ratio
XY
\ LM = 7k ... (Taking square roots) for ? Y Z
7k XZ
sin q = = =
LM 7
LN 25k 25 (A) cos Z (B) tan Z
7k (c) sin Z (d) None of above
tan q = = =
LM 7
24k 24
A
MN
 7   24  (6) In Δ ABC, B = 90°,
q
2 2

sin2 q + cos2 q =   +   A = θ.
 25   25 
c = ...........
= +
49 576
C B
625 625

= = =1 (A) (90 + θ) (B) θ


49 + 576 625

(c) (180 – θ) (d) (90 – θ)


625 625

\ sin2q + cos2q = 1
Trigonometry 117

3 (16) In the adjoining figure, A


(7) If sin θ = , then cosec θ = ........ find sin x
2
3
3 2 2 3 2 3 5
(a) (b) (c) (d)
2 3 3 4 x
44 61 B C
(8) If tan θ = and cos θ = , then sin θ = ........
21 66 5 5 2 13
3 11 2 3 (a) (b) (c) (d)
(a) (b) (c) (d) 2 3 3 2
2 4 3 11
(17) If sin θ = cos θ, then tan θ = ..................... .
(9) Find the value of 2 sin60°. cos 30°
(a) 2 (b) 3 (c) 1 (d) 0
5 3 3 1
(a) (b) (c) (d) (18) What is the value of sin2θ + cos2θ ?
2 2 2 4
(a) tan2 θ (b) 0 (c) 1 (d) 2
(10) Find the value of 2sin245° + cos245°.
1 3 (19) If 3 tan x – 1 = 0. What is the value of x?
3 3
(a) (b) (c) (d) (a) 30° (b) 60° (c) 90° (d) 45°
2 2 2 2
(20) What is the value of cosA, if A = 30°?
(11) Find the value of 2tan245° + cos230° + sin260°
9 9 11 7 3
(a) 0 (b) (c) 2
(a) (b) (c) (d) 2
2 4 4 2
(d) Not defined
1
(12) cos ................... = .
2
(a) 0° (b) 45° (c) 90° (d) 60° aNsWers
(13) If tan θ = 3 , then θ = ................... (1) (b) (2) (d) (3) (b) (4) (d)
(a) 60° (b) 30° (c) 45° (d) 90° (5) (c) (6) (d) (7) (b) (8) (c)
(14) tan 52° = ..................... . (9) (b) (10) (b) (11) (d) (12) (d)
(a) tan 38° (b) cosec 38° (c) cot 38° (d) sec 38° (13) (a) (14) (c) (15) (c) (16) (b)
(17) (c) (18) (c) (19) (a) (20) (b)
(15) sin 21° = ..................... .
(a) cos 21° (b) sin 69° (c) cos 69° (d) tan 69°
118 Master Key Mathematics - II (Geometry) (Std. IX)

ASSIGNMENT - 8
Time : 1 Hr. Marks : 20

Q.1. Attempt the following: (2)

2 1
(1) If sin θ = , cos θ = then find tan θ.
3 2
(2) Evaluate: sin 30° + cos 60°
U
Q.2. Attempt the following: (4)

(1) In the adjoining figure, ΔUVW is a right angled triangle and


∠UVW = 90°, UV = 6 cm, VW = 8 cm. Find the sinU and tanW.

V W

(2) Find the value of the following: 2tan245° + cos230° – sin260°


Q.3. Attempt the following: a (6)
(1) In the adjoining figure, Find the following ratios: x

tanx, cos(90 – y), siny, cos(90 – x), tan(90 – x) and sinx

y
b d c
(2) If ∠a = 30º, then show that sinA = 1 − cos 2 a
2
Q.4. Attempt the following: (8)
(1) If ∠a = 30º, then show that cos3A = 4cos3a – 3cosa

3
(2) If cos θ = , Find sin θ and tan θ and also prove that sin2θ + cos2θ = 1
2

vvv
Surface Area and Volume 119

9 Surface Area and Volume


Solution:
Points to Remember : Vertical surface area of the box = 2 (l + b) × h
q Introduction: = 2 (20 + 12) × 10
(I) Cuboid: = 2 × 32 × 10
In the adjoining cuboid, = 640 cm2
h
length, breadth and height are Total surface area of the box = 2(lb + bh + lh)
l, b, and h respectively. b = 2 (20 × 12 + 12 × 10 + 20 × 10)
l
(1) Perimeter of base = 2 (l + b). = 2 (240 + 120 + 200)
(2) Area of base = l × b. = 2 × 560
(3) Vertical surface area = 2 (l + b) × h. = 1120 cm2
(4) Total surface area = 2 (lb + bh + lh). (1) Vertical surface area of box is 640 cm2.
(5) Volume = l × b × h. (2) Total surface area of box is 1120 cm2.
(II) Cube: (2) Total surface area of a box of cuboid shape is
In the adjoining cube, each side 500 sq. unit. Its breadth and height is 6 unit
has length l. l and 5 unit respectively. What is the length of
(1) Perimeter of base = 4l that box?
l
l Solution:
(2) Area of base = l2
(3) Vertical surface area = 4l2 Total surface area = 2(lb + bh + lh)
(4) Total surface area = 6l2 \ 500 = 2 (l × 6 + 6 × 5 + l × 5)
500
(5) Volume = l3 \ = 6l + 30 + 5l
2
(III) Cylinder: (Right Circular r \ 250 = 11l + 30
Cylinder)
\ 250 – 30 = 11l
In the adjoining right circular
\ 220 = 11l
cylinder,
220
radius = r and height = h.
h \ =l
11
(1) Circumference of base = 2pr. \ l = 20 unit
(2) Area of base =pr2 r
Length of the box is 20 units.
(3) Curved surface area = 2prh.
(3) Side of a cube is 4.5 cm. Find the surface area of all
(4) Total surface area = 2pr (r + h). vertical faces and total surface area of the cube.
(5) Volume = pr2h Solution:
Vertical surface area of the cube = 4l2
Master Key Question set - 9 = 4 × 4.5 × 4.5
= 81cm2
Practice set - 9.1 (Textbook Page No. 115) Total surface area of the cube = 6l2
(1) Length, breadth and height of a cuboid shape = 6 × 4. 5 × 4.5
box of medicine is 20cm, 12 cm and 10 cm = 121.5 cm2
respectively. Find the surface area of vertical (1) Vertical surface area of cube is 81 cm2.
faces and total surface area of this box. (2) Total surface area of cube is 121.5 cm2.

(119)
120 Master Key Mathematics - II (Geometry) (Std. IX)

(4) Total surface area of a cube is 5400 sq. cm. Total surface area of a cylinder = 2pr (r + h)
Find the surface area of all vertical faces of the = 2 × 3.14 × 20 × (20 + 13)
cube. = 2 × 3.14 × 20 × 33
Solution: = 4144.8 cm2
Total surface area of the cube = 6l2 (1) Curved surface area is 1632.8 cm2
\ 5400 = 6 × l 2
(2) Total surface area is 4144.8 cm2
5400
\ = l2 (8) Curved surface area of a cylinder is 1980
6 cm2 and radius of its base is 15 cm. Find the
\ l2 = 900 cm2 height of the cylinder .
Vertical surface area of cube = 4l2.
Solution:
= 4 × 900 Curved surface area of a cylinder = 2prh
= 3600 cm2 22
\ 1980 = 2 × × 15 × h
Vertical surface area of cube is 3600 cm2. 7
1980 × 7
(5) Volume of a cuboid is 34.50 cubic metre. \ = h
2 × 22 × 15
Breadth and height of the cuboid is 1.5m and
90 × 7
1.15m respectively. Find its length. \ h =
30
Solution: \ h = 21 cm
Volume of a cuboid = l × b × h
Height of the cylinder is 21 cm
\ 34.5 = l × 1.5 × 1.15
345 15 115 PRoBLemS FoR PRaCTICe
\ = l× ×
10 10 100
(1) The dimensions of a cuboid in cm are 16 × 14
345 × 10 × 100
\ =l × 20. Find its total surface area.
10 × 15 × 115
(2) The cuboid water tank has length 2 m, breadth
\ l = 20 m 1.6 m and height 1.8 m. Find the capacity of the
Length of the cuboid is 20 m tank in litres.
(3) A fish tank is in the form of a cuboid, external
(6) What will be the volume of a cube having
measures of that cuboid are 80 cm × 40 cm ×
length of edge 7.5 cm?
30 cm. The base, side faces and back face are
Solution: to be covered with a coloured paper. Find the
Volume of a cube = l3 area of the paper needed.
= 7.5 × 7.5 × 7.5 (4) The length, breadth, and height of a cuboid
are in the 5 : 4 : 2. If the total surface area is
= 421.875 cm3
1216 cm2, find the dimension of the solid.
Volume of the cube is 421.875 cm3
(5) The volume of a cube is 1000 cm3. Find the total
(7) Radius of base of a cylinder is 20 cm and its surface area.
height is 13 cm, find its curved surface area and (6) The radius and height of a cylinder are in same
total surface area. (p = 3.14) ratio 3 : 7 and its volume is 1584 cm3. Find its
Solution: radius.
Curved surface area of a cylinder = 2prh (7) a cylindrical hole of diameter 30 cm is bored
through a cuboid wooden block with side 1
= 2 × 3.14 × 20 × 13
metre. Find the volume of the object so formed.
= 1632.8 cm2
(p = 3.14)
Surface Area and Volume 121

(8) a building has 8 cylindrical pillars whose cross (2) Find the volume of a cone, if its total surface
sectional diameter is 1 m and whose height area is 7128 sq.cm. and radius of base is 28 cm.
is 4.2 m. Find the expenditure to paint these
pillars at the rate of ` 24 per sq.m.
Solution:
answer Total surface area of a cone =pr (r + l)
22
\ 7128 = × 28 × (28 + l)
(1) 1648 cm2 (5) 5760 litres 7
(3) 8000 cm2 (4) 20 cm, 16 cm, 8 cm 7128 × 7
\ = 28 +
22 × 28
(5) 600 cm2 (6) 6 cm
324
\ = 28 +
(7) 929350 cm3 (8) ` 2534.40 4
\ 81 = 28 + l
\ l = 81 – 28
Points to Remember :
\ l = 53 cm
(IV) Right Circular Cone: l2 = r2 + h2
In the adjoining figure, for \ 532 = 282 + h2
a cone, \ 532 – 282 = h2
l
Radius = r, h \ h2 = (53 + 28) (53 – 28)
Perpendicular height = h, \ h2 = 81 × 25
Slant height = l. \ h=9×5 (Taking square roots)
r
\
2 2 2
(1) l = r + h h = 45 cm
(2) Circumference of base = 2pr 1
Volume = pr2h
(3) Area of base = pr2 3
1 22
(4) Curved surface area = prl = × × 28 × 28 × 45
3 7
(5) Total surface area =pr (r + l) V = 36960 cm3
1
(6) Volume = pr2h Volume of the cone is 36960 cm3
3
(3) Curved surface area of a cone is 251.2 cm2 and
radius of its base is 8 cm. Find its slant height
Practice set - 9.2 (Textbook Page No. 119) and perpendicular height. (p= 3.14)
Solution:
(1) Perpendicular height of cone is 12 cm and its
Curved surface area of a cone =p× r × l
slant height is 13 cm. Find the radius of the
base of the cone. \ 251.2 = 3.14 × 8 × l
Solution: 2512 314
\ = ×8×l
2
l =r +h 2 2 10 100
\ 132 = r2 + 122 2512 × 100
\ =
10 × 314 × 8
\ 169 = r2 + 144
314 × 10
\ 169 – 144 = r2 \ =
314
\ r2 = 25
\ l = 10 cm
\ r = 5 cm l2 = r2 + h2
Radius of the cone is 5 cm \ 102 = 82 + h2
\ 100 – 64 = h2
122 Master Key Mathematics - II (Geometry) (Std. IX)

\ h2 = 36 \ 102 = 62 + h2
\ h = 6 cm \ h2 = 100 – 36
(1) Slant height of the cone is 10 cm \ h2 = 64
(2) Perpendicular height of the cone is 6 cm \ h = 8 cm
(4) What will be the cost of making a closed cone Height of the cone is 8 cm
of tin sheet having radius of base 6 m and (7) Volume of a cone is 1232 cm3 and its height is
slant height 8 m if the rate of making is ` 10 24 cm. Find the surface area of the cone.
per sq.m?
Solution:
Total surface area of a cone = pr (r + l)
Solution:
22 1
= × 6 × (6 + 8) Volume of a cone = pr2h
7 3
22 1 22 2
= × 6 × 14 \ 1232 = × × r × 24
7 3 7
= 264 sq.m 1232 × 7 × 3
\ = r2
Cost = Area × Rate 22 × 24
= 264 × 10 = `2640 154 × 7
\ r2 = =7×7
22
Cost of making closed cone is ` 2640
\ r2 = 49
(5) Volume of a cone is 6280 cubic cm and
\ r = 7 cm
base radius of the cone is 30 cm. Find its
perpendicular height. (p = 3.14) l2 = r2 + h2
Solution: = 72 + 242
1
Volume of a cone = pr2h = 49 + 576
3
1 \ l2
= 625
\ 6280 = × 3.14 × 30 × 30 × h
3 \ l = 25 cm
1 314
\ 6280 = × × 30 × 30 × h Curved surface area of a cone = pr l
3 100
22
6280 × 3 × 100 = × 7 × 25
\ =h 7
314 × 30 × 30 = 550 sq. cm
20
\ h= Curved surface area of cone is 550 sq. cm
3
(8) The curved surface area of a cone is 2200 sq. cm
\ h = 6.67 cm
and its slant height is 50 cm. Find the total
Perpendicular height of a cone is 6.67 cm
surface area of cone.
(6) Surface area of a cone is 188.4 sq.cm and its
slant height is 10 cm. Find its perpendicular Solution:
height. (p= 3.14) Curved surface area of a cone =pr l
Solution: 22
\ 2200 = × r × 50
Surface area of a cone = p × r × l 7
2200 × 7
\ 188.4 = 3.14 × r × 10 \ =r
1884 314 50 × 22
\ = × r × 10 \ r = 14 cm
10 100
1884 × 100 Total surface area of a cone =pr (r + l)
\ =r 22
314 × 10 × 10 = × 14 × (14 + 50)
\ r = 6 cm 7
= 44 × 64
l2 = r2 + h2
Surface Area and Volume 123

= 2816 sq. cm Volume of fodder in Conical shape =


l2 2
= r +h 2 1
= pr2h
\ 50 2
= 142 + h2 3
1 22
\ 502 – 142 = h2 = × × 3.6 × 3.6 × 2.1
3 7
\ h2 = (50 + 14) (50 – 14) 1 22 36 36 21
\ h2 = 64 × 36 = × × × ×
3 7 10 10 10
\ h = 8×6 (Taking square roots) 28512
=
\ h = 48 cm 1000
1 = 28.512 cubic m.
Volume of a cone = pr2h
3
l2 = r 2 + h2
1 22
= × × 14 × 14 × 48 = 3.62 + 2.12
3 7
V = 9856 cubic cm. = 12.96 + 4.41
l2 = 17.37
(1) Total surface area of cone is 2816 sq.cm.
(2) Volume of cone is 9856 cubic cm l = 4.17 m (Taking square roots)
Area of polythene required = pr l
(9) There are 25 persons in a tent which is conical 22
in shape. every person needs an area of 4 sq.m. = × 3.6 × 4.17
7
of the ground inside the tent. If height of the = 47.18 m2
tent is 18 m, find the volume of the tent.
Area of polythene needed is 47.18m2
Solution:
Area of base of tent = Area occupied by a PRoBLemS FoR PRaCTICe
person × Number of
persons (1) a cone of height 24 m has a plane base of
surface area 154 cm2. Find its volume.
= 4 × 25
(2) Curved surface area of a cone with base radius
Area of base of tent = 100 sq. m.
40 cm is 1640psq. cm. Find the height of the
1 cone.
Volume of tent = × Area of base of
3
(3) The total surface area of a cone is 71.28 cm2.
tent × Height of tent Find the height of this cone if the diameter of
1 the base is 5.3 cm.
= × 100 × 18
3
(4) The volume of a cone of height 5 cm is 753.6 cm3.
= 600 cubic m.
This cone and a cylinder have equal radii and
Volume of the tent is 600 cubic metre height. Find the total surface area of cylinder.
(p = 3.14)
(10) In a field, dry fodder for the cattle is heaped in a
conical shape. The height of the cone is 2.1m and (5) The diameter of the base of metallic cone is
diameter of base is 7.2 m. Find the volume of the 2 cm and height is 10 cm. 900 such cones are
fodder if it is to be covered by polythene in rainy melted to form 1 right circular cylinder whose
season then how much minimum polythene radius is 10 cm. Find height of the right circular
cylinder so formed.
sheet is needed?
Solution: answer
Diameter of conical shape = 7.2 m
7.2 (1) 1232 cm3 (2) 9 cm
Radius of conical shape (r) = = 3.6 m
2 (3) 4.5 cm (4) 1281.12 cm2
Perpendicular height (h) = 2.1 m. (5) 30 cm
124 Master Key Mathematics - II (Geometry) (Std. IX)

(ii) Surface area of a sphere = 4pr2


= 4 × 3.14 × 9 × 9
Points to Remember : = 1017.36 cm2
(V) Sphere: 4
Volume of a sphere = pr3
3
In adjoining figure,
4
for the given sphere = × 3.14 × 9 × 9 × 9
3
radius is r. r
V = 3052.08 cm3
(1) Surface area of
sphere = 4pr2 (1) Curved surface area of sphere = 1017.36 cm2
(2) Volume of sphere = 3052.08 cm3
(2) Volume of
4
sphere = pr3 (iii) Surface area of a sphere = 4pr2
3
= 4 × 3.14 × 3.5 × 3.5
Note: Sphere has only one surface, so surface area
= 153.86 cm2
of sphere is often called as curved surface
4
area or total surface area. Volume of a sphere = pr3
3
(VI) Hemisphere: 4
r = × 3.14 × 3.5 × 3.5 × 3.5
In the adjoining 3
figure for the V = 179.50 cm3
given hemisphere
(1) Curved surface area of sphere = 153.86 cm2
radius is r.
(2) Volume of sphere is = 179.50 cm3
(1) Circumference of
flat surface = 2pr (2) If the radius of a solid hemisphere is 5 cm, then
(2) Area of flat surface =pr2 find its curved surface area and total surface
area. (p= 3.14)
(3) Curved surface area = 2pr2
Solution:
(4) Total surface area = 3pr2 Curved surface area of Hemisphere = 2pr2
2
(5) Volume of hemisphere = pr3 = 2 × 3.14 × 5 × 5
3
= 157 cm2
Practice set - 9.3 (Textbook Page No. 123) Total surface area of Hemisphere = 3pr3
= 3 × 3.14 × 5 × 5
(1) Find the surface areas and volumes of sphere = 235.5 cm2
of the following radii.
(1) Curved surface area of hemisphere is 157 cm2
(i) 4 cm (ii) 9 cm (ii) 3.5 cm (p= 3.14)
(2) Total surface area of hemisphere is 235.5 cm3
Solution:
(i) Surface area of a sphere = 4pr2 (3) If the surface area of a sphere is 2826 cm2 then,
find its volume. (p= 3.14)
= 4 × 3.14 × 4 × 4
Solution:
= 200.96 cm2
Surface area of sphere = 4pr2
4
Volume of a sphere = pr3 \ 2826 = 4 × 3.14 × r2
3
314 2
4
= × 3.14 × 4 × 4 × 4 \ 2826 = 4 × ×r
3 100
2826 × 100
V = 267.95 cm3 \ = r2
4 × 314
(1) Curved surface area of sphere = 200.96 cm2 \ r2 = 9 × 25
(2) Volume of sphere = 267.95 cm3 r = 3×5
Surface Area and Volume 125

r = 15
PRoBLemS FoR PRaCTICe
4
Volume of a sphere = pr3
3 (1) Find the volume and surface area of a sphere
4 of radius 4.2 cm.
= × 3.14 × 15 × 15 × 15
3 (2) The volumes of two spheres are in the ratio
4 314 27 : 64. Find their radii if the sum of their radii
= × × 15 × 15 × 15
3 100 is 28 cm.
= 14130 cm3 (3) The surface area of a sphere is 616 cm2. What
Volume of the sphere is 14130 cm3 is its volume?
(4) The curved surface area of a hemisphere is
(4) Find the surface area of a sphere, if its volume 1
905 cm2, what is its radius?
7
is 38808 cubic cm
(5) If the radius of a sphere is doubled, what will
Solution: be the ratio of its surface area to that of the first
4
Volume of a sphere = pr3 sphere.
3
4 22
\ 38808 = × × r3 answer
3 7
38808 × 3 × 7 (1) 310.46 cm3; 221.76 cm2 (2) 12 m, 16 m
\ = r3
4 × 22 (3) 1437.33 cm 3
(4) 12 cm
4,851 × 21
\ = r3 (5) 4:1
11
\ 441 × 21 = r3
ProbleM set - 9 (Textbook Page No. 123)
\ r3 = 21 × 21 × 21
\ r = 21 cm (1) If diameter of a road roller is 0.9 m and its
length is 1.4 m, how much area of a field will
Surface area of a sphere = 4pr2
22 be pressed in its 500 revolutions.
= 4 × × 21 × 21
7 solution:
= 5544 cm2 Diameter of roller = 0.9 m
Surface area of sphere is 5544 cm2 0.9 45
\ Radius of roller (r) = = 0.45 m = m
2 100
(5) Volume of a hemisphere is 18000pcubic cm. 14
Length of roller (h) = 1.4 m = m
Find its diameter. 10
Solution: Area pressed by roller in 1 revolution
2
Volume of a hemisphere = pr3 = curved surface area of roller.
3
2 \ Area pressed by roller in 500 revolution
\ 18000p = pr3 = 500 × curved surface area of roller
3
18000 ×p× 3 = 500 × 2prh
\ = r3
2 ×p 22 45 14
= 500 × 2 × × ×
7 100 10
\ 3 × 3 × 10 × 10 × 10 × 3 = r3
22 45 × 14
\ r = 30 cm = 1000 × ×
7 1000
d = 2r = 1980 sq. m.
\ d = 2 × 30
Area pressed by the roller in 500
\ d = 60 cm
revolutions is 1980 sq. m.
Diameter of hemisphere is 60 cm
126 Master Key Mathematics - II (Geometry) (Std. IX)

(2) To make an open fish tank, a glass sheet of \ r = 5x = 5 × 1 = 5 m


2 mm gauge is used. The outer length, breadth h = 12x = 12 × 1 = 12 m
and height of the tank are 60.4 cm, 40.4 cm and 2
l = r2 + h2
40.2 cm respectively. How much maximum
\ l2 = 52 + 122
volume water will be contained in it?
solution: \ l2 = 25 + 144
External length of fish tank = 60.4 cm \ l2
= 169
External breadth of fish tank = 40.4 cm \ l = 13 m
External height of fish tank = 40.2 cm
Perpendicular height is 12 m and slant
Thickness of glass = 2 mm = 0.2 cm
height is 13 m
Inner length of fish tank (l) = 60.4 – 0.4
= 60 cm (4) Find the radius of a sphere if its volume is
Inner breadth of fish tank (b) = 40.4 – 04 904.32 cubic cm. (p = 3.14)
= 40 cm solution:
Inner height if fish tank (h) = 40.2 – 0.2 4
Volume of sphere pr3
=
= 40cm 3
4
Inner volume of fish tank = l × b × h \ 904.32 = × 3.14 × r3
3
= 60 × 40 × 40 90432 4 314 3
= 96,000 cm3 \ = × ×r
100 3 100
Maximum quantity of water
90432 3 100
96000 \ × × = r3
fish tank can hold = l l = 1000 cm3 100 4 314
1000
\ 72 × 3 = r3
= 96 l
\ r3 = 216
Maximum quantity of water fish tank
\ r = 6 cm (Taking cube roots)
can hold is 96 l.
Radius of a sphere is 6 cm
(3) If the ratio of radius of base and height of a
cone is 5 : 12 and its volume is 314 cubic metre. (5) Total surface area of a cube is 864 sq. cm. Find
Find its perpendicular height and slant height its volume.
(p= 3.14)
solution:
solution:
Total surface area of cube = 6 l2
Let the radius, perpendicular height and slant
height be r, h and l respectively r : h = 5 : 12 \ 864 = 6l2
864
Let the common multiple be x. = l2
6
\ r = 5x ; h = 12x. \ l2 = 144
1
Volume of cone = pr2h. \ l = 12 cm (Taking square roots)
3
1 Volume of cube = l3
\ 314 = × 3.14 × 5x × 5x × 12x
3 = 123
1 314
314 = × × (5x) (5x) (12x) \ Volume of cube = 1728 cubic cm
3 100
314 × 3 × 100 Volume of a cube is 1728 cubic cm
\ = x3
314 × 5 × 5 × 12
(6) Find the volume of a sphere, if its surface area
100
\ x 3
= is 154 sq.cm.
100
solution:
\ x3 = 1
Total surface area of sphere = 4pr2
\ x = 1
Surface Area and Volume 127

22 = 132 × 52
\ 154 = 4 × × r2
7 = ` 6,864
154 × 7 (9) The length of a road roller is 2.1 m and its
\ = r2
4 × 22 diameter is 1.4 m. For levelling a ground 500
49 rotations of the road roller were required. How
\ r2 =
4 much area of ground was levelled by the road
7 roller? Find the cost of levelling at the rate of
\ r = = 3.5 cm (Taking square roots)
2 ` 7 per sq. m.
4
Volume of sphere = pr3 solution:
3
Length of the road roller (h) = 2.1 m
4 22
= × × 3.5 × 3.5 × 3.5 Diameter of the road roller (d) = 1.4 m
3 7
4 22 35 35 35 \ Radius of the road roller (r) = 0.7 m
= × × × × Area pressed in one revolution
3 7 10 10 10
3773 = Curved surface area of road roller
= = 179.67 cubic cm \ Area pressed in 500 revolutions
21
= 500 × curved surface area of road roller
Volume of sphere is 179.67 cubic cm
= 500 × 2prh
(7) Total surface area of a cone is 616 sq. cm. If 22
the slant height of the cone is three times the = 500 × 2 × × 0.7 × 2.1
7
radius of its base, find its slant height. 22 7 21
solution: = 5×2× × ×
7 10 10
For a cone, l = 3r ..... (i) (Given)
Area pressed in 500 revolutions = 4620 sq.m
Total surface area of a cone =pr (r + l)
Cost of pressing the ground = Area × Rate
\ 616 =pr × (r + 3r) (From (i))
= 4620 × 7
22
616 = × r × 4r = ` 32340
7
616 × 7 The cost of levelling the ground is ` 32340
\ = r2
22 × 4
MCQ’s
\ r2 = 49
\ r = 7 cm (Taking square roots) (1) The area of the four walls of a room is 80cm2 and
l = 3r its height is 4 m. Then the perimeter of the floor
\ l = 3×7 of the room is ..................... .
(A) 16 m (B) 5 m (C) 20 m (D) 10 m
\ l = 21 cm
(2) The capacity of a cuboidal tank is 50,000 litres of
Slant height of the cone is 21 cm water. Find the breadth of the tank, if its length
and depth are respectively 2.5 m and 10 m.
(8) The inner diameter of a well is 4.20 metre and (A) 3 m (B) 2 m (C) 4 m (D) 2.5 m
its depth is 10 metre. Find the inner surface
(3) Volume of a right circular cone of height 14 cm is
area of the well. Find the cost of plastering if
168p cm3. The radius of the cone is ..................... .
from inside at the rate ` 52 per sq. m.
solution: (A) 6 cm (B) 12 cm (C) 8 cm (D) 10 cm
Internal diameters of cylindrical well = 4.20 m (4) The ratio of the volumes of the two spheres is
\ Internal radius of cylindrical well (r) = 2.10 m 1 : 27. The ratio of their radii is ..................... .
Depth of the cylindrical well (h) = 10 m, (A) 1 : 3 (B) 1 : 9 (C) 3 : 1 (D) 1 : 27
Internal curved surface area (Sc) = 2prh (5) The volume and curved surface of a sphere are
(Formula) numerically equal. The radius of the sphere is
22 ..................... .
= 2× × 2.10 × 10
7 (A) 1 cm (B) 3 cm (C) 4 cm (D) 7 cm
\ = 132 sq. m. 3
Cost of repairing the well = Area × Rate (6) In a cylinder, radius is doubled and height is
128 Master Key Mathematics - II (Geometry) (Std. IX)

halved, curved surface area will be ..................... . (12) If the sum of the all the edges of a cube is 36 cm,
(a) halved (b) doubled then the volume of the cube is ..................... .
(c) same (d) four times (A) 9 cm3 (B) 27 cm3 (C) 216 cm3 (D) 1728 cm3
(7) For which solid figure is the TSA same as its (13) Three solid cubes of side 4 cm each are joined
CSA? to form a cuboid. Then its surface area is
(A) Cone (B) Sphere ..................... .
(C) Cylinder (D) Hemisphere (A) 285 cm2 (B) 200 cm2(C) 192 cm2 (D) 224cm2
(8) A semi-circle is folded to form a cone. The radius (14) If height of a right circular cylinder is 15 cm and
will form ..................... . volume is 2310 cm3, then radius of the base will
(A) height of the cone be ..................... .
(B) slant height of the cone (A) 7 cm (B) 10 cm (C) 14 cm (D) 12 cm
(C) circumference of the base (15) The base radii of the two cones are the same but
(D) half of the circumference of the base their volumes are 4p m3 and 9p m3 respectively.
(9) If the height of a cone is doubled then its volume The ratio of their heights is ..................... .
is increased by ..................... . (A) 2: 3 (B) 2 : 3
(A) 100% (B) 200% (C) 300% (D) 400% (C) 4 : 9 (D) 16 : 81
(10) If A, B and C denote the areas of three adjacent
faces of a cuboid, then its volume is ..................... .
answer
(A) ABC (B) 2ABC
(C) ABC (D) A + B + C (1) (C) (2) (B) (3) (A) (4) (A)
(11) A well of diameter 8 m has been dug out to a (5) (B) (6) (C) (7) (B) (8) (B)
depth of 21 m. The volume of earth (in m3) dug
(9) (B) (10) (C) (11) (B) (12) (B)
out is ..................... .
(13) (D) (14) (A) (15) (C)
(A) 4224 (B) 1056 (C) 528 (D) 2112

aSSIGNmeNT - 9
Time : 1 Hr. marks : 20

Q.1. Solve the following: 2


(1) Find slant height of a cone, if its base radius is 12 cm and height is 16 cm.
(2) Area of the base of a cylinder is 154 cm2 and its height is 7 cm. Find its volume.
Q.2. Solve the following: 4
(1) Total surface area of a box of cuboid shape is 500 sq. units. Its breadth and height is 6 units and
5 units respectively What is the length of that box?
(2) Find the radius of a sphere whose volume is 113040 cubic cm. (p= 3.14)
Q.3. Solve the following: 6
(1) Total surface area of a cone is 616 sq. cm. If the slant height of the cone is three times the radius
of its base, find its slant height.
(2) The inner diameter of a well is 4.2 metre and its depth is 10 metre. Find the inner surface area
of the well. Find the cost of plastering it from inside at the rate ` 52 per sq. m.
Q.4. Solve the following: 8
(1) There is 25 persons in a tent which is conical in shape. Every person needs an area of 4 sq. m of
the ground inside the tent. If height of the tent is 18 m, find the volume of the tent.
(2) The length of a road roller is 21 m and its diameter is 1.4 m. For levelling a ground 500 rotations
of the road roller were required. How much area of ground was levelled by the road roller? Find
the cost of levelling at the rate of ` 7 per sq. m.

vvv

You might also like